Is low-dose naltrexone effective in chronic pain management?

Article Type
Changed
Wed, 09/13/2023 - 10:50
Display Headline
Is low-dose naltrexone effective in chronic pain management?

Evidence summary

Naltrexone is comparable to amitriptyline for diabetic neuropathy pain

A 2021 randomized, double-blind, active-comparator, crossover clinical trial conducted in India examined the efficacy of low-dose naltrexone vs standard-of-care amitriptyline in patients (N = 67) with painful diabetic neuropathy. Participants were adults (ages 18 to 75 years) with painful diabetic neuropathy who had been on a stable dose of nonopioid pain medication for at least 1 month.1

Patients were randomly assigned to start receiving naltrexone 2 mg (n = 33) or amitriptyline 10 mg (n = 34). They received their starting medication for 6 weeks (with follow-up every 2 weeks), then completed a 2-week washout period, and then switched to the other study medication for 6 weeks (same follow-up schedule). If patients reported < 20% pain reduction on the Visual Analog Scale (VAS; 0-100 scoring system with 0 = no pain and 100 = worst pain) at a follow-up visit, their medication dose was titrated up, to a maximum of 4 mg of naltrexone or 25 to 50 mg of amitriptyline.1

The primary outcome of interest was the mean change in VAS pain score following 6 weeks of treatment. There was no statistically different change from baseline VAS pain score between the amitriptyline and nal­trexone groups (mean difference [MD] = 1.6; 95% CI, –0.9 to 4.2; P = 0.21). These findings were consistent across the secondary endpoints (Likert 5-point pain scale and McGill Pain Questionnaire scores). There was no statistically significant difference in Hamilton Depression Rating Scale scores (13 in the naltrexone group vs 11 in the amitriptyline group; P = .81), no reports of decreased sleep quality in either group, and no significant difference in Patients’ Global Impression of Change scores at 6-week evaluation.1

The naltrexone cohort experienced 8 adverse events (most commonly, mild diarrhea), while the amitriptyline cohort experienced 52 adverse events (most commonly, somnolence) (P < .001). The limitations of the study include the lack of a placebo arm and a relatively small sample size.1

Greater reduction in pain scores with naltrexone

A 2022 retrospective cohort study evaluated the effectiveness of naltrexone for patients treated at a single outpatient integrative pain management practice in Alaska between 2014 and 2019. The exposure group (n = 36) included patients who had completed at least a 2-month continuous regimen of oral naltrexone 4.5 mg. Controls (n = 42) were selected from the remaining practice population receiving standard care and were primarily matched by diagnosis code, followed by gender, then age +/– 5 years. Patients were divided into subgroups for inflammatory and neuropathic pain.2

The primary outcome measured was the mean change in VAS score or numeric rating score (NRS; both used a 1-10 rating system), which was assessed during a patient’s appointment from initiation of treatment to the most recent visit or at the termination of therapy (intervention interquartile range, 12-14 months). There was no statistically significant difference in VAS/NRS between the low-dose naltrexone and control groups at baseline (6.09 vs 6.38; P = .454). The low-dose naltrexone group experienced a greater reduction in VAS/NRS pain scores compared to the control group (–37.8% vs –4.3%; P < .001).2

Compared with control patients in each group, patients in the inflammatory pain subgroup and the neuropathic pain subgroup who received low-dose naltrexone reported reductions in pain scores of 32% (P < .001) and 44% (P = .048), respectively. There was no statistically significant difference in mean change in VAS/NRS scores between the inflammatory and neuropathic subgroups (P = .763). A multivariate linear regression analysis did not identify significant variables other than low-dose naltrexone that correlated with pain improvement. The number needed to treat to observe a ≥ 50% reduction in pain scores was 3.2.2

Continue to: Limitations for this study...

 

 

Limitations for this study include its small sample size and open-label design.2

Low-dose naltrexone is effective for fibromyalgia pain

A 2020 single-blind prospective dose-­response study utilized the up-and-down method to identify effective naltrexone dose for patients in a Danish university hospital pain clinic. Patients were White women ages 18 to 60 years (N = 25) who had a diagnosis of fibromyalgia unresponsive to traditional pharmacologic treatment. All patients received treatment with low-dose naltrexone (ranging from 0.75 mg to 6.0 mg) but were blinded to dose.3

Studies show that low-dose naltrexone has some effectiveness in a variety of pain conditions—including diabetic neuropathy and fibromyalgia—with few adverse effects.

Patients were evaluated for improvement in fibromyalgia symptoms using the Patient Global Impression of Improvement (PGI-I) scale—which ranges from 1 (very much improved) to 7 (very much worse), with 4 being “no change”—at baseline and after 2 to 3 weeks of treatment with low-dose naltrexone. A patient was considered a responder if they scored 1 to 3 on the follow-up PGI-I scale or if they experienced a > 30% pain reduction on the VAS. If a patient did not respond to their dose, the next patient began treatment at a dose 0.75 mg higher than the previous patient’s ending dose. If a patient did respond to low-dose naltrexone treatment, the next patient’s starting dose was 0.75 mg less than the previous patient’s. Eleven of 25 patients were considered responders.3

 

The primary outcomes were effective dose for 50% of fibromyalgia patients (3.88 mg; 95% CI, 3.39-4.35) and effective dose for 95% of fibromyalgia patients (5.4 mg; 95% CI, 4.66-6.13). Secondary outcomes were fibromyalgia symptoms as evaluated on the Fibromyalgia Impact Questionnaire Revised. Five of the 11 responders reported a > 30% improvement in tenderness and 8 of the 11 responders reported a > 30% decrease in waking unrefreshed.3

Limitations of the study include the short time period of treatment before response was assessed and the decision to use low test ­doses, which may have hindered detection of effective doses > 6 mg in fibromyalgia.3

Editor’s takeaway

Low-dose naltrexone, a less-often-used form of pain management, is a welcome option. Studies show some effectiveness in a variety of pain conditions with few adverse effects. The small number of studies, the small sample sizes, and the limited follow-up duration should encourage more investigation into how to best use this intervention.

References

1. Srinivasan A, Dutta P, Bansal D, et al. Efficacy and safety of low-dose naltrexone in painful diabetic neuropathy: a randomized, double-blind, active-control, crossover clinical trial. J Diabetes. 2021;13:770-778. doi: 10.1111/1753-0407.13202

2. Martin SJ, McAnally HB, Okediji P, et al. Low-dose naltrexone, an opioid-receptor antagonist, is a broad-spectrum analgesic: a retrospective cohort study. Pain Management. 2022;12:699-709. doi: 10.2217/pmt-2021-0122

3. Bruun-Plesner K, Blichfeldt-Eckhardt MR, Vaegter HB, et al. Low-dose naltrexone for the treatment of fibromyalgia: investigation of dose-response relationships. Pain Med. 2020;21:2253-2261. doi: 10.1093/pm/pnaa001

Article PDF
Author and Disclosure Information

Roxanne Radi, MD, MPH
Harriet Huang, MD
Jason Rivera, MD
Corey Lyon, DO

University of Colorado Family Medicine Residency, Denver

Kristen DeSanto, MSLS, MS, RD
University of Colorado Health Sciences Library, Denver

DEPUTY EDITOR
Rick Guthmann, MD, MPH

Advocate Health Care Illinois Masonic Medical Center Program, Chicago

Issue
The Journal of Family Practice - 72(7)
Publications
Topics
Page Number
320-321
Sections
Author and Disclosure Information

Roxanne Radi, MD, MPH
Harriet Huang, MD
Jason Rivera, MD
Corey Lyon, DO

University of Colorado Family Medicine Residency, Denver

Kristen DeSanto, MSLS, MS, RD
University of Colorado Health Sciences Library, Denver

DEPUTY EDITOR
Rick Guthmann, MD, MPH

Advocate Health Care Illinois Masonic Medical Center Program, Chicago

Author and Disclosure Information

Roxanne Radi, MD, MPH
Harriet Huang, MD
Jason Rivera, MD
Corey Lyon, DO

University of Colorado Family Medicine Residency, Denver

Kristen DeSanto, MSLS, MS, RD
University of Colorado Health Sciences Library, Denver

DEPUTY EDITOR
Rick Guthmann, MD, MPH

Advocate Health Care Illinois Masonic Medical Center Program, Chicago

Article PDF
Article PDF

Evidence summary

Naltrexone is comparable to amitriptyline for diabetic neuropathy pain

A 2021 randomized, double-blind, active-comparator, crossover clinical trial conducted in India examined the efficacy of low-dose naltrexone vs standard-of-care amitriptyline in patients (N = 67) with painful diabetic neuropathy. Participants were adults (ages 18 to 75 years) with painful diabetic neuropathy who had been on a stable dose of nonopioid pain medication for at least 1 month.1

Patients were randomly assigned to start receiving naltrexone 2 mg (n = 33) or amitriptyline 10 mg (n = 34). They received their starting medication for 6 weeks (with follow-up every 2 weeks), then completed a 2-week washout period, and then switched to the other study medication for 6 weeks (same follow-up schedule). If patients reported < 20% pain reduction on the Visual Analog Scale (VAS; 0-100 scoring system with 0 = no pain and 100 = worst pain) at a follow-up visit, their medication dose was titrated up, to a maximum of 4 mg of naltrexone or 25 to 50 mg of amitriptyline.1

The primary outcome of interest was the mean change in VAS pain score following 6 weeks of treatment. There was no statistically different change from baseline VAS pain score between the amitriptyline and nal­trexone groups (mean difference [MD] = 1.6; 95% CI, –0.9 to 4.2; P = 0.21). These findings were consistent across the secondary endpoints (Likert 5-point pain scale and McGill Pain Questionnaire scores). There was no statistically significant difference in Hamilton Depression Rating Scale scores (13 in the naltrexone group vs 11 in the amitriptyline group; P = .81), no reports of decreased sleep quality in either group, and no significant difference in Patients’ Global Impression of Change scores at 6-week evaluation.1

The naltrexone cohort experienced 8 adverse events (most commonly, mild diarrhea), while the amitriptyline cohort experienced 52 adverse events (most commonly, somnolence) (P < .001). The limitations of the study include the lack of a placebo arm and a relatively small sample size.1

Greater reduction in pain scores with naltrexone

A 2022 retrospective cohort study evaluated the effectiveness of naltrexone for patients treated at a single outpatient integrative pain management practice in Alaska between 2014 and 2019. The exposure group (n = 36) included patients who had completed at least a 2-month continuous regimen of oral naltrexone 4.5 mg. Controls (n = 42) were selected from the remaining practice population receiving standard care and were primarily matched by diagnosis code, followed by gender, then age +/– 5 years. Patients were divided into subgroups for inflammatory and neuropathic pain.2

The primary outcome measured was the mean change in VAS score or numeric rating score (NRS; both used a 1-10 rating system), which was assessed during a patient’s appointment from initiation of treatment to the most recent visit or at the termination of therapy (intervention interquartile range, 12-14 months). There was no statistically significant difference in VAS/NRS between the low-dose naltrexone and control groups at baseline (6.09 vs 6.38; P = .454). The low-dose naltrexone group experienced a greater reduction in VAS/NRS pain scores compared to the control group (–37.8% vs –4.3%; P < .001).2

Compared with control patients in each group, patients in the inflammatory pain subgroup and the neuropathic pain subgroup who received low-dose naltrexone reported reductions in pain scores of 32% (P < .001) and 44% (P = .048), respectively. There was no statistically significant difference in mean change in VAS/NRS scores between the inflammatory and neuropathic subgroups (P = .763). A multivariate linear regression analysis did not identify significant variables other than low-dose naltrexone that correlated with pain improvement. The number needed to treat to observe a ≥ 50% reduction in pain scores was 3.2.2

Continue to: Limitations for this study...

 

 

Limitations for this study include its small sample size and open-label design.2

Low-dose naltrexone is effective for fibromyalgia pain

A 2020 single-blind prospective dose-­response study utilized the up-and-down method to identify effective naltrexone dose for patients in a Danish university hospital pain clinic. Patients were White women ages 18 to 60 years (N = 25) who had a diagnosis of fibromyalgia unresponsive to traditional pharmacologic treatment. All patients received treatment with low-dose naltrexone (ranging from 0.75 mg to 6.0 mg) but were blinded to dose.3

Studies show that low-dose naltrexone has some effectiveness in a variety of pain conditions—including diabetic neuropathy and fibromyalgia—with few adverse effects.

Patients were evaluated for improvement in fibromyalgia symptoms using the Patient Global Impression of Improvement (PGI-I) scale—which ranges from 1 (very much improved) to 7 (very much worse), with 4 being “no change”—at baseline and after 2 to 3 weeks of treatment with low-dose naltrexone. A patient was considered a responder if they scored 1 to 3 on the follow-up PGI-I scale or if they experienced a > 30% pain reduction on the VAS. If a patient did not respond to their dose, the next patient began treatment at a dose 0.75 mg higher than the previous patient’s ending dose. If a patient did respond to low-dose naltrexone treatment, the next patient’s starting dose was 0.75 mg less than the previous patient’s. Eleven of 25 patients were considered responders.3

 

The primary outcomes were effective dose for 50% of fibromyalgia patients (3.88 mg; 95% CI, 3.39-4.35) and effective dose for 95% of fibromyalgia patients (5.4 mg; 95% CI, 4.66-6.13). Secondary outcomes were fibromyalgia symptoms as evaluated on the Fibromyalgia Impact Questionnaire Revised. Five of the 11 responders reported a > 30% improvement in tenderness and 8 of the 11 responders reported a > 30% decrease in waking unrefreshed.3

Limitations of the study include the short time period of treatment before response was assessed and the decision to use low test ­doses, which may have hindered detection of effective doses > 6 mg in fibromyalgia.3

Editor’s takeaway

Low-dose naltrexone, a less-often-used form of pain management, is a welcome option. Studies show some effectiveness in a variety of pain conditions with few adverse effects. The small number of studies, the small sample sizes, and the limited follow-up duration should encourage more investigation into how to best use this intervention.

Evidence summary

Naltrexone is comparable to amitriptyline for diabetic neuropathy pain

A 2021 randomized, double-blind, active-comparator, crossover clinical trial conducted in India examined the efficacy of low-dose naltrexone vs standard-of-care amitriptyline in patients (N = 67) with painful diabetic neuropathy. Participants were adults (ages 18 to 75 years) with painful diabetic neuropathy who had been on a stable dose of nonopioid pain medication for at least 1 month.1

Patients were randomly assigned to start receiving naltrexone 2 mg (n = 33) or amitriptyline 10 mg (n = 34). They received their starting medication for 6 weeks (with follow-up every 2 weeks), then completed a 2-week washout period, and then switched to the other study medication for 6 weeks (same follow-up schedule). If patients reported < 20% pain reduction on the Visual Analog Scale (VAS; 0-100 scoring system with 0 = no pain and 100 = worst pain) at a follow-up visit, their medication dose was titrated up, to a maximum of 4 mg of naltrexone or 25 to 50 mg of amitriptyline.1

The primary outcome of interest was the mean change in VAS pain score following 6 weeks of treatment. There was no statistically different change from baseline VAS pain score between the amitriptyline and nal­trexone groups (mean difference [MD] = 1.6; 95% CI, –0.9 to 4.2; P = 0.21). These findings were consistent across the secondary endpoints (Likert 5-point pain scale and McGill Pain Questionnaire scores). There was no statistically significant difference in Hamilton Depression Rating Scale scores (13 in the naltrexone group vs 11 in the amitriptyline group; P = .81), no reports of decreased sleep quality in either group, and no significant difference in Patients’ Global Impression of Change scores at 6-week evaluation.1

The naltrexone cohort experienced 8 adverse events (most commonly, mild diarrhea), while the amitriptyline cohort experienced 52 adverse events (most commonly, somnolence) (P < .001). The limitations of the study include the lack of a placebo arm and a relatively small sample size.1

Greater reduction in pain scores with naltrexone

A 2022 retrospective cohort study evaluated the effectiveness of naltrexone for patients treated at a single outpatient integrative pain management practice in Alaska between 2014 and 2019. The exposure group (n = 36) included patients who had completed at least a 2-month continuous regimen of oral naltrexone 4.5 mg. Controls (n = 42) were selected from the remaining practice population receiving standard care and were primarily matched by diagnosis code, followed by gender, then age +/– 5 years. Patients were divided into subgroups for inflammatory and neuropathic pain.2

The primary outcome measured was the mean change in VAS score or numeric rating score (NRS; both used a 1-10 rating system), which was assessed during a patient’s appointment from initiation of treatment to the most recent visit or at the termination of therapy (intervention interquartile range, 12-14 months). There was no statistically significant difference in VAS/NRS between the low-dose naltrexone and control groups at baseline (6.09 vs 6.38; P = .454). The low-dose naltrexone group experienced a greater reduction in VAS/NRS pain scores compared to the control group (–37.8% vs –4.3%; P < .001).2

Compared with control patients in each group, patients in the inflammatory pain subgroup and the neuropathic pain subgroup who received low-dose naltrexone reported reductions in pain scores of 32% (P < .001) and 44% (P = .048), respectively. There was no statistically significant difference in mean change in VAS/NRS scores between the inflammatory and neuropathic subgroups (P = .763). A multivariate linear regression analysis did not identify significant variables other than low-dose naltrexone that correlated with pain improvement. The number needed to treat to observe a ≥ 50% reduction in pain scores was 3.2.2

Continue to: Limitations for this study...

 

 

Limitations for this study include its small sample size and open-label design.2

Low-dose naltrexone is effective for fibromyalgia pain

A 2020 single-blind prospective dose-­response study utilized the up-and-down method to identify effective naltrexone dose for patients in a Danish university hospital pain clinic. Patients were White women ages 18 to 60 years (N = 25) who had a diagnosis of fibromyalgia unresponsive to traditional pharmacologic treatment. All patients received treatment with low-dose naltrexone (ranging from 0.75 mg to 6.0 mg) but were blinded to dose.3

Studies show that low-dose naltrexone has some effectiveness in a variety of pain conditions—including diabetic neuropathy and fibromyalgia—with few adverse effects.

Patients were evaluated for improvement in fibromyalgia symptoms using the Patient Global Impression of Improvement (PGI-I) scale—which ranges from 1 (very much improved) to 7 (very much worse), with 4 being “no change”—at baseline and after 2 to 3 weeks of treatment with low-dose naltrexone. A patient was considered a responder if they scored 1 to 3 on the follow-up PGI-I scale or if they experienced a > 30% pain reduction on the VAS. If a patient did not respond to their dose, the next patient began treatment at a dose 0.75 mg higher than the previous patient’s ending dose. If a patient did respond to low-dose naltrexone treatment, the next patient’s starting dose was 0.75 mg less than the previous patient’s. Eleven of 25 patients were considered responders.3

 

The primary outcomes were effective dose for 50% of fibromyalgia patients (3.88 mg; 95% CI, 3.39-4.35) and effective dose for 95% of fibromyalgia patients (5.4 mg; 95% CI, 4.66-6.13). Secondary outcomes were fibromyalgia symptoms as evaluated on the Fibromyalgia Impact Questionnaire Revised. Five of the 11 responders reported a > 30% improvement in tenderness and 8 of the 11 responders reported a > 30% decrease in waking unrefreshed.3

Limitations of the study include the short time period of treatment before response was assessed and the decision to use low test ­doses, which may have hindered detection of effective doses > 6 mg in fibromyalgia.3

Editor’s takeaway

Low-dose naltrexone, a less-often-used form of pain management, is a welcome option. Studies show some effectiveness in a variety of pain conditions with few adverse effects. The small number of studies, the small sample sizes, and the limited follow-up duration should encourage more investigation into how to best use this intervention.

References

1. Srinivasan A, Dutta P, Bansal D, et al. Efficacy and safety of low-dose naltrexone in painful diabetic neuropathy: a randomized, double-blind, active-control, crossover clinical trial. J Diabetes. 2021;13:770-778. doi: 10.1111/1753-0407.13202

2. Martin SJ, McAnally HB, Okediji P, et al. Low-dose naltrexone, an opioid-receptor antagonist, is a broad-spectrum analgesic: a retrospective cohort study. Pain Management. 2022;12:699-709. doi: 10.2217/pmt-2021-0122

3. Bruun-Plesner K, Blichfeldt-Eckhardt MR, Vaegter HB, et al. Low-dose naltrexone for the treatment of fibromyalgia: investigation of dose-response relationships. Pain Med. 2020;21:2253-2261. doi: 10.1093/pm/pnaa001

References

1. Srinivasan A, Dutta P, Bansal D, et al. Efficacy and safety of low-dose naltrexone in painful diabetic neuropathy: a randomized, double-blind, active-control, crossover clinical trial. J Diabetes. 2021;13:770-778. doi: 10.1111/1753-0407.13202

2. Martin SJ, McAnally HB, Okediji P, et al. Low-dose naltrexone, an opioid-receptor antagonist, is a broad-spectrum analgesic: a retrospective cohort study. Pain Management. 2022;12:699-709. doi: 10.2217/pmt-2021-0122

3. Bruun-Plesner K, Blichfeldt-Eckhardt MR, Vaegter HB, et al. Low-dose naltrexone for the treatment of fibromyalgia: investigation of dose-response relationships. Pain Med. 2020;21:2253-2261. doi: 10.1093/pm/pnaa001

Issue
The Journal of Family Practice - 72(7)
Issue
The Journal of Family Practice - 72(7)
Page Number
320-321
Page Number
320-321
Publications
Publications
Topics
Article Type
Display Headline
Is low-dose naltrexone effective in chronic pain management?
Display Headline
Is low-dose naltrexone effective in chronic pain management?
Sections
Teambase XML
<?xml version="1.0" encoding="UTF-8"?>
<!--$RCSfile: InCopy_agile.xsl,v $ $Revision: 1.35 $-->
<!--$RCSfile: drupal.xsl,v $ $Revision: 1.7 $-->
<root generator="drupal.xsl" gversion="1.7"> <header> <fileName>CI0923 (Naltrexone)</fileName> <TBEID>0C02DE46.SIG</TBEID> <TBUniqueIdentifier>NJ_0C02DE46</TBUniqueIdentifier> <newsOrJournal>Journal</newsOrJournal> <publisherName>Frontline Medical Communications Inc.</publisherName> <storyname>Is low-dose naltrexone effective</storyname> <articleType>1</articleType> <TBLocation>Copyfitting-JFP</TBLocation> <QCDate/> <firstPublished>20230912T084829</firstPublished> <LastPublished>20230912T084829</LastPublished> <pubStatus qcode="stat:"/> <embargoDate/> <killDate/> <CMSDate>20230912T084829</CMSDate> <articleSource/> <facebookInfo/> <meetingNumber/> <byline>Roxanne Radi, MD, MPH;&#13;Harriet Huang, MD;&#13;Jason Rivera, MD</byline> <bylineText/> <bylineFull>Roxanne Radi, MD, MPH;&#13;Harriet Huang, MD;&#13;Jason Rivera, MD</bylineFull> <bylineTitleText/> <USOrGlobal/> <wireDocType/> <newsDocType/> <journalDocType/> <linkLabel/> <pageRange>320-321</pageRange> <citation/> <quizID/> <indexIssueDate/> <itemClass qcode="ninat:text"/> <provider qcode="provider:"> <name/> <rightsInfo> <copyrightHolder> <name/> </copyrightHolder> <copyrightNotice/> </rightsInfo> </provider> <abstract/> <metaDescription>YES. Low-dose naltrexone is as effective as amitriptyline in the treatment of painful diabetic neuropathy and has a superior safety profile (strength of recomme</metaDescription> <articlePDF>297421</articlePDF> <teaserImage/> <title>Q Is low-dose naltrexone effective in chronic pain management?</title> <deck/> <disclaimer/> <AuthorList/> <articleURL/> <doi/> <pubMedID/> <publishXMLStatus/> <publishXMLVersion>1</publishXMLVersion> <useEISSN>0</useEISSN> <urgency/> <pubPubdateYear>2023</pubPubdateYear> <pubPubdateMonth>September</pubPubdateMonth> <pubPubdateDay/> <pubVolume>72</pubVolume> <pubNumber>7</pubNumber> <wireChannels/> <primaryCMSID/> <CMSIDs> <CMSID>3171</CMSID> </CMSIDs> <keywords> <keyword>pain</keyword> <keyword> diabetes</keyword> <keyword> chronic pain</keyword> </keywords> <seeAlsos/> <publications_g> <publicationData> <publicationCode>jfp</publicationCode> <pubIssueName>September 2023</pubIssueName> <pubArticleType>Clinical Inquiries | 3171</pubArticleType> <pubTopics/> <pubCategories/> <pubSections/> </publicationData> <publicationData> <publicationCode>mdfam</publicationCode> <pubIssueName/> <pubArticleType/> <pubTopics/> <pubCategories/> <pubSections/> <journalTitle/> <journalFullTitle/> <copyrightStatement/> </publicationData> </publications_g> <publications> <term canonical="true">30</term> <term>51948</term> </publications> <sections> <term canonical="true">27414</term> </sections> <topics> <term>205</term> <term canonical="true">68340</term> <term>27442</term> </topics> <links> <link> <itemClass qcode="ninat:composite"/> <altRep contenttype="application/pdf">images/1800257a.pdf</altRep> <description role="drol:caption"/> <description role="drol:credit"/> </link> </links> </header> <itemSet> <newsItem> <itemMeta> <itemRole>Main</itemRole> <itemClass>text</itemClass> <title>Q Is low-dose naltrexone effective in chronic pain management?</title> <deck/> </itemMeta> <itemContent> <p><b> </b><b>YES.</b> Low-dose naltrexone is as effective as amitriptyline in the treatment of painful diabetic neuropathy and has a superior safety profile (strength of recommendation [SOR], <b>B</b>; single randomized controlled trial [RCT]). </p> <p>Low-dose naltrexone significantly reduced pain by 32% in inflammatory conditions and 44% in neuropathic conditions (SOR, <b>B</b>; single retrospective cohort study). <br/><br/>Doses as low as 5.4 mg were found to reduce pain in 95% of patients with fibromyalgia (SOR, <b>B</b>; single prospective dose-response study).</p> <h3>Evidence summary</h3> <p class="sub1">Naltrexone is comparable to amitriptyline for diabetic neuropathy pain</p> <p>A 2021 randomized, double-blind, active-comparator, crossover clinical trial conducted in India examined the efficacy of low-dose naltrexone vs standard-of-care amitriptyline in patients (N = 67) with painful diabetic neuropathy. Participants were adults (ages 18 to 75 years) with painful diabetic neuropathy who had been on a stable dose of nonopioid pain medication for at least 1 month.<sup>1</sup> </p> <p>Patients were randomly assigned to start receiving naltrexone 2 mg (n = 33) or amitriptyline 10 mg (n = 34). They received their starting medication for 6 weeks (with follow-up every 2 weeks), then completed a 2-week washout period, and then switched to the other study medication for 6 weeks (same follow-up schedule). If patients reported &lt; 20% pain reduction on the Visual Analog Scale (VAS; 0-100 scoring system with 0 = no pain and 100 = worst pain) at a follow-up visit, their medication dose was titrated up, to a maximum of 4 mg of naltrexone or 25 to 50 mg of amitriptyline.<sup>1</sup> <br/><br/>The primary outcome of interest was the mean change in VAS pain score following 6 weeks of treatment. There was no statistically different change from baseline VAS pain score between the amitriptyline and nal­trexone groups (mean difference [MD] = 1.6; 95% CI, –0.9 to 4.2; <i>P</i> = 0.21). These findings were consistent across the secondary endpoints (Likert 5-point pain scale and McGill Pain Questionnaire scores). There was no statistically significant difference in Hamilton Depression Rating Scale scores (13 in the naltrexone group vs 11 in the amitriptyline group; <i>P</i> = .81), no reports of decreased sleep quality in either group, and no significant difference in Patients’ Global Impression of Change scores at 6-week evaluation.<sup>1</sup> <br/><br/>The naltrexone cohort experienced 8 adverse events (most commonly, mild diarrhea), while the amitriptyline cohort experienced 52 adverse events (most commonly, somnolence) (<i>P</i> &lt; .001). The limitations of the study include the lack of a placebo arm and a relatively small sample size.<sup>1</sup> </p> <p class="sub1">Greater reduction in pain scoreswith naltrexone</p> <p>A 2022 retrospective cohort study evaluated the effectiveness of naltrexone for patients treated at a single outpatient integrative pain management practice in Alaska between 2014 and 2019. The exposure group (n = 36) included patients who had completed at least a 2-month continuous regimen of oral naltrexone 4.5 mg. Controls (n = 42) were selected from the remaining practice population receiving standard care and were primarily matched by diagnosis code, followed by gender, then age +/– 5 years. Patients were divided into subgroups for inflammatory and neuropathic pain.<sup>2</sup> </p> <p>The primary outcome measured was the mean change in VAS score or numeric rating score (NRS; both used a 1-10 rating system), which was assessed during a patient’s appointment from initiation of treatment to the most recent visit or at the termination of therapy (intervention interquartile range, 12-14 months). There was no statistically significant difference in VAS/NRS between the low-dose naltrexone and control groups at baseline (6.09 vs 6.38; <i>P</i> = .454). The low-dose naltrexone group experienced a greater reduction in VAS/NRS pain scores compared to the control group (–37.8% vs –4.3%; <i>P</i> &lt; .001).<sup>2</sup> <br/><br/>Compared with control patients in each group, patients in the inflammatory pain subgroup and the neuropathic pain subgroup who received low-dose naltrexone reported reductions in pain scores of 32% (<i>P</i> &lt; .001) and 44% (<i>P</i> = .048), respectively. There was no statistically significant difference in mean change in VAS/NRS scores between the inflammatory and neuropathic subgroups (<i>P</i> = .763). A multivariate linear regression analysis did not identify significant variables other than low-dose naltrexone that correlated with pain improvement. The number needed to treat to observe a ≥ 50% reduction in pain scores was 3.2.<sup>2</sup> <br/><br/>Limitations for this study include its small sample size and open-label design.<sup>2</sup></p> <p class="sub1">Low-dose naltrexone is effective for fibromyalgia pain</p> <p>A 2020 single-blind prospective dose-­response study utilized the up-and-down method to identify effective naltrexone dose for patients in a Danish university hospital pain clinic. Patients were White women ages 18 to 60 years (N = 25) who had a diagnosis of fibromyalgia unresponsive to traditional pharmacologic treatment. All patients received treatment with low-dose naltrexone (ranging from 0.75 mg to 6.0 mg) but were blinded to dose.<sup>3</sup> </p> <p>Patients were evaluated for improvement in fibromyalgia symptoms using the Patient Global Impression of Improvement (PGI-I) scale—which ranges from 1 (very much improved) to 7 (very much worse), with 4 being “no change”—at baseline and after 2 to 3 weeks of treatment with low-dose naltrexone. A patient was considered a responder if they scored 1 to 3 on the follow-up PGI-I scale or if they experienced a &gt; 30% pain reduction on the VAS. If a patient did not respond to their dose, the next patient began treatment at a dose 0.75 mg higher than the previous patient’s ending dose. If a patient did respond to low-dose naltrexone treatment, the next patient’s starting dose was 0.75 mg less than the previous patient’s. Eleven of 25 patients were considered responders.<sup>3</sup> <br/><br/>The primary outcomes were effective dose for 50% of fibromyalgia patients (3.88 mg; 95% CI, 3.39-4.35) and effective dose for 95% of fibromyalgia patients (5.4 mg; 95% CI, 4.66-6.13). Secondary outcomes were fibromyalgia symptoms as evaluated on the Fibromyalgia Impact Questionnaire Revised. Five of the 11 responders reported a &gt; 30% improvement in tenderness and 8 of the 11 responders reported a &gt; 30% decrease in waking unrefreshed.<sup>3<br/><br/></sup>Limitations of the study include the short time period of treatment before response was assessed and the decision to use low test ­doses, which may have hindered detection of effective doses &gt; 6 mg in fibromyalgia.<sup>3</sup> </p> <h3>Editor’s takeaway</h3> <p>Low-dose naltrexone, a less-often-used form of pain management, is a welcome option. Studies show some effectiveness in a variety of pain conditions with few adverse effects. The small number of studies, the small sample sizes, and the limited follow-up duration should encourage more investigation into how to best use this intervention. <span class="end">JFP</span></p> <p class="reference"> 1. Srinivasan A, Dutta P, Bansal D, et al. Efficacy and safety of low-dose naltrexone in painful diabetic neuropathy: a randomized, double-blind, active-control, crossover clinical trial. <i>J Diabetes</i>. 2021;13:770-778. doi: 10.1111/1753-0407.13202<br/><br/> 2. Martin SJ, McAnally HB, Okediji P, et al. Low-dose naltrexone, an opioid-receptor antagonist, is a broad-spectrum analgesic: a retrospective cohort study. <i>Pain Management. </i>2022;12:699-709. doi: 10.2217/pmt-2021-0122 <br/><br/> 3. Bruun-Plesner K, Blichfeldt-Eckhardt MR, Vaegter HB, et al. Low-dose naltrexone for the treatment of fibromyalgia: investigation of dose-response relationships<i>. Pain Med</i>. 2020;21:2253-2261. doi: 10.1093/pm/pnaa001 </p> </itemContent> </newsItem> </itemSet></root>
PURLs Copyright
Evidence-based answers from the Family Physicians Inquiries Network
Inside the Article

EVIDENCE-BASED ANSWER:

YES. Low-dose naltrexone is as effective as amitriptyline in the treatment of painful diabetic neuropathy and has a superior safety profile (strength of recommendation [SOR], B; single randomized controlled trial [RCT]).

Low-dose naltrexone significantly reduced pain by 32% in inflammatory conditions and 44% in neuropathic conditions (SOR, B; single retrospective cohort study).

Doses as low as 5.4 mg were found to reduce pain in 95% of patients with fibromyalgia (SOR, B; single prospective dose-response study).

Disallow All Ads
Content Gating
No Gating (article Unlocked/Free)
Alternative CME
Disqus Comments
Default
Use ProPublica
Hide sidebar & use full width
render the right sidebar.
Conference Recap Checkbox
Not Conference Recap
Clinical Edge
Display the Slideshow in this Article
Medscape Article
Display survey writer
Reuters content
Disable Inline Native ads
WebMD Article
Article PDF Media
Image
Teambase ID
1800257A.SIG
Disable zoom
Off

Does hormone replacement therapy prevent cognitive decline in postmenopausal women?

Article Type
Changed
Wed, 05/17/2023 - 07:34
Display Headline
Does hormone replacement therapy prevent cognitive decline in postmenopausal women?

Evidence summary

Multiple analyses suggest HRT worsens rather than improves cognition

A 2017 Cochrane review of 22 randomized, double-blind studies compared use of HRT (estrogen only or combination estrogen + progesterone therapies) with placebo in postmenopausal women (N = 43,637). Age ranges varied, but the average age in most studies was > 60 years. Treatment duration was at least 1 year. Various outcomes were assessed across these 22 studies, including cardiovascular disease, bone health, and cognition.1

Cognitive outcomes were assessed with the Mini-Mental Status Exam in 5 of the trials (N = 12,789). Data were not combined due to heterogeneity. The authors found no significant difference in cognitive scores between the treatment and control groups in any of these 5 studies.1

In the largest included study, the Women’s Health Initiative (WHI) Memory Study (N = 10,739), participants were older than 65 years. Among those receiving estrogen-only HRT, there were no statistically significant differences compared to those receiving placebo. However, healthy postmenopausal women taking combination HRT had an increased risk for “probable dementia” compared to those taking placebo (relative risk [RR] = 1.97; 95% CI, 1.16-3.33). When researchers looked exclusively at women taking HRT, the risk for dementia increased from 9 in 1000 to 18 in 1000 (95% CI, 11-30) after 4 years of HRT use. This results in a number needed to harm of 4 to 50 patients.1

Two notable limitations of this evidence are that the average age of this population was > 60 years and 80% of the participants were White.1

A 2021 meta-analysis of 23 RCTs (N = 13,683) studied the effect of HRT on global cognitive function as well as specific cognitive domains including memory, executive function, attention, and language. Mean patient age in the studies varied from 48 to 81 years. Nine of these studies were also included in the previously discussed Cochrane review.2

There was a statistically significant but small decrease in overall global cognition (10 trials; N = 12,115; standardized mean difference [SMD] = –0.04; 95% CI, –0.08 to –0.01) in those receiving HRT compared to placebo. This effect was slightly more pronounced among those who initiated HRT at age > 60 years (8 trials; N = 11,914; SMD = –0.05; 95% CI, –0.08 to –0.01) and among patients with HRT duration > 6 months (7 trials; N = 11,828; SMD = –0.05; 95% CI, –0.08 to –0.01). There were no significant differences in specific cognitive domains.2

In a 2017 follow-up to the WHI trial, researchers analyzed data on long-term cognitive effects in women previously treated with HRT. There were 2 cohorts: participants who initiated HRT at a younger age (50-54; N = 1376) and those who initiated HRT later in life (age 65-79; N = 2880). Cognitive outcomes were assessed using the Telephone Interview for Cognitive Status-modified, with interviews conducted annually beginning 6 to 7 years after HRT was stopped.3

The investigators found no significant change in composite cognitive function in the younger HRT-treated group compared to placebo (estrogen alone: mean deviation [MD] = 0.014; 95% CI, –0.097 to 0.126; estrogen + progesterone: MD = –0.047; 95% CI, –0.134 to 0.04), or in the group who initiated HRT at an older age (estrogen alone: MD = –0.099; 95% CI, –0.202 to 0.004; estrogen + progesterone: MD = –0.022; 95% CI, –0.099 to 0.055). The authors state that although the data did not reach significance, this study also found a trend toward decreases in global cognitive function in the older age group.3

Editor’s takeaway

Abundant, consistent evidence with long-term follow-up shows postmenopausal HRT does not reduce cognitive decline. In fact, it appears to increase cognitive decline slightly. Renewed interest in postmenopausal HRT to alleviate menopausal symptoms should balance the risks and benefits to the individual patient.

References

1. Marjoribanks J, Farquhar C, Roberts H, et al. Long-term hormone therapy for perimenopausal and postmenopausal women. Cochrane Database Syst Rev. 2017;1:CD004143. doi: 10.1002/14651858.CD004143.pub5

2. Zhou HH, Yu Z, Luo L, et al. The effect of hormone replacement therapy on cognitive function in healthy postmenopausal women: a meta-analysis of 23 randomized controlled trials. Psychogeriatrics. 2021;21:926-938. doi: 10.1111/psyg.12768

3. Espeland MA, Rapp SR, Manson JE, et al. Long-term effects on cognitive trajectories of postmenopausal hormone therapy in two age groups. J Gerontol A Biol Sci Med Sci. 2017;72:838-845. doi: 10.1093/gerona/glw156

Article PDF
Author and Disclosure Information

Madeline Gates, MD
Melissa Beagle, MD, MPH
Lauren Bull, MD
Roxanne Radi, MD, MPH
Corey Lyon, DO

University of Colorado Family Medicine Residency, Denver

Kristen DeSanto, MSLS, MS, RD
University of Colorado Health Sciences Library, Denver

DEPUTY EDITOR
Rick Guthmann, MD, MPH

Advocate Health Care Illinois Masonic Medical Center Program, Chicago

Issue
The Journal of Family Practice - 72(4)
Publications
Topics
Page Number
E10-E11
Sections
Author and Disclosure Information

Madeline Gates, MD
Melissa Beagle, MD, MPH
Lauren Bull, MD
Roxanne Radi, MD, MPH
Corey Lyon, DO

University of Colorado Family Medicine Residency, Denver

Kristen DeSanto, MSLS, MS, RD
University of Colorado Health Sciences Library, Denver

DEPUTY EDITOR
Rick Guthmann, MD, MPH

Advocate Health Care Illinois Masonic Medical Center Program, Chicago

Author and Disclosure Information

Madeline Gates, MD
Melissa Beagle, MD, MPH
Lauren Bull, MD
Roxanne Radi, MD, MPH
Corey Lyon, DO

University of Colorado Family Medicine Residency, Denver

Kristen DeSanto, MSLS, MS, RD
University of Colorado Health Sciences Library, Denver

DEPUTY EDITOR
Rick Guthmann, MD, MPH

Advocate Health Care Illinois Masonic Medical Center Program, Chicago

Article PDF
Article PDF

Evidence summary

Multiple analyses suggest HRT worsens rather than improves cognition

A 2017 Cochrane review of 22 randomized, double-blind studies compared use of HRT (estrogen only or combination estrogen + progesterone therapies) with placebo in postmenopausal women (N = 43,637). Age ranges varied, but the average age in most studies was > 60 years. Treatment duration was at least 1 year. Various outcomes were assessed across these 22 studies, including cardiovascular disease, bone health, and cognition.1

Cognitive outcomes were assessed with the Mini-Mental Status Exam in 5 of the trials (N = 12,789). Data were not combined due to heterogeneity. The authors found no significant difference in cognitive scores between the treatment and control groups in any of these 5 studies.1

In the largest included study, the Women’s Health Initiative (WHI) Memory Study (N = 10,739), participants were older than 65 years. Among those receiving estrogen-only HRT, there were no statistically significant differences compared to those receiving placebo. However, healthy postmenopausal women taking combination HRT had an increased risk for “probable dementia” compared to those taking placebo (relative risk [RR] = 1.97; 95% CI, 1.16-3.33). When researchers looked exclusively at women taking HRT, the risk for dementia increased from 9 in 1000 to 18 in 1000 (95% CI, 11-30) after 4 years of HRT use. This results in a number needed to harm of 4 to 50 patients.1

Two notable limitations of this evidence are that the average age of this population was > 60 years and 80% of the participants were White.1

A 2021 meta-analysis of 23 RCTs (N = 13,683) studied the effect of HRT on global cognitive function as well as specific cognitive domains including memory, executive function, attention, and language. Mean patient age in the studies varied from 48 to 81 years. Nine of these studies were also included in the previously discussed Cochrane review.2

There was a statistically significant but small decrease in overall global cognition (10 trials; N = 12,115; standardized mean difference [SMD] = –0.04; 95% CI, –0.08 to –0.01) in those receiving HRT compared to placebo. This effect was slightly more pronounced among those who initiated HRT at age > 60 years (8 trials; N = 11,914; SMD = –0.05; 95% CI, –0.08 to –0.01) and among patients with HRT duration > 6 months (7 trials; N = 11,828; SMD = –0.05; 95% CI, –0.08 to –0.01). There were no significant differences in specific cognitive domains.2

In a 2017 follow-up to the WHI trial, researchers analyzed data on long-term cognitive effects in women previously treated with HRT. There were 2 cohorts: participants who initiated HRT at a younger age (50-54; N = 1376) and those who initiated HRT later in life (age 65-79; N = 2880). Cognitive outcomes were assessed using the Telephone Interview for Cognitive Status-modified, with interviews conducted annually beginning 6 to 7 years after HRT was stopped.3

The investigators found no significant change in composite cognitive function in the younger HRT-treated group compared to placebo (estrogen alone: mean deviation [MD] = 0.014; 95% CI, –0.097 to 0.126; estrogen + progesterone: MD = –0.047; 95% CI, –0.134 to 0.04), or in the group who initiated HRT at an older age (estrogen alone: MD = –0.099; 95% CI, –0.202 to 0.004; estrogen + progesterone: MD = –0.022; 95% CI, –0.099 to 0.055). The authors state that although the data did not reach significance, this study also found a trend toward decreases in global cognitive function in the older age group.3

Editor’s takeaway

Abundant, consistent evidence with long-term follow-up shows postmenopausal HRT does not reduce cognitive decline. In fact, it appears to increase cognitive decline slightly. Renewed interest in postmenopausal HRT to alleviate menopausal symptoms should balance the risks and benefits to the individual patient.

Evidence summary

Multiple analyses suggest HRT worsens rather than improves cognition

A 2017 Cochrane review of 22 randomized, double-blind studies compared use of HRT (estrogen only or combination estrogen + progesterone therapies) with placebo in postmenopausal women (N = 43,637). Age ranges varied, but the average age in most studies was > 60 years. Treatment duration was at least 1 year. Various outcomes were assessed across these 22 studies, including cardiovascular disease, bone health, and cognition.1

Cognitive outcomes were assessed with the Mini-Mental Status Exam in 5 of the trials (N = 12,789). Data were not combined due to heterogeneity. The authors found no significant difference in cognitive scores between the treatment and control groups in any of these 5 studies.1

In the largest included study, the Women’s Health Initiative (WHI) Memory Study (N = 10,739), participants were older than 65 years. Among those receiving estrogen-only HRT, there were no statistically significant differences compared to those receiving placebo. However, healthy postmenopausal women taking combination HRT had an increased risk for “probable dementia” compared to those taking placebo (relative risk [RR] = 1.97; 95% CI, 1.16-3.33). When researchers looked exclusively at women taking HRT, the risk for dementia increased from 9 in 1000 to 18 in 1000 (95% CI, 11-30) after 4 years of HRT use. This results in a number needed to harm of 4 to 50 patients.1

Two notable limitations of this evidence are that the average age of this population was > 60 years and 80% of the participants were White.1

A 2021 meta-analysis of 23 RCTs (N = 13,683) studied the effect of HRT on global cognitive function as well as specific cognitive domains including memory, executive function, attention, and language. Mean patient age in the studies varied from 48 to 81 years. Nine of these studies were also included in the previously discussed Cochrane review.2

There was a statistically significant but small decrease in overall global cognition (10 trials; N = 12,115; standardized mean difference [SMD] = –0.04; 95% CI, –0.08 to –0.01) in those receiving HRT compared to placebo. This effect was slightly more pronounced among those who initiated HRT at age > 60 years (8 trials; N = 11,914; SMD = –0.05; 95% CI, –0.08 to –0.01) and among patients with HRT duration > 6 months (7 trials; N = 11,828; SMD = –0.05; 95% CI, –0.08 to –0.01). There were no significant differences in specific cognitive domains.2

In a 2017 follow-up to the WHI trial, researchers analyzed data on long-term cognitive effects in women previously treated with HRT. There were 2 cohorts: participants who initiated HRT at a younger age (50-54; N = 1376) and those who initiated HRT later in life (age 65-79; N = 2880). Cognitive outcomes were assessed using the Telephone Interview for Cognitive Status-modified, with interviews conducted annually beginning 6 to 7 years after HRT was stopped.3

The investigators found no significant change in composite cognitive function in the younger HRT-treated group compared to placebo (estrogen alone: mean deviation [MD] = 0.014; 95% CI, –0.097 to 0.126; estrogen + progesterone: MD = –0.047; 95% CI, –0.134 to 0.04), or in the group who initiated HRT at an older age (estrogen alone: MD = –0.099; 95% CI, –0.202 to 0.004; estrogen + progesterone: MD = –0.022; 95% CI, –0.099 to 0.055). The authors state that although the data did not reach significance, this study also found a trend toward decreases in global cognitive function in the older age group.3

Editor’s takeaway

Abundant, consistent evidence with long-term follow-up shows postmenopausal HRT does not reduce cognitive decline. In fact, it appears to increase cognitive decline slightly. Renewed interest in postmenopausal HRT to alleviate menopausal symptoms should balance the risks and benefits to the individual patient.

References

1. Marjoribanks J, Farquhar C, Roberts H, et al. Long-term hormone therapy for perimenopausal and postmenopausal women. Cochrane Database Syst Rev. 2017;1:CD004143. doi: 10.1002/14651858.CD004143.pub5

2. Zhou HH, Yu Z, Luo L, et al. The effect of hormone replacement therapy on cognitive function in healthy postmenopausal women: a meta-analysis of 23 randomized controlled trials. Psychogeriatrics. 2021;21:926-938. doi: 10.1111/psyg.12768

3. Espeland MA, Rapp SR, Manson JE, et al. Long-term effects on cognitive trajectories of postmenopausal hormone therapy in two age groups. J Gerontol A Biol Sci Med Sci. 2017;72:838-845. doi: 10.1093/gerona/glw156

References

1. Marjoribanks J, Farquhar C, Roberts H, et al. Long-term hormone therapy for perimenopausal and postmenopausal women. Cochrane Database Syst Rev. 2017;1:CD004143. doi: 10.1002/14651858.CD004143.pub5

2. Zhou HH, Yu Z, Luo L, et al. The effect of hormone replacement therapy on cognitive function in healthy postmenopausal women: a meta-analysis of 23 randomized controlled trials. Psychogeriatrics. 2021;21:926-938. doi: 10.1111/psyg.12768

3. Espeland MA, Rapp SR, Manson JE, et al. Long-term effects on cognitive trajectories of postmenopausal hormone therapy in two age groups. J Gerontol A Biol Sci Med Sci. 2017;72:838-845. doi: 10.1093/gerona/glw156

Issue
The Journal of Family Practice - 72(4)
Issue
The Journal of Family Practice - 72(4)
Page Number
E10-E11
Page Number
E10-E11
Publications
Publications
Topics
Article Type
Display Headline
Does hormone replacement therapy prevent cognitive decline in postmenopausal women?
Display Headline
Does hormone replacement therapy prevent cognitive decline in postmenopausal women?
Sections
Teambase XML
<?xml version="1.0" encoding="UTF-8"?>
<!--$RCSfile: InCopy_agile.xsl,v $ $Revision: 1.35 $-->
<!--$RCSfile: drupal.xsl,v $ $Revision: 1.7 $-->
<root generator="drupal.xsl" gversion="1.7"> <header> <fileName>CIs0523</fileName> <TBEID>0C02CD07.SIG</TBEID> <TBUniqueIdentifier>NJ_0C02CD07</TBUniqueIdentifier> <newsOrJournal>Journal</newsOrJournal> <publisherName>Frontline Medical Communications Inc.</publisherName> <storyname>Does hormone replacement&#13;therap</storyname> <articleType>1</articleType> <TBLocation>Copyfitting-JFP</TBLocation> <QCDate/> <firstPublished>20230515T151355</firstPublished> <LastPublished>20230515T151355</LastPublished> <pubStatus qcode="stat:"/> <embargoDate/> <killDate/> <CMSDate>20230515T151355</CMSDate> <articleSource/> <facebookInfo/> <meetingNumber/> <byline>Madeline Gates, MD;&#13;Melissa Beagle, MD,&#13;MPH; Lauren Bull, MD</byline> <bylineText/> <bylineFull>Madeline Gates, MD;&#13;Melissa Beagle, MD,&#13;MPH; Lauren Bull, MD</bylineFull> <bylineTitleText>Madeline Gates, MD; Melissa Beagle, MD, MPH; Lauren Bull, MD; Roxanne Radi, MD, MPH; Corey Lyon, DOUniversity of Colorado Family Medicine Residency, DenverKristen DeSanto, MSLS, MS, RDUniversity of Colorado Health Sciences Library, DenverDEPUTY EDITORRick Guthmann, MD, MPHAdvocate Health Care Illinois Masonic Medical Center Program, Chicago</bylineTitleText> <USOrGlobal/> <wireDocType/> <newsDocType/> <journalDocType/> <linkLabel/> <pageRange>E10-E11</pageRange> <citation/> <quizID/> <indexIssueDate/> <itemClass qcode="ninat:text"/> <provider qcode="provider:"> <name/> <rightsInfo> <copyrightHolder> <name/> </copyrightHolder> <copyrightNotice/> </rightsInfo> </provider> <abstract/> <metaDescription>NO. Hormone replacement therapy (HRT) does not prevent cognitive decline in postmenopausal women—and in fact, it may slightly increase risk (strength of recomme</metaDescription> <articlePDF>294971</articlePDF> <teaserImage/> <title>Q Does hormone replacement therapy prevent cognitive decline in postmenopausal women?</title> <deck/> <disclaimer/> <AuthorList/> <articleURL/> <doi/> <pubMedID/> <publishXMLStatus/> <publishXMLVersion>1</publishXMLVersion> <useEISSN>0</useEISSN> <urgency/> <pubPubdateYear>2023</pubPubdateYear> <pubPubdateMonth>May</pubPubdateMonth> <pubPubdateDay/> <pubVolume>72</pubVolume> <pubNumber>4</pubNumber> <wireChannels/> <primaryCMSID/> <CMSIDs> <CMSID>3171</CMSID> </CMSIDs> <keywords> <keyword>women's health</keyword> <keyword> hormone replacement therapy</keyword> <keyword> HRT</keyword> <keyword> postmenopausal</keyword> <keyword> cognitive decline</keyword> </keywords> <seeAlsos/> <publications_g> <publicationData> <publicationCode>jfp</publicationCode> <pubIssueName>May 2023</pubIssueName> <pubArticleType>Clinical Inquiries | 3171</pubArticleType> <pubTopics/> <pubCategories/> <pubSections/> </publicationData> <publicationData> <publicationCode>mdfam</publicationCode> <pubIssueName/> <pubArticleType/> <pubTopics/> <pubCategories/> <pubSections/> <journalTitle/> <journalFullTitle/> <copyrightStatement/> </publicationData> </publications_g> <publications> <term canonical="true">30</term> <term>51948</term> </publications> <sections> <term canonical="true">27414</term> </sections> <topics> <term canonical="true">322</term> </topics> <links> <link> <itemClass qcode="ninat:composite"/> <altRep contenttype="application/pdf">images/1800245f.pdf</altRep> <description role="drol:caption"/> <description role="drol:credit"/> </link> </links> </header> <itemSet> <newsItem> <itemMeta> <itemRole>Main</itemRole> <itemClass>text</itemClass> <title>Q Does hormone replacement therapy prevent cognitive decline in postmenopausal women?</title> <deck/> </itemMeta> <itemContent> <p> <b>NO.</b> Hormone replacement therapy (HRT) does not prevent cognitive decline in postmenopausal women—and in fact, it may slightly increase risk (strength of recommendation, <b>A</b>; systematic review, meta-analysis of randomized controlled trials [RCTs], and individual RCT).</p> <h3>Evidence summary</h3> <p class="sub1">Multiple analyses suggest HRTworsens rather than improves cognition</p> <p>A 2017 Cochrane review of 22 randomized, double-blind studies compared use of HRT (estrogen only or combination estrogen + progesterone therapies) with placebo in postmenopausal women (N = 43,637). Age ranges varied, but the average age in most studies was &gt; 60 years. Treatment duration was at least 1 year. Various outcomes were assessed across these 22 studies, including cardiovascular disease, bone health, and cognition.<sup>1</sup> </p> <p>Cognitive outcomes were assessed with the Mini-Mental Status Exam in 5 of the trials (N = 12,789). Data were not combined due to heterogeneity. The authors found no significant difference in cognitive scores between the treatment and control groups in any of these 5 studies.<sup>1</sup> <br/><br/>In the largest included study, the Women’s Health Initiative (WHI) Memory Study (N = 10,739), participants were older than 65 years. Among those receiving estrogen-only HRT, there were no statistically significant differences compared to those receiving placebo. However, healthy postmenopausal women taking combination HRT had an increased risk for “probable dementia” compared to those taking placebo (relative risk [RR] = 1.97; 95% CI, 1.16-3.33). When researchers looked exclusively at women taking HRT, the risk for dementia increased from 9 in 1000 to 18 in 1000 (95% CI, 11-30) after 4 years of HRT use. This results in a number needed to harm of 4 to 50 patients.<sup>1</sup> <br/><br/>Two notable limitations of this evidence are that the average age of this population was &gt; 60 years and 80% of the participants were White.<sup>1<br/><br/></sup>A 2021 meta-analysis of 23 RCTs (N = 13,683) studied the effect of HRT on global cognitive function as well as specific cognitive domains including memory, executive function, attention, and language. Mean patient age in the studies varied from 48 to 81 years. Nine of these studies were also included in the previously discussed Cochrane review.<sup>2</sup> <br/><br/>There was a statistically significant but small decrease in overall global cognition (10 trials; N = 12,115; standardized mean difference [SMD] = –0.04; 95% CI, –0.08 to –0.01) in those receiving HRT compared to placebo. This effect was slightly more pronounced among those who initiated HRT at age &gt; 60 years (8 trials; N = 11,914; SMD = –0.05; 95% CI, –0.08 to –0.01) and among patients with HRT duration &gt; 6 months (7 trials; N = 11,828; SMD = –0.05; 95% CI, –0.08 to –0.01). There were no significant differences in specific cognitive domains.<sup>2<br/><br/></sup>In a 2017 follow-up to the WHI trial, researchers analyzed data on long-term cognitive effects in women previously treated with HRT. There were 2 cohorts: participants who initiated HRT at a younger age (50-54; N = 1376) and those who initiated HRT later in life (age 65-79; N = 2880). Cognitive outcomes were assessed using the Telephone Interview for Cognitive Status-modified, with interviews conducted annually beginning 6 to 7 years after HRT was stopped.<sup>3</sup> <br/><br/>The investigators found no significant change in composite cognitive function in the younger HRT-treated group compared to placebo (estrogen alone: mean deviation [MD] = 0.014; 95% CI, –0.097 to 0.126; estrogen + progesterone: MD = –0.047; 95% CI, –0.134 to 0.04), or in the group who initiated HRT at an older age (estrogen alone: MD = –0.099; 95% CI, –0.202 to 0.004; estrogen + progesterone: MD = –0.022; 95% CI, –0.099 to 0.055). The authors state that although the data did not reach significance, this study also found a trend toward decreases in global cognitive function in the older age group.<sup>3</sup></p> <h3>Editor’s takeaway</h3> <p>Abundant, consistent evidence with long-term follow-up shows postmenopausal HRT does not reduce cognitive decline. In fact, it appears to increase cognitive decline slightly. Renewed interest in postmenopausal HRT to alleviate menopausal symptoms should balance the risks and benefits to the individual patient. <span class="end">JFP</span></p> <p class="reference"> 1. Marjoribanks J, Farquhar C, Roberts H, et al. Long-term hormone therapy for perimenopausal and postmenopausal women. <i>Cochrane Database Syst Rev</i>. 2017;1:CD004143. doi: 10.1002/14651858.CD004143.pub5 <br/><br/> 2. Zhou HH, Yu Z, Luo L, et al. The effect of hormone replacement therapy on cognitive function in healthy postmenopausal women: a meta-analysis of 23 randomized controlled trials. <i>Psychogeriatrics</i>. 2021;21:926-938. doi: 10.1111/psyg.12768 <br/><br/> 3. Espeland MA, Rapp SR, Manson JE, et al. Long-term effects on cognitive trajectories of postmenopausal hormone therapy in two age groups. <i>J Gerontol A Biol Sci Med Sci</i>. 2017;72:838-845. doi: 10.1093/gerona/glw156 </p> </itemContent> </newsItem> <newsItem> <itemMeta> <itemRole>bio</itemRole> <itemClass>text</itemClass> <title/> <deck/> </itemMeta> <itemContent> <p class="DOI">doi: 10.12788/jfp.0594<hl name="299"/></p> </itemContent> </newsItem> </itemSet></root>
PURLs Copyright
Evidence-based answers from the Family Physicians Inquiries Network
Inside the Article

EVIDENCE-BASED REVIEW:

NO. Hormone replacement therapy (HRT) does not prevent cognitive decline in postmenopausal women—and in fact, it may slightly increase risk (strength of recommendation, A; systematic review, meta-analysis of randomized controlled trials [RCTs], and individual RCT).

Disallow All Ads
Content Gating
No Gating (article Unlocked/Free)
Alternative CME
Disqus Comments
Default
Use ProPublica
Hide sidebar & use full width
render the right sidebar.
Conference Recap Checkbox
Not Conference Recap
Clinical Edge
Display the Slideshow in this Article
Medscape Article
Display survey writer
Reuters content
Disable Inline Native ads
WebMD Article
Article PDF Media
Image
Teambase ID
1800245F.SIG
Disable zoom
Off

Do behavioral interventions improve nighttime sleep in children < 1 year old?

Article Type
Changed
Tue, 07/12/2022 - 12:24
Display Headline
Do behavioral interventions improve nighttime sleep in children < 1 year old?

Most interventions resulted in at least modest improvements in sleep

A randomized controlled trial (RCT) of 279 newborn infants and their mothers evaluated developmentally appropriate sleep interventions.1 Mothers were given guidance on bedtime sleep routines, including starting the routine 30 to 45 minutes before bedtime, choosing age-appropriate calming bedtime activities, not using feeding as the last step before bedtime, and offering the child choices with their routine. Mothers were also given guidance on sleep location and behaviors, including recommendations on the best bedtime (between 7 and 8 pm), avoidance of a stimulating environment, and transition of the infant to their own room by age 3 months. To address nighttime awakenings, the researchers advised not waking the infant routinely to feed, allowing the infant some time to self-soothe after waking at night, and keeping nighttime interactions with the child boring.

These interventions were compared to a control group that received instructions on crib safety, sudden infant death syndrome prevention, and other sleep safety recommendations. Infant nocturnal sleep duration was determined by maternal report using the Brief Infant Sleep Questionnaire (BISQ). After 40 weeks, infants in the intervention group demonstrated longer sleep duration than did those in the control group (624.6 ± 67.6 minutes vs 602.9 ± 76.1 minutes; P = .01).1

An RCT of 82 infants (ages 2-4 months) and their mothers evaluated the effect of behavioral sleep interventions on maternal and infant sleep.2 Parents were offered either a 90-minute class and take-home booklet about behavioral sleep interventions or a 30-minute training on general infant safety with an accompanying pamphlet.

The behavioral interventions booklet included instructions on differentiating day and night routines for baby, avoiding digital devices and television in the evenings, playing more active games in the morning, dimming lights and reducing house noises in the afternoon, and having a consistent nighttime routine with consistent bedtime and sleep space. Participants completed an infant sleep diary prior to the intervention and repeated the sleep diary 8 weeks after the intervention. The infants whose mothers received the education on behavioral sleep interventions demonstrated an increase in nighttime sleep duration when compared to the control group (7.4 to 8.8 hours vs 7.3 to 7.5 hours; ANCOVA P < .001).

An RCT of 235 families with infants ages 6 to 8 months evaluated the effect of 45 minutes of nurse-provided education regarding normal infant sleep, effects of inadequate sleep, setting limits around infant sleep, importance of daytime routines, and negative sleep associations combined with a booklet and weekly phone follow-ups.3 This intervention was compared to routine infant education. At age 6 weeks, infants were monitored for 48 hours with actigraphy and the mothers completed a sleep diary to correlate activities. There was no difference in average nightly waking (2 nightly wakes; risk difference = –0.2%; 95% CI, –1.32 to 0.91).

The low cost and risk of these behavioral interventions to improve infants’ sleep make them worthwhile.

An RCT of 268 families with infants (ages 2-3 weeks) evaluated the effect of 45 minutes of nurse-provided education on behavioral sleep interventions including the cyclical nature of infant sleep, environmental factors that influence sleep, and parent-independent sleep cues (eg, leaving a settling infant alone for 5 minutes before responding) combined with written information.4 This was compared to infants receiving standard care without parental sleep intervention education. Participants recorded sleep diaries for 7 days when their infant reached age 6 weeks and again at age 12 weeks. At both 6 weeks and 12 weeks, there was a significant increase in infant nocturnal sleep time in the intervention group vs the control group (mean difference [MD] at 6 weeks = 0.5 hours; 95% CI, 0.32 to 0.69 vs MD at 12 weeks = 0.64 hours; 95% CI, 0.19 to 0.89).

A nonrandomized controlled trial with 84 mothers and infants (ages 0-6 months) evaluated the effectiveness of a multifaceted intervention involving brief focused negotiation by pediatricians, motivational counseling by a health educator, and group parenting workshops, compared to mother–infant pairs receiving standard care.5 Parents completed the BISQ at 0 and 6 months to assess nocturnal sleep duration. At 6 months, the intervention group had a significantly higher increase in infant nocturnal sleep duration compared to the control group (mean increase = 1.9 vs 1.3 hours; P = .05).

In a prospective cohort study involving 79 infants (ages 3-24 months) with parent- or pediatrician-reported day and night sleep problems, parents were given education on the promotion of nighttime sleep by gradually reducing contact with the infant over several nights and only leaving the room after the infant fell asleep or allowing the child to self-soothe for 1-3 minutes.6 The intervention was performed over 3 weeks, with in-person follow-up performed on Day 15 and phone follow-up on Days 8 and 21. Infants in this study demonstrated an increase in the average hours of total night sleep from 10.2 to 10.5 hours (P < .001).

Editor’s takeaway

Providing behavioral recommendations to parents about infant sleep routines improves sleep duration. This increased sleep duration, and the supporting evidence, is modest, but the low cost and risk of these interventions make them worthwhile.

References

1. Paul IM, Savage JS, Anzman-Frasca S, et al. INSIGHT responsive parenting intervention and infant sleep. Pediatrics. 2016;138:e20160762. doi:10.1542/peds.2016-0762

2. Rouzafzoon M, Farnam F, Khakbazan Z. The effects of infant behavioural sleep interventions on maternal sleep and mood, and infant sleep: a randomised controlled trial. J Sleep Res. 2021;30:e13344. doi: 10.1111/jsr.13344

3. Hall WA, Hutton E, Brant RF, et al. A randomized controlled trial of an intervention for infants’ behavioral sleep problems. BMC Pediatr. 2015;15:181. doi:10.1186/s12887-015-0492-7

4. Symon BG, Marley JE, Martin AJ, et al. Effect of a consultation teaching behaviour modification on sleep performance in infants: a randomised controlled trial. Med J Aust. 2005;182:215-218. doi: 10.5694/j.1326-5377.2005.tb06669.x

5. Taveras EM, Blackburn K, Gillman MW, et al. First steps for mommy and me: a pilot intervention to improve nutrition and physical activity behaviors of postpartum mothers and their infants. Matern Child Health J. 2011;15:1217-1227. doi: 10.1007/s10995-010-0696-2

6. Skuladottir A, Thome M, Ramel A. Improving day and night sleep problems in infants by changing day time sleep rhythm: a single group before and after study. Int J Nurs Stud. 2005;42:843-850. doi: 10.1016/j.ijnurstu.2004.12.004

Article PDF
Author and Disclosure Information

Morgan Hungenberg, DO
BreAnna Houss, MD
Madhav Narayan, MD
Corey Lyon, DO

University of Colorado Family Medicine Residency, Denver

Kristen DeSanto, MSLS, MS, RD
University of Colorado Health Sciences Library, Denver

DEPUTY EDITOR
Rick Guthmann, MD, MPH

Advocate Health Care Illinois Masonic Medical Center Program, Chicago

Issue
The Journal of Family Practice - 71(6)
Publications
Topics
Page Number
E16-E17
Sections
Author and Disclosure Information

Morgan Hungenberg, DO
BreAnna Houss, MD
Madhav Narayan, MD
Corey Lyon, DO

University of Colorado Family Medicine Residency, Denver

Kristen DeSanto, MSLS, MS, RD
University of Colorado Health Sciences Library, Denver

DEPUTY EDITOR
Rick Guthmann, MD, MPH

Advocate Health Care Illinois Masonic Medical Center Program, Chicago

Author and Disclosure Information

Morgan Hungenberg, DO
BreAnna Houss, MD
Madhav Narayan, MD
Corey Lyon, DO

University of Colorado Family Medicine Residency, Denver

Kristen DeSanto, MSLS, MS, RD
University of Colorado Health Sciences Library, Denver

DEPUTY EDITOR
Rick Guthmann, MD, MPH

Advocate Health Care Illinois Masonic Medical Center Program, Chicago

Article PDF
Article PDF

Most interventions resulted in at least modest improvements in sleep

A randomized controlled trial (RCT) of 279 newborn infants and their mothers evaluated developmentally appropriate sleep interventions.1 Mothers were given guidance on bedtime sleep routines, including starting the routine 30 to 45 minutes before bedtime, choosing age-appropriate calming bedtime activities, not using feeding as the last step before bedtime, and offering the child choices with their routine. Mothers were also given guidance on sleep location and behaviors, including recommendations on the best bedtime (between 7 and 8 pm), avoidance of a stimulating environment, and transition of the infant to their own room by age 3 months. To address nighttime awakenings, the researchers advised not waking the infant routinely to feed, allowing the infant some time to self-soothe after waking at night, and keeping nighttime interactions with the child boring.

These interventions were compared to a control group that received instructions on crib safety, sudden infant death syndrome prevention, and other sleep safety recommendations. Infant nocturnal sleep duration was determined by maternal report using the Brief Infant Sleep Questionnaire (BISQ). After 40 weeks, infants in the intervention group demonstrated longer sleep duration than did those in the control group (624.6 ± 67.6 minutes vs 602.9 ± 76.1 minutes; P = .01).1

An RCT of 82 infants (ages 2-4 months) and their mothers evaluated the effect of behavioral sleep interventions on maternal and infant sleep.2 Parents were offered either a 90-minute class and take-home booklet about behavioral sleep interventions or a 30-minute training on general infant safety with an accompanying pamphlet.

The behavioral interventions booklet included instructions on differentiating day and night routines for baby, avoiding digital devices and television in the evenings, playing more active games in the morning, dimming lights and reducing house noises in the afternoon, and having a consistent nighttime routine with consistent bedtime and sleep space. Participants completed an infant sleep diary prior to the intervention and repeated the sleep diary 8 weeks after the intervention. The infants whose mothers received the education on behavioral sleep interventions demonstrated an increase in nighttime sleep duration when compared to the control group (7.4 to 8.8 hours vs 7.3 to 7.5 hours; ANCOVA P < .001).

An RCT of 235 families with infants ages 6 to 8 months evaluated the effect of 45 minutes of nurse-provided education regarding normal infant sleep, effects of inadequate sleep, setting limits around infant sleep, importance of daytime routines, and negative sleep associations combined with a booklet and weekly phone follow-ups.3 This intervention was compared to routine infant education. At age 6 weeks, infants were monitored for 48 hours with actigraphy and the mothers completed a sleep diary to correlate activities. There was no difference in average nightly waking (2 nightly wakes; risk difference = –0.2%; 95% CI, –1.32 to 0.91).

The low cost and risk of these behavioral interventions to improve infants’ sleep make them worthwhile.

An RCT of 268 families with infants (ages 2-3 weeks) evaluated the effect of 45 minutes of nurse-provided education on behavioral sleep interventions including the cyclical nature of infant sleep, environmental factors that influence sleep, and parent-independent sleep cues (eg, leaving a settling infant alone for 5 minutes before responding) combined with written information.4 This was compared to infants receiving standard care without parental sleep intervention education. Participants recorded sleep diaries for 7 days when their infant reached age 6 weeks and again at age 12 weeks. At both 6 weeks and 12 weeks, there was a significant increase in infant nocturnal sleep time in the intervention group vs the control group (mean difference [MD] at 6 weeks = 0.5 hours; 95% CI, 0.32 to 0.69 vs MD at 12 weeks = 0.64 hours; 95% CI, 0.19 to 0.89).

A nonrandomized controlled trial with 84 mothers and infants (ages 0-6 months) evaluated the effectiveness of a multifaceted intervention involving brief focused negotiation by pediatricians, motivational counseling by a health educator, and group parenting workshops, compared to mother–infant pairs receiving standard care.5 Parents completed the BISQ at 0 and 6 months to assess nocturnal sleep duration. At 6 months, the intervention group had a significantly higher increase in infant nocturnal sleep duration compared to the control group (mean increase = 1.9 vs 1.3 hours; P = .05).

In a prospective cohort study involving 79 infants (ages 3-24 months) with parent- or pediatrician-reported day and night sleep problems, parents were given education on the promotion of nighttime sleep by gradually reducing contact with the infant over several nights and only leaving the room after the infant fell asleep or allowing the child to self-soothe for 1-3 minutes.6 The intervention was performed over 3 weeks, with in-person follow-up performed on Day 15 and phone follow-up on Days 8 and 21. Infants in this study demonstrated an increase in the average hours of total night sleep from 10.2 to 10.5 hours (P < .001).

Editor’s takeaway

Providing behavioral recommendations to parents about infant sleep routines improves sleep duration. This increased sleep duration, and the supporting evidence, is modest, but the low cost and risk of these interventions make them worthwhile.

Most interventions resulted in at least modest improvements in sleep

A randomized controlled trial (RCT) of 279 newborn infants and their mothers evaluated developmentally appropriate sleep interventions.1 Mothers were given guidance on bedtime sleep routines, including starting the routine 30 to 45 minutes before bedtime, choosing age-appropriate calming bedtime activities, not using feeding as the last step before bedtime, and offering the child choices with their routine. Mothers were also given guidance on sleep location and behaviors, including recommendations on the best bedtime (between 7 and 8 pm), avoidance of a stimulating environment, and transition of the infant to their own room by age 3 months. To address nighttime awakenings, the researchers advised not waking the infant routinely to feed, allowing the infant some time to self-soothe after waking at night, and keeping nighttime interactions with the child boring.

These interventions were compared to a control group that received instructions on crib safety, sudden infant death syndrome prevention, and other sleep safety recommendations. Infant nocturnal sleep duration was determined by maternal report using the Brief Infant Sleep Questionnaire (BISQ). After 40 weeks, infants in the intervention group demonstrated longer sleep duration than did those in the control group (624.6 ± 67.6 minutes vs 602.9 ± 76.1 minutes; P = .01).1

An RCT of 82 infants (ages 2-4 months) and their mothers evaluated the effect of behavioral sleep interventions on maternal and infant sleep.2 Parents were offered either a 90-minute class and take-home booklet about behavioral sleep interventions or a 30-minute training on general infant safety with an accompanying pamphlet.

The behavioral interventions booklet included instructions on differentiating day and night routines for baby, avoiding digital devices and television in the evenings, playing more active games in the morning, dimming lights and reducing house noises in the afternoon, and having a consistent nighttime routine with consistent bedtime and sleep space. Participants completed an infant sleep diary prior to the intervention and repeated the sleep diary 8 weeks after the intervention. The infants whose mothers received the education on behavioral sleep interventions demonstrated an increase in nighttime sleep duration when compared to the control group (7.4 to 8.8 hours vs 7.3 to 7.5 hours; ANCOVA P < .001).

An RCT of 235 families with infants ages 6 to 8 months evaluated the effect of 45 minutes of nurse-provided education regarding normal infant sleep, effects of inadequate sleep, setting limits around infant sleep, importance of daytime routines, and negative sleep associations combined with a booklet and weekly phone follow-ups.3 This intervention was compared to routine infant education. At age 6 weeks, infants were monitored for 48 hours with actigraphy and the mothers completed a sleep diary to correlate activities. There was no difference in average nightly waking (2 nightly wakes; risk difference = –0.2%; 95% CI, –1.32 to 0.91).

The low cost and risk of these behavioral interventions to improve infants’ sleep make them worthwhile.

An RCT of 268 families with infants (ages 2-3 weeks) evaluated the effect of 45 minutes of nurse-provided education on behavioral sleep interventions including the cyclical nature of infant sleep, environmental factors that influence sleep, and parent-independent sleep cues (eg, leaving a settling infant alone for 5 minutes before responding) combined with written information.4 This was compared to infants receiving standard care without parental sleep intervention education. Participants recorded sleep diaries for 7 days when their infant reached age 6 weeks and again at age 12 weeks. At both 6 weeks and 12 weeks, there was a significant increase in infant nocturnal sleep time in the intervention group vs the control group (mean difference [MD] at 6 weeks = 0.5 hours; 95% CI, 0.32 to 0.69 vs MD at 12 weeks = 0.64 hours; 95% CI, 0.19 to 0.89).

A nonrandomized controlled trial with 84 mothers and infants (ages 0-6 months) evaluated the effectiveness of a multifaceted intervention involving brief focused negotiation by pediatricians, motivational counseling by a health educator, and group parenting workshops, compared to mother–infant pairs receiving standard care.5 Parents completed the BISQ at 0 and 6 months to assess nocturnal sleep duration. At 6 months, the intervention group had a significantly higher increase in infant nocturnal sleep duration compared to the control group (mean increase = 1.9 vs 1.3 hours; P = .05).

In a prospective cohort study involving 79 infants (ages 3-24 months) with parent- or pediatrician-reported day and night sleep problems, parents were given education on the promotion of nighttime sleep by gradually reducing contact with the infant over several nights and only leaving the room after the infant fell asleep or allowing the child to self-soothe for 1-3 minutes.6 The intervention was performed over 3 weeks, with in-person follow-up performed on Day 15 and phone follow-up on Days 8 and 21. Infants in this study demonstrated an increase in the average hours of total night sleep from 10.2 to 10.5 hours (P < .001).

Editor’s takeaway

Providing behavioral recommendations to parents about infant sleep routines improves sleep duration. This increased sleep duration, and the supporting evidence, is modest, but the low cost and risk of these interventions make them worthwhile.

References

1. Paul IM, Savage JS, Anzman-Frasca S, et al. INSIGHT responsive parenting intervention and infant sleep. Pediatrics. 2016;138:e20160762. doi:10.1542/peds.2016-0762

2. Rouzafzoon M, Farnam F, Khakbazan Z. The effects of infant behavioural sleep interventions on maternal sleep and mood, and infant sleep: a randomised controlled trial. J Sleep Res. 2021;30:e13344. doi: 10.1111/jsr.13344

3. Hall WA, Hutton E, Brant RF, et al. A randomized controlled trial of an intervention for infants’ behavioral sleep problems. BMC Pediatr. 2015;15:181. doi:10.1186/s12887-015-0492-7

4. Symon BG, Marley JE, Martin AJ, et al. Effect of a consultation teaching behaviour modification on sleep performance in infants: a randomised controlled trial. Med J Aust. 2005;182:215-218. doi: 10.5694/j.1326-5377.2005.tb06669.x

5. Taveras EM, Blackburn K, Gillman MW, et al. First steps for mommy and me: a pilot intervention to improve nutrition and physical activity behaviors of postpartum mothers and their infants. Matern Child Health J. 2011;15:1217-1227. doi: 10.1007/s10995-010-0696-2

6. Skuladottir A, Thome M, Ramel A. Improving day and night sleep problems in infants by changing day time sleep rhythm: a single group before and after study. Int J Nurs Stud. 2005;42:843-850. doi: 10.1016/j.ijnurstu.2004.12.004

References

1. Paul IM, Savage JS, Anzman-Frasca S, et al. INSIGHT responsive parenting intervention and infant sleep. Pediatrics. 2016;138:e20160762. doi:10.1542/peds.2016-0762

2. Rouzafzoon M, Farnam F, Khakbazan Z. The effects of infant behavioural sleep interventions on maternal sleep and mood, and infant sleep: a randomised controlled trial. J Sleep Res. 2021;30:e13344. doi: 10.1111/jsr.13344

3. Hall WA, Hutton E, Brant RF, et al. A randomized controlled trial of an intervention for infants’ behavioral sleep problems. BMC Pediatr. 2015;15:181. doi:10.1186/s12887-015-0492-7

4. Symon BG, Marley JE, Martin AJ, et al. Effect of a consultation teaching behaviour modification on sleep performance in infants: a randomised controlled trial. Med J Aust. 2005;182:215-218. doi: 10.5694/j.1326-5377.2005.tb06669.x

5. Taveras EM, Blackburn K, Gillman MW, et al. First steps for mommy and me: a pilot intervention to improve nutrition and physical activity behaviors of postpartum mothers and their infants. Matern Child Health J. 2011;15:1217-1227. doi: 10.1007/s10995-010-0696-2

6. Skuladottir A, Thome M, Ramel A. Improving day and night sleep problems in infants by changing day time sleep rhythm: a single group before and after study. Int J Nurs Stud. 2005;42:843-850. doi: 10.1016/j.ijnurstu.2004.12.004

Issue
The Journal of Family Practice - 71(6)
Issue
The Journal of Family Practice - 71(6)
Page Number
E16-E17
Page Number
E16-E17
Publications
Publications
Topics
Article Type
Display Headline
Do behavioral interventions improve nighttime sleep in children < 1 year old?
Display Headline
Do behavioral interventions improve nighttime sleep in children < 1 year old?
Sections
Teambase XML
<?xml version="1.0" encoding="UTF-8"?>
<!--$RCSfile: InCopy_agile.xsl,v $ $Revision: 1.35 $-->
<!--$RCSfile: drupal.xsl,v $ $Revision: 1.7 $-->
<root generator="drupal.xsl" gversion="1.7"> <header> <fileName>CI0722_Sleep</fileName> <TBEID>0C02A03A.SIG</TBEID> <TBUniqueIdentifier>NJ_0C02A03A</TBUniqueIdentifier> <newsOrJournal>Journal</newsOrJournal> <publisherName>Frontline Medical Communications Inc.</publisherName> <storyname>Do behavioral interventions&#13;imp</storyname> <articleType>1</articleType> <TBLocation>Copyfitting-JFP</TBLocation> <QCDate/> <firstPublished>20220712T120807</firstPublished> <LastPublished>20220712T120807</LastPublished> <pubStatus qcode="stat:"/> <embargoDate/> <killDate/> <CMSDate>20220712T120806</CMSDate> <articleSource/> <facebookInfo/> <meetingNumber/> <byline>Morgan Hungenberg,&#13;DO; BreAnna Houss, MD;&#13;Madhav Narayan, MD</byline> <bylineText/> <bylineFull>Morgan Hungenberg,&#13;DO; BreAnna Houss, MD;&#13;Madhav Narayan, MD</bylineFull> <bylineTitleText/> <USOrGlobal/> <wireDocType/> <newsDocType/> <journalDocType/> <linkLabel/> <pageRange>E16-E17</pageRange> <citation/> <quizID/> <indexIssueDate/> <itemClass qcode="ninat:text"/> <provider qcode="provider:"> <name/> <rightsInfo> <copyrightHolder> <name/> </copyrightHolder> <copyrightNotice/> </rightsInfo> </provider> <abstract/> <metaDescription>YES. Infants respond to behavioral interventions, although objective data are limited. Behavioral interventions include establishing regular daytime and sleep r</metaDescription> <articlePDF>287802</articlePDF> <teaserImage/> <title>Q Do behavioral interventions improve nighttime sleep in children &lt; 1 year old?</title> <deck/> <disclaimer/> <AuthorList/> <articleURL/> <doi/> <pubMedID/> <publishXMLStatus/> <publishXMLVersion>1</publishXMLVersion> <useEISSN>0</useEISSN> <urgency/> <pubPubdateYear>2022</pubPubdateYear> <pubPubdateMonth>July</pubPubdateMonth> <pubPubdateDay/> <pubVolume>71</pubVolume> <pubNumber>6</pubNumber> <wireChannels/> <primaryCMSID/> <CMSIDs> <CMSID>3167</CMSID> </CMSIDs> <keywords> <keyword>pediatrics</keyword> <keyword> sleep medicine</keyword> <keyword> sleep</keyword> <keyword> children</keyword> </keywords> <seeAlsos/> <publications_g> <publicationData> <publicationCode>jfp</publicationCode> <pubIssueName>July 2022</pubIssueName> <pubArticleType>Applied Evidence | 3167</pubArticleType> <pubTopics/> <pubCategories/> <pubSections/> </publicationData> </publications_g> <publications> <term canonical="true">30</term> </publications> <sections> <term canonical="true">27414</term> </sections> <topics> <term>271</term> <term canonical="true">296</term> </topics> <links> <link> <itemClass qcode="ninat:composite"/> <altRep contenttype="application/pdf">images/18002186.PDF</altRep> <description role="drol:caption"/> <description role="drol:credit"/> </link> </links> </header> <itemSet> <newsItem> <itemMeta> <itemRole>Main</itemRole> <itemClass>text</itemClass> <title>Q Do behavioral interventions improve nighttime sleep in children &lt; 1 year old?</title> <deck/> </itemMeta> <itemContent> <p><b> YES.</b> Infants respond to behavioral interventions, although objective data are limited. Behavioral interventions include establishing regular daytime and sleep routines for the infant, reducing environmental noises or distractions, and allowing for self-soothing at bedtime (strength of recommendation: <b>B</b>, based on multiple randomized and nonrandomized studies).</p> <p class="sub1">Most interventions resulted in at least modest improvements in sleep</p> <p>A randomized controlled trial (RCT) of 279 newborn infants and their mothers evaluated developmentally appropriate sleep interventions.<sup>1</sup> Mothers were given guidance on bedtime sleep routines, including starting the routine 30 to 45 minutes before bedtime, choosing age-appropriate calming bedtime activities, not using feeding as the last step before bedtime, and offering the child choices with their routine. Mothers were also given guidance on sleep location and behaviors, including recommendations on the best bedtime (between 7 and 8 <scaps>pm)</scaps>, avoidance of a stimulating environment, and transition of the infant to their own room by age 3 months. To address nighttime awakenings, the researchers advised not waking the infant routinely to feed, allowing the infant some time to self-soothe after waking at night, and keeping nighttime interactions with the child boring. </p> <p>These interventions were compared to a control group that received instructions on crib safety, sudden infant death syndrome prevention, and other sleep safety recommendations. Infant nocturnal sleep duration was determined by maternal report using the Brief Infant Sleep Questionnaire (BISQ). After 40 weeks, infants in the intervention group demonstrated longer sleep duration than did those in the control group (624.6 ± 67.6 minutes vs 602.9 ± 76.1 minutes; <i>P</i> = .01).<sup>1</sup> <br/><br/>An RCT of 82 infants (ages 2-4 months) and their mothers evaluated the effect of behavioral sleep interventions on maternal and infant sleep.<sup>2</sup> Parents were offered either a 90-minute class and take-home booklet about behavioral sleep interventions or a 30-minute training on general infant safety with an accompanying pamphlet. <br/><br/>The behavioral interventions booklet included instructions on differentiating day and night routines for baby, avoiding digital devices and television in the evenings, playing more active games in the morning, dimming lights and reducing house noises in the afternoon, and having a consistent nighttime routine with consistent bedtime and sleep space. Participants completed an infant sleep diary prior to the intervention and repeated the sleep diary 8 weeks after the intervention. The infants whose mothers received the education on behavioral sleep interventions demonstrated an increase in nighttime sleep duration when compared to the control group (7.4 to 8.8 hours vs 7.3 to 7.5 hours; ANCOVA <i>P</i> &lt; .001). <br/><br/>An RCT of 235 families with infants ages 6 to 8 months evaluated the effect of 45 minutes of nurse-provided education regarding normal infant sleep, effects of inadequate sleep, setting limits around infant sleep, importance of daytime routines, and negative sleep associations combined with a booklet and weekly phone follow-ups.<sup>3</sup> This intervention was compared to routine infant education. At age 6 weeks, infants were monitored for 48 hours with actigraphy and the mothers completed a sleep diary to correlate activities. There was no difference in average nightly waking (2 nightly wakes; risk difference = –0.2%; 95% CI, –1.32 to 0.91). <br/><br/>An RCT of 268 families with infants (ages 2-3 weeks) evaluated the effect of 45 minutes of nurse-provided education on behavioral sleep interventions including the cyclical nature of infant sleep, environmental factors that influence sleep, and parent-independent sleep cues (eg, leaving a settling infant alone for 5 minutes before responding) combined with written information.<sup>4</sup> This was compared to infants receiving standard care without parental sleep intervention education. Participants recorded sleep diaries for 7 days when their infant reached age 6 weeks and again at age 12 weeks. At both 6 weeks and 12 weeks, there was a significant increase in infant nocturnal sleep time in the intervention group vs the control group (mean difference [MD] at 6 weeks = 0.5 hours; 95% CI, 0.32 to 0.69 vs MD at 12 weeks = 0.64 hours; 95% CI, 0.19 to 0.89). <br/><br/>A nonrandomized controlled trial with 84 mothers and infants (ages 0-6 months) evaluated the effectiveness of a multifaceted intervention involving brief focused negotiation by pediatricians, motivational counseling by a health educator, and group parenting workshops, compared to mother–infant pairs receiving standard care.<sup>5</sup> Parents completed the BISQ at 0 and 6 months to assess nocturnal sleep duration. At 6 months, the intervention group had a significantly higher increase in infant nocturnal sleep duration compared to the control group (mean increase = 1.9 vs 1.3 hours; <i>P</i> = .05). <br/><br/>In a prospective cohort study involving 79 infants (ages 3-24 months) with parent- or pediatrician-reported day and night sleep problems, parents were given education on the promotion of nighttime sleep by gradually reducing contact with the infant over several nights and only leaving the room after the infant fell asleep or allowing the child to self-soothe for 1-3 minutes.<sup>6</sup> The intervention was performed over 3 weeks, with in-person follow-up performed on Day 15 and phone follow-up on Days 8 and 21. Infants in this study demonstrated an increase in the average hours of total night sleep from 10.2 to 10.5 hours (<i>P</i> &lt; .001). </p> <p class="sub1">Editor’s takeaway</p> <p>Providing behavioral recommendations to parents about infant sleep routines improves sleep duration. This increased sleep duration, and the supporting evidence, is modest, but the low cost and risk of these interventions make them worthwhile. <span class="end">JFP</span></p> <p class="reference"> 1. Paul IM, Savage JS, Anzman-Frasca S, et al. INSIGHT responsive parenting intervention and infant sleep. <i>Pediatrics</i>. 2016;138:e20160762. doi:10.1542/peds.2016-0762 <br/><br/> 2. Rouzafzoon M, Farnam F, Khakbazan Z. The effects of infant behavioural sleep interventions on maternal sleep and mood, and infant sleep: a randomised controlled trial. <i>J Sleep Res</i>. 2021;30:e13344. doi: 10.1111/jsr.13344 <br/><br/> 3. Hall WA, Hutton E, Brant RF, et al. A randomized controlled trial of an intervention for infants’ behavioral sleep problems. <i>BMC Pediatr</i>. 2015;15:181. doi:10.1186/s12887-015-0492-7 <br/><br/> 4. Symon BG, Marley JE, Martin AJ, et al. Effect of a consultation teaching behaviour modification on sleep performance in infants: a randomised controlled trial. <i>Med J Aust</i>. 2005;182:215-218. doi: 10.5694/j.1326-5377.2005.tb06669.x </p> <p class="reference"> 5. Taveras EM, Blackburn K, Gillman MW, et al. First steps for mommy and me: a pilot intervention to improve nutrition and physical activity behaviors of postpartum mothers and their infants. <i>Matern Child Health J</i>. 2011;15:1217-1227. doi: 10.1007/s10995-010-0696-2 </p> <p class="reference"> 6. Skuladottir A, Thome M, Ramel A. Improving day and night sleep problems in infants by changing day time sleep rhythm: a single group before and after study. <i>Int J Nurs Stud</i>. 2005;42:843-850. doi: 10.1016/j.ijnurstu.2004.12.004 </p> </itemContent> </newsItem> </itemSet></root>
PURLs Copyright
Evidence-based answers from the Family Physicians Inquiries Network
Inside the Article

EVIDENCE-BASED ANSWER:

YES. Infants respond to behavioral interventions, although objective data are limited. Behavioral interventions include establishing regular daytime and sleep routines for the infant, reducing environmental noises or distractions, and allowing for self-soothing at bedtime (strength of recommendation: B, based on multiple randomized and nonrandomized studies).

Disallow All Ads
Content Gating
No Gating (article Unlocked/Free)
Alternative CME
Disqus Comments
Default
Use ProPublica
Hide sidebar & use full width
render the right sidebar.
Conference Recap Checkbox
Not Conference Recap
Clinical Edge
Display the Slideshow in this Article
Medscape Article
Display survey writer
Reuters content
Disable Inline Native ads
WebMD Article
Article PDF Media
Image
Teambase ID
18002186.SIG
Disable zoom
Off

Antibiotics and COPD: Time to order a C-reactive protein test?

Article Type
Changed
Wed, 06/23/2021 - 15:05

This RCT provided valuable insights as to whether CRP-guided prescribing could safely reduce antibiotic use during acute COPD exacerbations.

 

ILLUSTRATIVE CASE

A 55-year-old man with a history of chronic obstructive pulmonary disease (COPD) presents to you with increased sputum volume and increased dyspnea, but no fever. You diagnose a COPD exacerbation. Would point-of-care C-reactive protein (CRP) testing be a useful tool to guide antibiotic prescribing?

COPD is a common respiratory condition and one of the leading causes of death in the world.2 COPD requires chronic therapy and frequent treatment for acute exacerbations.3 A systematic review found that exacerbations occur an average of 1.3 times per year for patients with known COPD.4 Antibiotics are often prescribed for COPD exacerbations, but which patients benefit most from antibiotic treatment is unclear and identification often is based on clinical features alone. Additionally, overprescribing of antibiotics can lead to unnecessary adverse effects, drive antibiotic resistance, and be a waste of resources.5

The European Respiratory Society/American Thoracic Society (ERS/ATS) provides a conditional recommendation to consider antibiotics in ambulatory patients with COPD exacerbation based on moderate-quality evidence.6 The 2020 Global Initiative for Chronic Obstructive Lung Disease (GOLD) guidelines recommend antibiotics for moderately or severely ill patients with a COPD exacerbation who have increased cough and sputum purulence.7 While the ERS/ATS recommendations do not mention CRP, the GOLD guidelines discuss biomarkers as emerging tools in determining antibiotic utility.

Biomarkers such as procalcitonin and CRP are being examined as potential tools to distinguish which patients would benefit from antibiotic treatment in COPD exacerbations. In a 2013 study, CRP levels > 19.6 mg/L in the serum and > 15.2 mg/L in the sputum indicated a bacterial infection, but more research was needed to determine if CRP could help guide antibiotic prescribing.8 In a 2019 randomized trial of 101 patients with COPD exacerbations, researchers compared the GOLD strategy for antibiotic prescribing with a CRP-guided antibiotic strategy (CRP ≥ 50 mg/L) and found no difference in adverse events between study groups.9

This trial focused on point-of-care CRP-guided prescribing of antibiotics for patients with COPD exacerbations in the outpatient setting.

STUDY SUMMARY

Point-of-care CRP testing is noninferior to usual care

This open-label, multicenter, randomized controlled trial at 86 general medical practices in the United Kingdom examined whether the use of point-of-care CRP testing could reduce antibiotic use during acute exacerbations of COPD. Patients (N = 653; 650 needed to provide 81% to 90% power) were ages 40 years and older, had a diagnosis of COPD, and presented for an acute exacerbation of COPD based on the presence of at least 1 Anthonisen criteria (increased dyspnea, increase in sputum volume, and increase in purulent sputum).

Patients were randomized in a 1:1 fashion to receive care guided by point-of-care CRP testing (CRP-guided) or usual care for their COPD exacerbation. Patients in the CRP-guided group received a point-of-care CRP test as part of their assessment at presentation, or at any other appointments for COPD over the following 4 weeks.

The research team provided clinicians with CRP interpretation guidance based on the following CRP values: < 20 mg/L, antibiotics are typically not needed; 20 to 40 mg/L, antibiotics might be beneficial if purulent sputum is present; and > 40 mg/L, antibiotics are usually beneficial. Primary outcomes were patient-reported antibiotic use within 4 weeks and COPD-related health status. Of the patients who received a point-of-care CRP test, the median value was 6 mg/L; 76% had a value < 20 mg/L, 12% had values between 20 and 40 mg/L, and 12% had values > 40 mg/L. In the intention-to-treat analysis, fewer patients in the CRP-guided group reported antibiotic use vs those in the usual-care group (57% vs 77%; adjusted odds ratio [aOR] = 0.31; 95% CI, 0.20-0.47) within 4 weeks. The CRP-guided group also received fewer antibiotics at the initial visit compared to the usual-care group (48% vs 70%; aOR = 0.31; 95% CI, 0.21-0.45).

COPD-related health status was assessed with the Clinical COPD Questionnaire (score range, 0-6; a difference of 0.4 represents minimal clinical importance). At 2 weeks, the adjusted mean difference in the total health status score with the use of CRP was noninferior to usual care and was in favor of the CRP-guided group (mean difference = −0.19 points; two-sided 90% CI, −0.33 to −0.05). There was no evidence of clinically important between-group differences in pneumonia (3% vs 4%; aOR = 0.73; 95% CI, 0.29-1.82) at 6-month follow-up. Rates of hospitalization at 6 months were similar between groups (9.3% vs 8.6%; no P value provided).

Fewer patients in the CRPguided group reported antibiotic use vs those in the usual-care group within 4 weeks.


Limitations of this trial included patient report of antibiotic use and the lack of a sham test.

WHAT'S NEW

RCT provides evidence to support use of CRP testing

Point-of-care CRP testing can reduce antibiotic prescribing in patients presenting with a COPD exacerbation without affecting symptom improvement or adverse events.

CAVEATS

CRP testing may not be cost effective

CRP testing—especially point-of-care ­testing—remains expensive in many parts of the United States. A 2015 cost-effectiveness analysis of point-of-care CRP tests for respiratory tract infection in England concluded the cost of the test per patient was not cost effective.10 It is unknown if point-of-care CRP testing would be cost effective in guiding antibiotic prescribing for ­primary care providers with a focus on COPD exacerbations.

CHALLENGES TO IMPLEMENTATION 

Virtual visits and variable access may limit use

CRP-guided antibiotic prescribing may be challenging in some clinical scenarios or clinics with the rise of virtual visits and differential access in primary care clinics to point-of-care CRP tests. JFP

ACKNOWLEDGEMENT

The PURLs Surveillance System was supported in part by Grant Number UL1RR024999 from the National Center For Research Resources, a Clinical Translational Science Award to the University of Chicago. The content is solely the responsibility of the authors and does not necessarily represent the official views of the National Center For Research Resources or the National Institutes of Health. Copyright © 2021. The Family Physicians Inquiries Network. All rights reserved.

References
  1. Butler CC, Gillespie D, White P, et al. C-reactive protein testing to guide antibiotic prescribing for COPD exacerbations. N Engl J Med. 2019;381:111-120.
  2. Lopez AD, Mathers CD, Ezzati M, et al. Global Burden of Disease and Risk Factors. The World Bank; 2006.
  3. Buist AS, McBurnie MA, Vollmer WM, et al. International variation in the prevalence of COPD (the BOLD Study): a population-based prevalence study. Lancet. 2007;370:741-750.
  4. Singh J, Palda V, Stanbrook M, et al. Corticosteroid therapy for patients with acute exacerbations of chronic obstructive pulmonary disease: a systematic review. Arch Intern Med. 2002;162:2527-2536.
  5. Schroeck JL, Ruh CA, Sellick JA, et al. Factors associated with antibiotic misuse in outpatient treatment for upper respiratory tract infections. Antimicrob Agents Chemother. 2015;59:3848-3852.
  6. Wedzicha JA, Miravitlles M, Hurst JR, et al. Management of COPD exacerbations: a European Respiratory Society/American Thoracic Society guideline. Eur Respir J. 2017;49:1600791.
  7. Global Initiative for Chronic Obstructive Lung Disease. Global Strategy for the Diagnosis, and Management and Prevention of Chronic Obstructive Pulmonary Disease (2020 report). Accessed May 12, 2021. https://goldcopd.org/gold-reports/
  8. Peng C, Tian C, Zhang Y, et al. C-reactive protein levels predict bacterial exacerbation in patients with chronic obstructive pulmonary disease. Am J Med Sci. 2013;345:190-194.
  9. Prins H, Duijkers R, van der Valk P, et al. CRP-guided antibiotic treatment in acute exacerbations of COPD in hospital admissions. Eur Respir J. 2019;53:1802014.
  10. Hunter R. Cost-effectiveness of point-of-care C-reactive protein tests for respiratory tract infection in primary care in England. Adv Ther. 2015;32:69-85.
Article PDF
Author and Disclosure Information

University of Colorado Family Medicine Residency, Aurora

DEPUTY EDITOR
Katherine Hale, PharmD, BCPS, MFA

Department of Nursing, Heritage University, Toppenish, WA

Issue
The Journal of Family Practice - 70(5)
Publications
Topics
Page Number
244-246
Sections
Author and Disclosure Information

University of Colorado Family Medicine Residency, Aurora

DEPUTY EDITOR
Katherine Hale, PharmD, BCPS, MFA

Department of Nursing, Heritage University, Toppenish, WA

Author and Disclosure Information

University of Colorado Family Medicine Residency, Aurora

DEPUTY EDITOR
Katherine Hale, PharmD, BCPS, MFA

Department of Nursing, Heritage University, Toppenish, WA

Article PDF
Article PDF

This RCT provided valuable insights as to whether CRP-guided prescribing could safely reduce antibiotic use during acute COPD exacerbations.

This RCT provided valuable insights as to whether CRP-guided prescribing could safely reduce antibiotic use during acute COPD exacerbations.

 

ILLUSTRATIVE CASE

A 55-year-old man with a history of chronic obstructive pulmonary disease (COPD) presents to you with increased sputum volume and increased dyspnea, but no fever. You diagnose a COPD exacerbation. Would point-of-care C-reactive protein (CRP) testing be a useful tool to guide antibiotic prescribing?

COPD is a common respiratory condition and one of the leading causes of death in the world.2 COPD requires chronic therapy and frequent treatment for acute exacerbations.3 A systematic review found that exacerbations occur an average of 1.3 times per year for patients with known COPD.4 Antibiotics are often prescribed for COPD exacerbations, but which patients benefit most from antibiotic treatment is unclear and identification often is based on clinical features alone. Additionally, overprescribing of antibiotics can lead to unnecessary adverse effects, drive antibiotic resistance, and be a waste of resources.5

The European Respiratory Society/American Thoracic Society (ERS/ATS) provides a conditional recommendation to consider antibiotics in ambulatory patients with COPD exacerbation based on moderate-quality evidence.6 The 2020 Global Initiative for Chronic Obstructive Lung Disease (GOLD) guidelines recommend antibiotics for moderately or severely ill patients with a COPD exacerbation who have increased cough and sputum purulence.7 While the ERS/ATS recommendations do not mention CRP, the GOLD guidelines discuss biomarkers as emerging tools in determining antibiotic utility.

Biomarkers such as procalcitonin and CRP are being examined as potential tools to distinguish which patients would benefit from antibiotic treatment in COPD exacerbations. In a 2013 study, CRP levels > 19.6 mg/L in the serum and > 15.2 mg/L in the sputum indicated a bacterial infection, but more research was needed to determine if CRP could help guide antibiotic prescribing.8 In a 2019 randomized trial of 101 patients with COPD exacerbations, researchers compared the GOLD strategy for antibiotic prescribing with a CRP-guided antibiotic strategy (CRP ≥ 50 mg/L) and found no difference in adverse events between study groups.9

This trial focused on point-of-care CRP-guided prescribing of antibiotics for patients with COPD exacerbations in the outpatient setting.

STUDY SUMMARY

Point-of-care CRP testing is noninferior to usual care

This open-label, multicenter, randomized controlled trial at 86 general medical practices in the United Kingdom examined whether the use of point-of-care CRP testing could reduce antibiotic use during acute exacerbations of COPD. Patients (N = 653; 650 needed to provide 81% to 90% power) were ages 40 years and older, had a diagnosis of COPD, and presented for an acute exacerbation of COPD based on the presence of at least 1 Anthonisen criteria (increased dyspnea, increase in sputum volume, and increase in purulent sputum).

Patients were randomized in a 1:1 fashion to receive care guided by point-of-care CRP testing (CRP-guided) or usual care for their COPD exacerbation. Patients in the CRP-guided group received a point-of-care CRP test as part of their assessment at presentation, or at any other appointments for COPD over the following 4 weeks.

The research team provided clinicians with CRP interpretation guidance based on the following CRP values: < 20 mg/L, antibiotics are typically not needed; 20 to 40 mg/L, antibiotics might be beneficial if purulent sputum is present; and > 40 mg/L, antibiotics are usually beneficial. Primary outcomes were patient-reported antibiotic use within 4 weeks and COPD-related health status. Of the patients who received a point-of-care CRP test, the median value was 6 mg/L; 76% had a value < 20 mg/L, 12% had values between 20 and 40 mg/L, and 12% had values > 40 mg/L. In the intention-to-treat analysis, fewer patients in the CRP-guided group reported antibiotic use vs those in the usual-care group (57% vs 77%; adjusted odds ratio [aOR] = 0.31; 95% CI, 0.20-0.47) within 4 weeks. The CRP-guided group also received fewer antibiotics at the initial visit compared to the usual-care group (48% vs 70%; aOR = 0.31; 95% CI, 0.21-0.45).

COPD-related health status was assessed with the Clinical COPD Questionnaire (score range, 0-6; a difference of 0.4 represents minimal clinical importance). At 2 weeks, the adjusted mean difference in the total health status score with the use of CRP was noninferior to usual care and was in favor of the CRP-guided group (mean difference = −0.19 points; two-sided 90% CI, −0.33 to −0.05). There was no evidence of clinically important between-group differences in pneumonia (3% vs 4%; aOR = 0.73; 95% CI, 0.29-1.82) at 6-month follow-up. Rates of hospitalization at 6 months were similar between groups (9.3% vs 8.6%; no P value provided).

Fewer patients in the CRPguided group reported antibiotic use vs those in the usual-care group within 4 weeks.


Limitations of this trial included patient report of antibiotic use and the lack of a sham test.

WHAT'S NEW

RCT provides evidence to support use of CRP testing

Point-of-care CRP testing can reduce antibiotic prescribing in patients presenting with a COPD exacerbation without affecting symptom improvement or adverse events.

CAVEATS

CRP testing may not be cost effective

CRP testing—especially point-of-care ­testing—remains expensive in many parts of the United States. A 2015 cost-effectiveness analysis of point-of-care CRP tests for respiratory tract infection in England concluded the cost of the test per patient was not cost effective.10 It is unknown if point-of-care CRP testing would be cost effective in guiding antibiotic prescribing for ­primary care providers with a focus on COPD exacerbations.

CHALLENGES TO IMPLEMENTATION 

Virtual visits and variable access may limit use

CRP-guided antibiotic prescribing may be challenging in some clinical scenarios or clinics with the rise of virtual visits and differential access in primary care clinics to point-of-care CRP tests. JFP

ACKNOWLEDGEMENT

The PURLs Surveillance System was supported in part by Grant Number UL1RR024999 from the National Center For Research Resources, a Clinical Translational Science Award to the University of Chicago. The content is solely the responsibility of the authors and does not necessarily represent the official views of the National Center For Research Resources or the National Institutes of Health. Copyright © 2021. The Family Physicians Inquiries Network. All rights reserved.

 

ILLUSTRATIVE CASE

A 55-year-old man with a history of chronic obstructive pulmonary disease (COPD) presents to you with increased sputum volume and increased dyspnea, but no fever. You diagnose a COPD exacerbation. Would point-of-care C-reactive protein (CRP) testing be a useful tool to guide antibiotic prescribing?

COPD is a common respiratory condition and one of the leading causes of death in the world.2 COPD requires chronic therapy and frequent treatment for acute exacerbations.3 A systematic review found that exacerbations occur an average of 1.3 times per year for patients with known COPD.4 Antibiotics are often prescribed for COPD exacerbations, but which patients benefit most from antibiotic treatment is unclear and identification often is based on clinical features alone. Additionally, overprescribing of antibiotics can lead to unnecessary adverse effects, drive antibiotic resistance, and be a waste of resources.5

The European Respiratory Society/American Thoracic Society (ERS/ATS) provides a conditional recommendation to consider antibiotics in ambulatory patients with COPD exacerbation based on moderate-quality evidence.6 The 2020 Global Initiative for Chronic Obstructive Lung Disease (GOLD) guidelines recommend antibiotics for moderately or severely ill patients with a COPD exacerbation who have increased cough and sputum purulence.7 While the ERS/ATS recommendations do not mention CRP, the GOLD guidelines discuss biomarkers as emerging tools in determining antibiotic utility.

Biomarkers such as procalcitonin and CRP are being examined as potential tools to distinguish which patients would benefit from antibiotic treatment in COPD exacerbations. In a 2013 study, CRP levels > 19.6 mg/L in the serum and > 15.2 mg/L in the sputum indicated a bacterial infection, but more research was needed to determine if CRP could help guide antibiotic prescribing.8 In a 2019 randomized trial of 101 patients with COPD exacerbations, researchers compared the GOLD strategy for antibiotic prescribing with a CRP-guided antibiotic strategy (CRP ≥ 50 mg/L) and found no difference in adverse events between study groups.9

This trial focused on point-of-care CRP-guided prescribing of antibiotics for patients with COPD exacerbations in the outpatient setting.

STUDY SUMMARY

Point-of-care CRP testing is noninferior to usual care

This open-label, multicenter, randomized controlled trial at 86 general medical practices in the United Kingdom examined whether the use of point-of-care CRP testing could reduce antibiotic use during acute exacerbations of COPD. Patients (N = 653; 650 needed to provide 81% to 90% power) were ages 40 years and older, had a diagnosis of COPD, and presented for an acute exacerbation of COPD based on the presence of at least 1 Anthonisen criteria (increased dyspnea, increase in sputum volume, and increase in purulent sputum).

Patients were randomized in a 1:1 fashion to receive care guided by point-of-care CRP testing (CRP-guided) or usual care for their COPD exacerbation. Patients in the CRP-guided group received a point-of-care CRP test as part of their assessment at presentation, or at any other appointments for COPD over the following 4 weeks.

The research team provided clinicians with CRP interpretation guidance based on the following CRP values: < 20 mg/L, antibiotics are typically not needed; 20 to 40 mg/L, antibiotics might be beneficial if purulent sputum is present; and > 40 mg/L, antibiotics are usually beneficial. Primary outcomes were patient-reported antibiotic use within 4 weeks and COPD-related health status. Of the patients who received a point-of-care CRP test, the median value was 6 mg/L; 76% had a value < 20 mg/L, 12% had values between 20 and 40 mg/L, and 12% had values > 40 mg/L. In the intention-to-treat analysis, fewer patients in the CRP-guided group reported antibiotic use vs those in the usual-care group (57% vs 77%; adjusted odds ratio [aOR] = 0.31; 95% CI, 0.20-0.47) within 4 weeks. The CRP-guided group also received fewer antibiotics at the initial visit compared to the usual-care group (48% vs 70%; aOR = 0.31; 95% CI, 0.21-0.45).

COPD-related health status was assessed with the Clinical COPD Questionnaire (score range, 0-6; a difference of 0.4 represents minimal clinical importance). At 2 weeks, the adjusted mean difference in the total health status score with the use of CRP was noninferior to usual care and was in favor of the CRP-guided group (mean difference = −0.19 points; two-sided 90% CI, −0.33 to −0.05). There was no evidence of clinically important between-group differences in pneumonia (3% vs 4%; aOR = 0.73; 95% CI, 0.29-1.82) at 6-month follow-up. Rates of hospitalization at 6 months were similar between groups (9.3% vs 8.6%; no P value provided).

Fewer patients in the CRPguided group reported antibiotic use vs those in the usual-care group within 4 weeks.


Limitations of this trial included patient report of antibiotic use and the lack of a sham test.

WHAT'S NEW

RCT provides evidence to support use of CRP testing

Point-of-care CRP testing can reduce antibiotic prescribing in patients presenting with a COPD exacerbation without affecting symptom improvement or adverse events.

CAVEATS

CRP testing may not be cost effective

CRP testing—especially point-of-care ­testing—remains expensive in many parts of the United States. A 2015 cost-effectiveness analysis of point-of-care CRP tests for respiratory tract infection in England concluded the cost of the test per patient was not cost effective.10 It is unknown if point-of-care CRP testing would be cost effective in guiding antibiotic prescribing for ­primary care providers with a focus on COPD exacerbations.

CHALLENGES TO IMPLEMENTATION 

Virtual visits and variable access may limit use

CRP-guided antibiotic prescribing may be challenging in some clinical scenarios or clinics with the rise of virtual visits and differential access in primary care clinics to point-of-care CRP tests. JFP

ACKNOWLEDGEMENT

The PURLs Surveillance System was supported in part by Grant Number UL1RR024999 from the National Center For Research Resources, a Clinical Translational Science Award to the University of Chicago. The content is solely the responsibility of the authors and does not necessarily represent the official views of the National Center For Research Resources or the National Institutes of Health. Copyright © 2021. The Family Physicians Inquiries Network. All rights reserved.

References
  1. Butler CC, Gillespie D, White P, et al. C-reactive protein testing to guide antibiotic prescribing for COPD exacerbations. N Engl J Med. 2019;381:111-120.
  2. Lopez AD, Mathers CD, Ezzati M, et al. Global Burden of Disease and Risk Factors. The World Bank; 2006.
  3. Buist AS, McBurnie MA, Vollmer WM, et al. International variation in the prevalence of COPD (the BOLD Study): a population-based prevalence study. Lancet. 2007;370:741-750.
  4. Singh J, Palda V, Stanbrook M, et al. Corticosteroid therapy for patients with acute exacerbations of chronic obstructive pulmonary disease: a systematic review. Arch Intern Med. 2002;162:2527-2536.
  5. Schroeck JL, Ruh CA, Sellick JA, et al. Factors associated with antibiotic misuse in outpatient treatment for upper respiratory tract infections. Antimicrob Agents Chemother. 2015;59:3848-3852.
  6. Wedzicha JA, Miravitlles M, Hurst JR, et al. Management of COPD exacerbations: a European Respiratory Society/American Thoracic Society guideline. Eur Respir J. 2017;49:1600791.
  7. Global Initiative for Chronic Obstructive Lung Disease. Global Strategy for the Diagnosis, and Management and Prevention of Chronic Obstructive Pulmonary Disease (2020 report). Accessed May 12, 2021. https://goldcopd.org/gold-reports/
  8. Peng C, Tian C, Zhang Y, et al. C-reactive protein levels predict bacterial exacerbation in patients with chronic obstructive pulmonary disease. Am J Med Sci. 2013;345:190-194.
  9. Prins H, Duijkers R, van der Valk P, et al. CRP-guided antibiotic treatment in acute exacerbations of COPD in hospital admissions. Eur Respir J. 2019;53:1802014.
  10. Hunter R. Cost-effectiveness of point-of-care C-reactive protein tests for respiratory tract infection in primary care in England. Adv Ther. 2015;32:69-85.
References
  1. Butler CC, Gillespie D, White P, et al. C-reactive protein testing to guide antibiotic prescribing for COPD exacerbations. N Engl J Med. 2019;381:111-120.
  2. Lopez AD, Mathers CD, Ezzati M, et al. Global Burden of Disease and Risk Factors. The World Bank; 2006.
  3. Buist AS, McBurnie MA, Vollmer WM, et al. International variation in the prevalence of COPD (the BOLD Study): a population-based prevalence study. Lancet. 2007;370:741-750.
  4. Singh J, Palda V, Stanbrook M, et al. Corticosteroid therapy for patients with acute exacerbations of chronic obstructive pulmonary disease: a systematic review. Arch Intern Med. 2002;162:2527-2536.
  5. Schroeck JL, Ruh CA, Sellick JA, et al. Factors associated with antibiotic misuse in outpatient treatment for upper respiratory tract infections. Antimicrob Agents Chemother. 2015;59:3848-3852.
  6. Wedzicha JA, Miravitlles M, Hurst JR, et al. Management of COPD exacerbations: a European Respiratory Society/American Thoracic Society guideline. Eur Respir J. 2017;49:1600791.
  7. Global Initiative for Chronic Obstructive Lung Disease. Global Strategy for the Diagnosis, and Management and Prevention of Chronic Obstructive Pulmonary Disease (2020 report). Accessed May 12, 2021. https://goldcopd.org/gold-reports/
  8. Peng C, Tian C, Zhang Y, et al. C-reactive protein levels predict bacterial exacerbation in patients with chronic obstructive pulmonary disease. Am J Med Sci. 2013;345:190-194.
  9. Prins H, Duijkers R, van der Valk P, et al. CRP-guided antibiotic treatment in acute exacerbations of COPD in hospital admissions. Eur Respir J. 2019;53:1802014.
  10. Hunter R. Cost-effectiveness of point-of-care C-reactive protein tests for respiratory tract infection in primary care in England. Adv Ther. 2015;32:69-85.
Issue
The Journal of Family Practice - 70(5)
Issue
The Journal of Family Practice - 70(5)
Page Number
244-246
Page Number
244-246
Publications
Publications
Topics
Article Type
Sections
PURLs Copyright
Copyright © 2021. The Family Physicians Inquiries Network. All rights reserved.
Inside the Article

PRACTICE CHANGER

Consider C-reactive protein–guided prescribing of antibiotics in acute chronic obstructive pulmonary disease exacerbations in the outpatient setting, as it results in fewer antibiotic prescriptions without adverse effects.1

STRENGTH OF RECOMMENDATION

B: Based on a single randomized controlled trial.1

Butler CC, Gillespie D, White P, et al. C-Reactive protein testing to guide antibiotic prescribing for COPD exacerbations. N Engl J Med. 2019;381:111-120.

Disallow All Ads
Content Gating
No Gating (article Unlocked/Free)
Alternative CME
Disqus Comments
Default
Use ProPublica
Hide sidebar & use full width
render the right sidebar.
Attach Teaching Materials
Conference Recap Checkbox
Not Conference Recap
Clinical Edge
Display the Slideshow in this Article
Medscape Article
Display survey writer
Reuters content
Disable Inline Native ads
WebMD Article
Article PDF Media
Image
Teambase ID
18001CC3.SIG
Disable zoom
Off
Attach Teaching Materials

Is NPH associated with fewer adverse events than analog basal insulin for adults with T2D?

Article Type
Changed
Tue, 05/03/2022 - 15:06
Display Headline
Is NPH associated with fewer adverse events than analog basal insulin for adults with T2D?

Evidence summary

No difference in overall hypoglycemia risk between glargine and NPH

A 2015 systematic review and meta-analysis of 28 RCTs compared efficacy and safety outcomes for insulin glargine, NPH insulin, premixed insulin preparations, and insulin detemir in 12,669 adults with type 2 ­diabetes (T2D) who were also taking an oral antidiabetic drug (OAD).1 In the comparison of glargine to NPH, there was no difference in risk for hypoglycemia (5 trials; N not provided; risk ratio [RR] = 0.92; 0.84-1.001).

Symptomatic hypoglycemia (6 RCTs; RR = 0.89; 0.83-0.96) and nocturnal hypoglycemia (6 RCTs; RR = 0.63; 0.51-0.77) occurred significantly less frequently in those treated with glargine and an OAD compared to NPH and an OAD. The risk for severe hypoglycemia was not different between regimens (5 RCTs; RR = 0.76; 0.47-1.23). Weight gain was also similar (6 RCTs; weighted mean difference [WMD] = 0.36 kg [–0.12 to 0.84]). This review was limited by the fact that many of the trials were of moderate quality, the majority were funded by pharmaceutical companies, fasting glucose goals varied between trials, and some trials had a short duration (6 months).

There may be some advantages of glargine over NPH

A 2008 meta-analysis of 12 RCTs (5 of which were not included in the 2015 review) with 4385 patients with T2D compared fasting plasma glucose (FPG), A1C, hypoglycemia, and body weight for patients treated with NPH vs with glargine.2 Researchers found a significant difference in patient-reported hypoglycemia (10 trials; N not provided; 59% vs 53%; P < .001), symptomatic hypoglycemia (6 trials; 51% vs 43%; P < .0001), and nocturnal hypoglycemia (8 trials; 33% vs 19%; P < .001), favoring glargine over NPH. However, there was no difference between these 2 groups in confirmed hypoglycemia (2 trials; 10% vs 6.3%; P = .11) or severe hypoglycemia (7 trials; 2.4% vs 1.4%; P = .07). Of note, there was no difference between groups in FPG or A1C and a smaller weight gain in the NPH group (6 trials; WMD = 0.33 kg; 95% CI, –0.61 to –0.06). This review did not assess potential biases in the included trials.

 

Other results indicate a significant benefit from glargine

A 2014 RCT (published after the systematic review search date) compared hypoglycemia risk between NPH and glargine in 1017 adults ages 30 to 70 years who’d had T2D for at least 1 year.3 Patients were randomized to receive an OAD paired with either once-daily glargine or twice-daily NPH. Insulin doses were titrated over the first 3 years of the study to achieve standard glycemic control (described as FPG < 120 mg/dL; this goal was changed to < 100 mg/dL after the first year).

Over 5 years, once-daily glargine resulted in a significantly lower risk for all symptomatic hypoglycemia (odds ratio [OR] = 0.71; 95% CI, 0.52-0.98) and for any severe event (OR = 0.62; 95% CI, 0.41-0.95) compared to NPH. Using a logistic regression model, the authors predicted that if 25 patients were treated with NPH instead of glargine, 1 additional patient would experience at least 1 severe hypoglycemic event. This trial was funded by a pharmaceutical company.

Hypoglycemia requiring hospital care was similar for basal insulin and NPH

A 2018 retrospective observational study (N = 25,489) analyzed the association between the initiation of basal insulin analogs vs NPH with hypoglycemia-related ED visits or hospital admissions.4 Adults older than 19 years with clinically recognized diabetes were identified using electronic medical records; those included in the analysis had newly initiated basal insulin therapy during the prior 12 months. Data was gathered via chart review.

The difference in ED visits or hospital admissions was not different between groups (mean difference = 3.1 events per 100 person-years; 95% CI, –1.5 to 7.7). Among 4428 patients matched by propensity score, there was again no difference for hypoglycemia-related ED visits or hospital admissions with insulin analog use (adjusted hazard ratio = 1.16; 95% CI, 0.71-1.78).

Editor’s takeaway

Meta-analysis of large RCTs shows the glargine insulin adverse effects profile, specifically nonsevere hypoglycemia, to be inconsistently better than NPH. These small differences, plus once-daily dosing, may encourage prescribing of analog basal insulin, but price and the need for more than once-daily dosing remain worthy considerations.

References

1. Rys P, Wojciechowski P, Rogoz-Sitek A, et al. Systematic review and meta-analysis of randomized clinical trials comparing efficacy and safety outcomes of insulin glargine with NPH insulin, premixed insulin preparations or with insulin detemir in type 2 diabetes mellitus. Acta Diabetol. 2015;52:649-662. doi:10.1007/s00592-014-0698-4

2. Bazzano LA, Lee LJ, Shi L, et al. Safety and efficacy of glargine compared with NPH insulin for the treatment of type 2 diabetes: a meta-analysis of randomized controlled trials. Diabet Med. 2008;25:924-932. doi:10.1111/j.1464-5491.2008.02517.x

3. Rosenstock J, Fonseca V, Schinzel S, et al. Reduced risk of hypoglycemia with once-daily glargine versus twice-daily NPH and number needed to harm with NPH to demonstrate the risk of one additional hypoglycemic event in type 2 diabetes: evidence from a long-term controlled trial. J Diabetes Complications. 2014;28:742-749. doi:10.1016/j.jdiacomp.2014.04.003

4. Lipska KJ, Parker MM, Moffet HH, et al. Association of initiation of basal insulin analogs vs neutral protamine Hagedorn insulin with hypoglycemia-related emergency department visits or hospital admissions and with glycemic control in patients with type 2 diabetes. JAMA. 2018;320:53-62. doi:10.1001/jama.2018.7993

Article PDF
Author and Disclosure Information

Corey Lyon, DO
Swetha Iruku, MD
Molly Hoss, MD

University of Colorado Family Medicine Residency

Kristen DeSanto, MSLS, MS, RD, AHIP
University of Colorado Health Sciences Library, Denver

ASSISTANT EDITOR
Richard Guthmann, MD, MPH

Advocate Illinois Masonic Family Medicine Residency

Issue
The Journal of Family Practice - 70(4)
Publications
Topics
Page Number
E5-E6
Sections
Author and Disclosure Information

Corey Lyon, DO
Swetha Iruku, MD
Molly Hoss, MD

University of Colorado Family Medicine Residency

Kristen DeSanto, MSLS, MS, RD, AHIP
University of Colorado Health Sciences Library, Denver

ASSISTANT EDITOR
Richard Guthmann, MD, MPH

Advocate Illinois Masonic Family Medicine Residency

Author and Disclosure Information

Corey Lyon, DO
Swetha Iruku, MD
Molly Hoss, MD

University of Colorado Family Medicine Residency

Kristen DeSanto, MSLS, MS, RD, AHIP
University of Colorado Health Sciences Library, Denver

ASSISTANT EDITOR
Richard Guthmann, MD, MPH

Advocate Illinois Masonic Family Medicine Residency

Article PDF
Article PDF

Evidence summary

No difference in overall hypoglycemia risk between glargine and NPH

A 2015 systematic review and meta-analysis of 28 RCTs compared efficacy and safety outcomes for insulin glargine, NPH insulin, premixed insulin preparations, and insulin detemir in 12,669 adults with type 2 ­diabetes (T2D) who were also taking an oral antidiabetic drug (OAD).1 In the comparison of glargine to NPH, there was no difference in risk for hypoglycemia (5 trials; N not provided; risk ratio [RR] = 0.92; 0.84-1.001).

Symptomatic hypoglycemia (6 RCTs; RR = 0.89; 0.83-0.96) and nocturnal hypoglycemia (6 RCTs; RR = 0.63; 0.51-0.77) occurred significantly less frequently in those treated with glargine and an OAD compared to NPH and an OAD. The risk for severe hypoglycemia was not different between regimens (5 RCTs; RR = 0.76; 0.47-1.23). Weight gain was also similar (6 RCTs; weighted mean difference [WMD] = 0.36 kg [–0.12 to 0.84]). This review was limited by the fact that many of the trials were of moderate quality, the majority were funded by pharmaceutical companies, fasting glucose goals varied between trials, and some trials had a short duration (6 months).

There may be some advantages of glargine over NPH

A 2008 meta-analysis of 12 RCTs (5 of which were not included in the 2015 review) with 4385 patients with T2D compared fasting plasma glucose (FPG), A1C, hypoglycemia, and body weight for patients treated with NPH vs with glargine.2 Researchers found a significant difference in patient-reported hypoglycemia (10 trials; N not provided; 59% vs 53%; P < .001), symptomatic hypoglycemia (6 trials; 51% vs 43%; P < .0001), and nocturnal hypoglycemia (8 trials; 33% vs 19%; P < .001), favoring glargine over NPH. However, there was no difference between these 2 groups in confirmed hypoglycemia (2 trials; 10% vs 6.3%; P = .11) or severe hypoglycemia (7 trials; 2.4% vs 1.4%; P = .07). Of note, there was no difference between groups in FPG or A1C and a smaller weight gain in the NPH group (6 trials; WMD = 0.33 kg; 95% CI, –0.61 to –0.06). This review did not assess potential biases in the included trials.

 

Other results indicate a significant benefit from glargine

A 2014 RCT (published after the systematic review search date) compared hypoglycemia risk between NPH and glargine in 1017 adults ages 30 to 70 years who’d had T2D for at least 1 year.3 Patients were randomized to receive an OAD paired with either once-daily glargine or twice-daily NPH. Insulin doses were titrated over the first 3 years of the study to achieve standard glycemic control (described as FPG < 120 mg/dL; this goal was changed to < 100 mg/dL after the first year).

Over 5 years, once-daily glargine resulted in a significantly lower risk for all symptomatic hypoglycemia (odds ratio [OR] = 0.71; 95% CI, 0.52-0.98) and for any severe event (OR = 0.62; 95% CI, 0.41-0.95) compared to NPH. Using a logistic regression model, the authors predicted that if 25 patients were treated with NPH instead of glargine, 1 additional patient would experience at least 1 severe hypoglycemic event. This trial was funded by a pharmaceutical company.

Hypoglycemia requiring hospital care was similar for basal insulin and NPH

A 2018 retrospective observational study (N = 25,489) analyzed the association between the initiation of basal insulin analogs vs NPH with hypoglycemia-related ED visits or hospital admissions.4 Adults older than 19 years with clinically recognized diabetes were identified using electronic medical records; those included in the analysis had newly initiated basal insulin therapy during the prior 12 months. Data was gathered via chart review.

The difference in ED visits or hospital admissions was not different between groups (mean difference = 3.1 events per 100 person-years; 95% CI, –1.5 to 7.7). Among 4428 patients matched by propensity score, there was again no difference for hypoglycemia-related ED visits or hospital admissions with insulin analog use (adjusted hazard ratio = 1.16; 95% CI, 0.71-1.78).

Editor’s takeaway

Meta-analysis of large RCTs shows the glargine insulin adverse effects profile, specifically nonsevere hypoglycemia, to be inconsistently better than NPH. These small differences, plus once-daily dosing, may encourage prescribing of analog basal insulin, but price and the need for more than once-daily dosing remain worthy considerations.

Evidence summary

No difference in overall hypoglycemia risk between glargine and NPH

A 2015 systematic review and meta-analysis of 28 RCTs compared efficacy and safety outcomes for insulin glargine, NPH insulin, premixed insulin preparations, and insulin detemir in 12,669 adults with type 2 ­diabetes (T2D) who were also taking an oral antidiabetic drug (OAD).1 In the comparison of glargine to NPH, there was no difference in risk for hypoglycemia (5 trials; N not provided; risk ratio [RR] = 0.92; 0.84-1.001).

Symptomatic hypoglycemia (6 RCTs; RR = 0.89; 0.83-0.96) and nocturnal hypoglycemia (6 RCTs; RR = 0.63; 0.51-0.77) occurred significantly less frequently in those treated with glargine and an OAD compared to NPH and an OAD. The risk for severe hypoglycemia was not different between regimens (5 RCTs; RR = 0.76; 0.47-1.23). Weight gain was also similar (6 RCTs; weighted mean difference [WMD] = 0.36 kg [–0.12 to 0.84]). This review was limited by the fact that many of the trials were of moderate quality, the majority were funded by pharmaceutical companies, fasting glucose goals varied between trials, and some trials had a short duration (6 months).

There may be some advantages of glargine over NPH

A 2008 meta-analysis of 12 RCTs (5 of which were not included in the 2015 review) with 4385 patients with T2D compared fasting plasma glucose (FPG), A1C, hypoglycemia, and body weight for patients treated with NPH vs with glargine.2 Researchers found a significant difference in patient-reported hypoglycemia (10 trials; N not provided; 59% vs 53%; P < .001), symptomatic hypoglycemia (6 trials; 51% vs 43%; P < .0001), and nocturnal hypoglycemia (8 trials; 33% vs 19%; P < .001), favoring glargine over NPH. However, there was no difference between these 2 groups in confirmed hypoglycemia (2 trials; 10% vs 6.3%; P = .11) or severe hypoglycemia (7 trials; 2.4% vs 1.4%; P = .07). Of note, there was no difference between groups in FPG or A1C and a smaller weight gain in the NPH group (6 trials; WMD = 0.33 kg; 95% CI, –0.61 to –0.06). This review did not assess potential biases in the included trials.

 

Other results indicate a significant benefit from glargine

A 2014 RCT (published after the systematic review search date) compared hypoglycemia risk between NPH and glargine in 1017 adults ages 30 to 70 years who’d had T2D for at least 1 year.3 Patients were randomized to receive an OAD paired with either once-daily glargine or twice-daily NPH. Insulin doses were titrated over the first 3 years of the study to achieve standard glycemic control (described as FPG < 120 mg/dL; this goal was changed to < 100 mg/dL after the first year).

Over 5 years, once-daily glargine resulted in a significantly lower risk for all symptomatic hypoglycemia (odds ratio [OR] = 0.71; 95% CI, 0.52-0.98) and for any severe event (OR = 0.62; 95% CI, 0.41-0.95) compared to NPH. Using a logistic regression model, the authors predicted that if 25 patients were treated with NPH instead of glargine, 1 additional patient would experience at least 1 severe hypoglycemic event. This trial was funded by a pharmaceutical company.

Hypoglycemia requiring hospital care was similar for basal insulin and NPH

A 2018 retrospective observational study (N = 25,489) analyzed the association between the initiation of basal insulin analogs vs NPH with hypoglycemia-related ED visits or hospital admissions.4 Adults older than 19 years with clinically recognized diabetes were identified using electronic medical records; those included in the analysis had newly initiated basal insulin therapy during the prior 12 months. Data was gathered via chart review.

The difference in ED visits or hospital admissions was not different between groups (mean difference = 3.1 events per 100 person-years; 95% CI, –1.5 to 7.7). Among 4428 patients matched by propensity score, there was again no difference for hypoglycemia-related ED visits or hospital admissions with insulin analog use (adjusted hazard ratio = 1.16; 95% CI, 0.71-1.78).

Editor’s takeaway

Meta-analysis of large RCTs shows the glargine insulin adverse effects profile, specifically nonsevere hypoglycemia, to be inconsistently better than NPH. These small differences, plus once-daily dosing, may encourage prescribing of analog basal insulin, but price and the need for more than once-daily dosing remain worthy considerations.

References

1. Rys P, Wojciechowski P, Rogoz-Sitek A, et al. Systematic review and meta-analysis of randomized clinical trials comparing efficacy and safety outcomes of insulin glargine with NPH insulin, premixed insulin preparations or with insulin detemir in type 2 diabetes mellitus. Acta Diabetol. 2015;52:649-662. doi:10.1007/s00592-014-0698-4

2. Bazzano LA, Lee LJ, Shi L, et al. Safety and efficacy of glargine compared with NPH insulin for the treatment of type 2 diabetes: a meta-analysis of randomized controlled trials. Diabet Med. 2008;25:924-932. doi:10.1111/j.1464-5491.2008.02517.x

3. Rosenstock J, Fonseca V, Schinzel S, et al. Reduced risk of hypoglycemia with once-daily glargine versus twice-daily NPH and number needed to harm with NPH to demonstrate the risk of one additional hypoglycemic event in type 2 diabetes: evidence from a long-term controlled trial. J Diabetes Complications. 2014;28:742-749. doi:10.1016/j.jdiacomp.2014.04.003

4. Lipska KJ, Parker MM, Moffet HH, et al. Association of initiation of basal insulin analogs vs neutral protamine Hagedorn insulin with hypoglycemia-related emergency department visits or hospital admissions and with glycemic control in patients with type 2 diabetes. JAMA. 2018;320:53-62. doi:10.1001/jama.2018.7993

References

1. Rys P, Wojciechowski P, Rogoz-Sitek A, et al. Systematic review and meta-analysis of randomized clinical trials comparing efficacy and safety outcomes of insulin glargine with NPH insulin, premixed insulin preparations or with insulin detemir in type 2 diabetes mellitus. Acta Diabetol. 2015;52:649-662. doi:10.1007/s00592-014-0698-4

2. Bazzano LA, Lee LJ, Shi L, et al. Safety and efficacy of glargine compared with NPH insulin for the treatment of type 2 diabetes: a meta-analysis of randomized controlled trials. Diabet Med. 2008;25:924-932. doi:10.1111/j.1464-5491.2008.02517.x

3. Rosenstock J, Fonseca V, Schinzel S, et al. Reduced risk of hypoglycemia with once-daily glargine versus twice-daily NPH and number needed to harm with NPH to demonstrate the risk of one additional hypoglycemic event in type 2 diabetes: evidence from a long-term controlled trial. J Diabetes Complications. 2014;28:742-749. doi:10.1016/j.jdiacomp.2014.04.003

4. Lipska KJ, Parker MM, Moffet HH, et al. Association of initiation of basal insulin analogs vs neutral protamine Hagedorn insulin with hypoglycemia-related emergency department visits or hospital admissions and with glycemic control in patients with type 2 diabetes. JAMA. 2018;320:53-62. doi:10.1001/jama.2018.7993

Issue
The Journal of Family Practice - 70(4)
Issue
The Journal of Family Practice - 70(4)
Page Number
E5-E6
Page Number
E5-E6
Publications
Publications
Topics
Article Type
Display Headline
Is NPH associated with fewer adverse events than analog basal insulin for adults with T2D?
Display Headline
Is NPH associated with fewer adverse events than analog basal insulin for adults with T2D?
Sections
PURLs Copyright
Evidence-based answers from the Family Physicians Inquiries Network
Inside the Article

EVIDENCE-BASED ANSWER:

NO. Insulin glargine may lead to less patient-reported, ­symptomatic, and nocturnal hypoglycemia, although overall, there may not be a difference in the risk for severe hypoglycemia or­ hypoglycemia-related emergency ­department (ED) visits and hospitalizations (strength of recommendation [SOR]: B, systematic review of randomized controlled trials [RCTs], individual RCTs, and observational study).

Disallow All Ads
Content Gating
No Gating (article Unlocked/Free)
Alternative CME
Disqus Comments
Default
Use ProPublica
Hide sidebar & use full width
render the right sidebar.
Conference Recap Checkbox
Not Conference Recap
Clinical Edge
Display the Slideshow in this Article
Medscape Article
Display survey writer
Reuters content
Disable Inline Native ads
WebMD Article
Article PDF Media
Image
Teambase ID
18001C5C.SIG
Disable zoom
Off

Consider this Rx for patients with high triglycerides?

Article Type
Changed
Mon, 05/03/2021 - 14:39

ILLUSTRATIVE CASE

A 63-year-old man with a medical history significant for myocardial infarction (MI) 5 years ago presents to you for an annual exam. His medications include a daily aspirin, ­angiotensin-converting enzyme inhibitor, beta-blocker, and a high-intensity statin for coronary artery disease (CAD). On his fasting lipid panel, his low-density lipoprotein (LDL) level is 70 mg/dL, but his triglycerides remain elevated at 200 mg/dL despite dietary changes.

In addition to lifestyle modifications, what can be done to reduce his risk of another MI?

Patients with known cardiovascular disease (CVD) or multiple risk factors for CVD are at high risk of cardiovascular events, even when taking primary or secondary preventive medications such as statins.2,3 In these patients, elevated triglycerides are an independent risk factor for increased rates of cardiovascular events.4,5

The 2018 American College of Cardiology/American Heart Association (ACC/AHA) guidelines for the treatment of blood cholesterol recommend statin therapy for moderate (175-499 mg/dL) to severe (≥ 500 mg/dL) hypertriglyceridemia in appropriate patients with atherosclerotic CVD risk ≥ 7.5%, after appropriately addressing secondary causes of hypertriglycidemia.6

Previous studies have shown no benefit from combination therapy with triglyceride-lowering medications (eg, extended-release niacin and fibrates) and statins, compared with statin monotherapy.7 A recent meta-analysis concluded that omega-3 fatty acid supplements offer no reduction in cardiovascular morbidity or mortality, whether taken with or without statins.8

Interestingly, the randomized controlled Japan EPA Lipid Intervention Study (JELIS) demonstrated fewer major coronary events in patients with elevated cholesterol, with or without CAD, who took eicosapentaenoic acid (EPA)—a subtype of omega-3 fatty acids—plus a statin, compared with statin monotherapy.9

The REDUCE-IT trial evaluated icosapent ethyl, a highly purified EPA that has been shown to reduce triglycerides and, at the time this study was conducted, was approved for use solely for the reduction of triglyceride levels in adults with severe hypertriglyceridemia.10,11

Continue to: Study Summary

 

 

STUDY SUMMARY

Patients with known CVD had fewercardiovascular events on icosapent ethyl

The multicenter, randomized controlled REDUCE-IT trial evaluated the effectiveness of icosapent ethyl, 2 g orally twice daily, on cardiovascular outcomes.1 A total of 8179 patients, ≥ 45 years of age with hypertriglyceridemia and known CVD or ≥ 50 years with diabetes and at least 1 additional risk factor and no known CVD, were enrolled at 473 participating sites in 11 countries, including the United States.

Patients had a triglyceride level of 150 to 499 mg/dL and an LDL cholesterol level of 41 to 100 mg/dL, and were taking a stable dose of a statin for at least 4 weeks. The enrollment protocol was amended to increase the lower limit of triglycerides from 150 to 200 mg/dL about one-third of the way through the study. Among the study population, 70.7% of patients were enrolled for secondary prevention (ie, had established CVD) and 29.3% of patients were enrolled for primary prevention (ie, had diabetes and at least 1 additional risk factor but no known CVD). Exclusion criteria included severe heart failure, active severe liver disease, glycated hemoglobin > 10%, a planned surgical cardiac intervention, history of pancreatitis, or allergies to fish or shellfish products.

Outcomes. The primary end point was a composite outcome of cardiovascular death, nonfatal MI, nonfatal stroke, coronary revascularization, or unstable angina.

Results. The median duration of follow-up was 4.9 years. From baseline to 1 year, the median change in triglycerides was an 18% reduction in the icosapent ethyl group but a 2% increase in the placebo group. Fewer patients in the icosapent ethyl group than the placebo group had a composite outcome event (17% vs 22%, respectively; hazard ratio [HR] = 0.75; 95% confidence interval [CI], 0.68-0.83; number needed to treat [NNT] to avoid 1 primary end point event = 21). Patients with known CVD had fewer composite outcome events in the icosapent ethyl group than the placebo group (19% vs 26%; HR = 0.73; 95% CI, 0.65-0.81; NNT = 14) but not in the primary prevention group vs the placebo group (12% vs 14%; HR = 0.88; 95% CI, 0.70-1.1).

In the entire population, all individual outcomes in the composite were significantly fewer in the icosapent ethyl group (cardiovascular death: HR = 0.8; 95% CI, 0.66-0.98; fatal or nonfatal MI: HR = 0.69; 95% CI, 0.58-0.81; revascularization: HR = 0.65; 95% CI, 0.55-0.78; unstable angina: HR = 0.68; 95% CI, 0.53-0.87; and fatal or nonfatal stroke: HR = 0.72; 95% CI, 0.55-0.93). All-cause mortality did not differ between groups (HR = 0.87; 95% CI, 0.74-1.02).

No significant differences in adverse events leading to discontinuation of the drug were reported between groups. Atrial fibrillation occurred more frequently in the icosapent ethyl group (5.3% vs 3.9%), but anemia (4.7% vs 5.8%) and gastrointestinal adverse events (33% vs 35%) were less common.

Continue to: What's New

 

 

WHAT’S NEW

First RCT to demonstrate valueof pairing icosapent ethyl with a statin

Many prior studies on use of omega-3 fatty acid supplements to treat hypertriglyceridemia did not show any benefit, possibly due to a low dose or low ratio of EPA in the study drug.8 One trial (JELIS) with favorable results was an open-label study, limited to patients in Japan. The REDUCE-IT study was the first randomized, placebo-controlled trial to show that icosapent ethyl treatment for hypertriglyceridemia in patients with known CVD who are taking a statin results in fewer cardiovascular events than statin use alone.

In this multinational trial of patients with hypertriglyceridemia and CVD or cardiovascular risk factors, icosapent ethyl significantly reduced the risk of cardiovascular events.

Also worth noting: Since publication of the REDUCE-IT study, the FDA has approved an expanded indication for icosapent ethyl for reduction of risk of cardiovascular events in statin-treated patients with hypertriglyceridemia and established CVD or diabetes and ≥ 2 additional cardiovascular risk factors.11

CAVEATS

Drug’s benefit was not linkedto triglyceride level reductions

The cardiovascular benefits of icosapent ethyl were obtained irrespective of triglyceride levels achieved. This raises the question of other potential mechanisms of action of icosapent ethyl in achieving cardiovascular benefit. However, this should not preclude the use of icosapent ethyl for secondary prevention in appropriate patients.

CHALLENGES TO IMPLEMENTATION

Medication is pricey

Icosapent ethyl is an expensive medication, currently priced at an estimated $351/month using a nationally available discount pharmacy plan, although additional manufacturer’s discounts may apply.12,13 The cost of the medication could be a consideration for widespread implementation of this recommendation.

ACKNOWLEDGEMENT

The PURLs Surveillance System was supported in part by Grant Number UL1RR024999 from the National Center For Research Resources, a Clinical Translational Science Award to the University of Chicago. The content is solely the responsibility of the authors and does not necessarily represent the official views of the National Center For Research Resources or the National Institutes of Health.

Copyright © 2020. The Family Physicians Inquiries Network. All rights reserved.

Files
References

1. Bhatt DL, Steg PG, Miller M, et al; REDUCE-IT Investigators. Cardiovascular risk reduction with icosapent ethyl for hypertriglyceridemia. N Engl J Med. 2019;380:11-22.

2. Bhatt DL, Eagle KA, Ohman EM, et al; REACH Registry Investigators. Comparative determinants of 4-year cardiovascular event rates in stable outpatients at risk of or with atherothrombosis. JAMA. 2010;304:1350-1357.

3. Cannon CP, Braunwald E, McCabe CH, et al; Pravastatin or Atorvastatin Evaluation and Infection Therapy–Thrombolysis in Myocardial Infarction 22 Investigators. Intensive versus moderate lipid lowering with statins after acute coronary syndromes [published correction appears in N Engl J Med. 2006;354:778]. N Engl J Med. 2004;350:1495-1504.

4. Klempfner R, Erez A, Sagit BZ, et al. Elevated triglyceride level is independently associated with increased all-cause mortality in patients with established coronary heart disease: twenty-two-year follow-up of the Bezafibrate Infarction Prevention Study and Registry [published correction appears in Circ Cardiovasc Qual Outcomes. 2016;9:613]. Circ Cardiovasc Qual Outcomes. 2016;9:100-108.

5. Nichols GA, Philip S, Reynolds K, Granowitz CB, Fazio S. Increased cardiovascular risk in hypertriglyceridemic patients with statin-controlled LDL cholesterol. J Clin Endocrinol Metab. 2018;103:3019-3027.

6. Grundy SM, Stone NJ, Bailey AL, et al. 2018 AHA/ACC/AACVPR/AAPA/ABC/ACPM/ADA/AGS/APhA/ASPC/NLA/PCNA guideline on the management of blood cholesterol: a report of the American College of Cardiology/American Heart Association Task Force on Clinical Practice Guidelines [published correction appears in J Am Coll Cardiol. 2019;73:3237-3241]. J Am Coll Cardiol. 2019;73:e285-e350.

7. Ganda OP, Bhatt DL, Mason RP, Miller M, Boden WE. Unmet need for adjunctive dyslipidemia therapy in hypertriglyceridemia management. J Am Coll Cardiol. 2018;72:330-343.

8. Aung T, Halsey J, Kromhout D, et al; Omega-3 Treatment Trialists’ Collaboration. Associations of omega-3 fatty acid supplement use with cardiovascular disease risks: meta-analysis of 10 trials involving 77 917 individuals. JAMA Cardiol. 2018;3:225-234.

9. Yokoyama M, Origasa H, Matsuzaki M, et al; Japan EPA lipid intervention study (JELIS) Investigators. Effects of eicosapentaenoic acid on major coronary events in hypercholesterolaemic patients (JELIS): a randomised open-label, blinded endpoint analysis [published correction appears in Lancet. 2007;370:220]. Lancet. 2007;369:1090-1098.

10. Ballantyne CM, Bays HE, Kastelein JJ, et al. Efficacy and safety of eicosapentaenoic acid ethyl ester (AMR101) therapy in statin-treated patients with persistent high triglycerides (from the ANCHOR study). Am J Cardiol. 2012;110:984-992.

11. FDA approves use of drug to reduce risk of cardiovascular events in certain adult patient groups [news release]. Silver Spring, MD: US Food and Drug Administration; December 13, 2019. www.fda.gov/news-events/press-announcements/fda-approves-use-drug-reduce-risk-cardiovascular-events-certain-adult-patient-groups. Accessed November 30, 2020.

12. Vascepa. GoodRx. www.goodrx.com/vascepa. Accessed November 30, 2020.

13. The VASCEPA Savings Program. www.vascepa.com/getting-started/savings-card/. Accessed November 30, 2020.

Article PDF
Author and Disclosure Information

University of Colorado Department of Family Medicine, Aurora ; University of Colorado Family Medicine Residency, Denver 

E D I T O R
Shailendra Prasad, MBBS, MPH
University of Minnesota North Memorial Family Medicine Residency Program, Minneapolis

Issue
The Journal of Family Practice - 69(10)
Publications
Topics
Page Number
518-525
Sections
Files
Files
Author and Disclosure Information

University of Colorado Department of Family Medicine, Aurora ; University of Colorado Family Medicine Residency, Denver 

E D I T O R
Shailendra Prasad, MBBS, MPH
University of Minnesota North Memorial Family Medicine Residency Program, Minneapolis

Author and Disclosure Information

University of Colorado Department of Family Medicine, Aurora ; University of Colorado Family Medicine Residency, Denver 

E D I T O R
Shailendra Prasad, MBBS, MPH
University of Minnesota North Memorial Family Medicine Residency Program, Minneapolis

Article PDF
Article PDF

ILLUSTRATIVE CASE

A 63-year-old man with a medical history significant for myocardial infarction (MI) 5 years ago presents to you for an annual exam. His medications include a daily aspirin, ­angiotensin-converting enzyme inhibitor, beta-blocker, and a high-intensity statin for coronary artery disease (CAD). On his fasting lipid panel, his low-density lipoprotein (LDL) level is 70 mg/dL, but his triglycerides remain elevated at 200 mg/dL despite dietary changes.

In addition to lifestyle modifications, what can be done to reduce his risk of another MI?

Patients with known cardiovascular disease (CVD) or multiple risk factors for CVD are at high risk of cardiovascular events, even when taking primary or secondary preventive medications such as statins.2,3 In these patients, elevated triglycerides are an independent risk factor for increased rates of cardiovascular events.4,5

The 2018 American College of Cardiology/American Heart Association (ACC/AHA) guidelines for the treatment of blood cholesterol recommend statin therapy for moderate (175-499 mg/dL) to severe (≥ 500 mg/dL) hypertriglyceridemia in appropriate patients with atherosclerotic CVD risk ≥ 7.5%, after appropriately addressing secondary causes of hypertriglycidemia.6

Previous studies have shown no benefit from combination therapy with triglyceride-lowering medications (eg, extended-release niacin and fibrates) and statins, compared with statin monotherapy.7 A recent meta-analysis concluded that omega-3 fatty acid supplements offer no reduction in cardiovascular morbidity or mortality, whether taken with or without statins.8

Interestingly, the randomized controlled Japan EPA Lipid Intervention Study (JELIS) demonstrated fewer major coronary events in patients with elevated cholesterol, with or without CAD, who took eicosapentaenoic acid (EPA)—a subtype of omega-3 fatty acids—plus a statin, compared with statin monotherapy.9

The REDUCE-IT trial evaluated icosapent ethyl, a highly purified EPA that has been shown to reduce triglycerides and, at the time this study was conducted, was approved for use solely for the reduction of triglyceride levels in adults with severe hypertriglyceridemia.10,11

Continue to: Study Summary

 

 

STUDY SUMMARY

Patients with known CVD had fewercardiovascular events on icosapent ethyl

The multicenter, randomized controlled REDUCE-IT trial evaluated the effectiveness of icosapent ethyl, 2 g orally twice daily, on cardiovascular outcomes.1 A total of 8179 patients, ≥ 45 years of age with hypertriglyceridemia and known CVD or ≥ 50 years with diabetes and at least 1 additional risk factor and no known CVD, were enrolled at 473 participating sites in 11 countries, including the United States.

Patients had a triglyceride level of 150 to 499 mg/dL and an LDL cholesterol level of 41 to 100 mg/dL, and were taking a stable dose of a statin for at least 4 weeks. The enrollment protocol was amended to increase the lower limit of triglycerides from 150 to 200 mg/dL about one-third of the way through the study. Among the study population, 70.7% of patients were enrolled for secondary prevention (ie, had established CVD) and 29.3% of patients were enrolled for primary prevention (ie, had diabetes and at least 1 additional risk factor but no known CVD). Exclusion criteria included severe heart failure, active severe liver disease, glycated hemoglobin > 10%, a planned surgical cardiac intervention, history of pancreatitis, or allergies to fish or shellfish products.

Outcomes. The primary end point was a composite outcome of cardiovascular death, nonfatal MI, nonfatal stroke, coronary revascularization, or unstable angina.

Results. The median duration of follow-up was 4.9 years. From baseline to 1 year, the median change in triglycerides was an 18% reduction in the icosapent ethyl group but a 2% increase in the placebo group. Fewer patients in the icosapent ethyl group than the placebo group had a composite outcome event (17% vs 22%, respectively; hazard ratio [HR] = 0.75; 95% confidence interval [CI], 0.68-0.83; number needed to treat [NNT] to avoid 1 primary end point event = 21). Patients with known CVD had fewer composite outcome events in the icosapent ethyl group than the placebo group (19% vs 26%; HR = 0.73; 95% CI, 0.65-0.81; NNT = 14) but not in the primary prevention group vs the placebo group (12% vs 14%; HR = 0.88; 95% CI, 0.70-1.1).

In the entire population, all individual outcomes in the composite were significantly fewer in the icosapent ethyl group (cardiovascular death: HR = 0.8; 95% CI, 0.66-0.98; fatal or nonfatal MI: HR = 0.69; 95% CI, 0.58-0.81; revascularization: HR = 0.65; 95% CI, 0.55-0.78; unstable angina: HR = 0.68; 95% CI, 0.53-0.87; and fatal or nonfatal stroke: HR = 0.72; 95% CI, 0.55-0.93). All-cause mortality did not differ between groups (HR = 0.87; 95% CI, 0.74-1.02).

No significant differences in adverse events leading to discontinuation of the drug were reported between groups. Atrial fibrillation occurred more frequently in the icosapent ethyl group (5.3% vs 3.9%), but anemia (4.7% vs 5.8%) and gastrointestinal adverse events (33% vs 35%) were less common.

Continue to: What's New

 

 

WHAT’S NEW

First RCT to demonstrate valueof pairing icosapent ethyl with a statin

Many prior studies on use of omega-3 fatty acid supplements to treat hypertriglyceridemia did not show any benefit, possibly due to a low dose or low ratio of EPA in the study drug.8 One trial (JELIS) with favorable results was an open-label study, limited to patients in Japan. The REDUCE-IT study was the first randomized, placebo-controlled trial to show that icosapent ethyl treatment for hypertriglyceridemia in patients with known CVD who are taking a statin results in fewer cardiovascular events than statin use alone.

In this multinational trial of patients with hypertriglyceridemia and CVD or cardiovascular risk factors, icosapent ethyl significantly reduced the risk of cardiovascular events.

Also worth noting: Since publication of the REDUCE-IT study, the FDA has approved an expanded indication for icosapent ethyl for reduction of risk of cardiovascular events in statin-treated patients with hypertriglyceridemia and established CVD or diabetes and ≥ 2 additional cardiovascular risk factors.11

CAVEATS

Drug’s benefit was not linkedto triglyceride level reductions

The cardiovascular benefits of icosapent ethyl were obtained irrespective of triglyceride levels achieved. This raises the question of other potential mechanisms of action of icosapent ethyl in achieving cardiovascular benefit. However, this should not preclude the use of icosapent ethyl for secondary prevention in appropriate patients.

CHALLENGES TO IMPLEMENTATION

Medication is pricey

Icosapent ethyl is an expensive medication, currently priced at an estimated $351/month using a nationally available discount pharmacy plan, although additional manufacturer’s discounts may apply.12,13 The cost of the medication could be a consideration for widespread implementation of this recommendation.

ACKNOWLEDGEMENT

The PURLs Surveillance System was supported in part by Grant Number UL1RR024999 from the National Center For Research Resources, a Clinical Translational Science Award to the University of Chicago. The content is solely the responsibility of the authors and does not necessarily represent the official views of the National Center For Research Resources or the National Institutes of Health.

Copyright © 2020. The Family Physicians Inquiries Network. All rights reserved.

ILLUSTRATIVE CASE

A 63-year-old man with a medical history significant for myocardial infarction (MI) 5 years ago presents to you for an annual exam. His medications include a daily aspirin, ­angiotensin-converting enzyme inhibitor, beta-blocker, and a high-intensity statin for coronary artery disease (CAD). On his fasting lipid panel, his low-density lipoprotein (LDL) level is 70 mg/dL, but his triglycerides remain elevated at 200 mg/dL despite dietary changes.

In addition to lifestyle modifications, what can be done to reduce his risk of another MI?

Patients with known cardiovascular disease (CVD) or multiple risk factors for CVD are at high risk of cardiovascular events, even when taking primary or secondary preventive medications such as statins.2,3 In these patients, elevated triglycerides are an independent risk factor for increased rates of cardiovascular events.4,5

The 2018 American College of Cardiology/American Heart Association (ACC/AHA) guidelines for the treatment of blood cholesterol recommend statin therapy for moderate (175-499 mg/dL) to severe (≥ 500 mg/dL) hypertriglyceridemia in appropriate patients with atherosclerotic CVD risk ≥ 7.5%, after appropriately addressing secondary causes of hypertriglycidemia.6

Previous studies have shown no benefit from combination therapy with triglyceride-lowering medications (eg, extended-release niacin and fibrates) and statins, compared with statin monotherapy.7 A recent meta-analysis concluded that omega-3 fatty acid supplements offer no reduction in cardiovascular morbidity or mortality, whether taken with or without statins.8

Interestingly, the randomized controlled Japan EPA Lipid Intervention Study (JELIS) demonstrated fewer major coronary events in patients with elevated cholesterol, with or without CAD, who took eicosapentaenoic acid (EPA)—a subtype of omega-3 fatty acids—plus a statin, compared with statin monotherapy.9

The REDUCE-IT trial evaluated icosapent ethyl, a highly purified EPA that has been shown to reduce triglycerides and, at the time this study was conducted, was approved for use solely for the reduction of triglyceride levels in adults with severe hypertriglyceridemia.10,11

Continue to: Study Summary

 

 

STUDY SUMMARY

Patients with known CVD had fewercardiovascular events on icosapent ethyl

The multicenter, randomized controlled REDUCE-IT trial evaluated the effectiveness of icosapent ethyl, 2 g orally twice daily, on cardiovascular outcomes.1 A total of 8179 patients, ≥ 45 years of age with hypertriglyceridemia and known CVD or ≥ 50 years with diabetes and at least 1 additional risk factor and no known CVD, were enrolled at 473 participating sites in 11 countries, including the United States.

Patients had a triglyceride level of 150 to 499 mg/dL and an LDL cholesterol level of 41 to 100 mg/dL, and were taking a stable dose of a statin for at least 4 weeks. The enrollment protocol was amended to increase the lower limit of triglycerides from 150 to 200 mg/dL about one-third of the way through the study. Among the study population, 70.7% of patients were enrolled for secondary prevention (ie, had established CVD) and 29.3% of patients were enrolled for primary prevention (ie, had diabetes and at least 1 additional risk factor but no known CVD). Exclusion criteria included severe heart failure, active severe liver disease, glycated hemoglobin > 10%, a planned surgical cardiac intervention, history of pancreatitis, or allergies to fish or shellfish products.

Outcomes. The primary end point was a composite outcome of cardiovascular death, nonfatal MI, nonfatal stroke, coronary revascularization, or unstable angina.

Results. The median duration of follow-up was 4.9 years. From baseline to 1 year, the median change in triglycerides was an 18% reduction in the icosapent ethyl group but a 2% increase in the placebo group. Fewer patients in the icosapent ethyl group than the placebo group had a composite outcome event (17% vs 22%, respectively; hazard ratio [HR] = 0.75; 95% confidence interval [CI], 0.68-0.83; number needed to treat [NNT] to avoid 1 primary end point event = 21). Patients with known CVD had fewer composite outcome events in the icosapent ethyl group than the placebo group (19% vs 26%; HR = 0.73; 95% CI, 0.65-0.81; NNT = 14) but not in the primary prevention group vs the placebo group (12% vs 14%; HR = 0.88; 95% CI, 0.70-1.1).

In the entire population, all individual outcomes in the composite were significantly fewer in the icosapent ethyl group (cardiovascular death: HR = 0.8; 95% CI, 0.66-0.98; fatal or nonfatal MI: HR = 0.69; 95% CI, 0.58-0.81; revascularization: HR = 0.65; 95% CI, 0.55-0.78; unstable angina: HR = 0.68; 95% CI, 0.53-0.87; and fatal or nonfatal stroke: HR = 0.72; 95% CI, 0.55-0.93). All-cause mortality did not differ between groups (HR = 0.87; 95% CI, 0.74-1.02).

No significant differences in adverse events leading to discontinuation of the drug were reported between groups. Atrial fibrillation occurred more frequently in the icosapent ethyl group (5.3% vs 3.9%), but anemia (4.7% vs 5.8%) and gastrointestinal adverse events (33% vs 35%) were less common.

Continue to: What's New

 

 

WHAT’S NEW

First RCT to demonstrate valueof pairing icosapent ethyl with a statin

Many prior studies on use of omega-3 fatty acid supplements to treat hypertriglyceridemia did not show any benefit, possibly due to a low dose or low ratio of EPA in the study drug.8 One trial (JELIS) with favorable results was an open-label study, limited to patients in Japan. The REDUCE-IT study was the first randomized, placebo-controlled trial to show that icosapent ethyl treatment for hypertriglyceridemia in patients with known CVD who are taking a statin results in fewer cardiovascular events than statin use alone.

In this multinational trial of patients with hypertriglyceridemia and CVD or cardiovascular risk factors, icosapent ethyl significantly reduced the risk of cardiovascular events.

Also worth noting: Since publication of the REDUCE-IT study, the FDA has approved an expanded indication for icosapent ethyl for reduction of risk of cardiovascular events in statin-treated patients with hypertriglyceridemia and established CVD or diabetes and ≥ 2 additional cardiovascular risk factors.11

CAVEATS

Drug’s benefit was not linkedto triglyceride level reductions

The cardiovascular benefits of icosapent ethyl were obtained irrespective of triglyceride levels achieved. This raises the question of other potential mechanisms of action of icosapent ethyl in achieving cardiovascular benefit. However, this should not preclude the use of icosapent ethyl for secondary prevention in appropriate patients.

CHALLENGES TO IMPLEMENTATION

Medication is pricey

Icosapent ethyl is an expensive medication, currently priced at an estimated $351/month using a nationally available discount pharmacy plan, although additional manufacturer’s discounts may apply.12,13 The cost of the medication could be a consideration for widespread implementation of this recommendation.

ACKNOWLEDGEMENT

The PURLs Surveillance System was supported in part by Grant Number UL1RR024999 from the National Center For Research Resources, a Clinical Translational Science Award to the University of Chicago. The content is solely the responsibility of the authors and does not necessarily represent the official views of the National Center For Research Resources or the National Institutes of Health.

Copyright © 2020. The Family Physicians Inquiries Network. All rights reserved.

References

1. Bhatt DL, Steg PG, Miller M, et al; REDUCE-IT Investigators. Cardiovascular risk reduction with icosapent ethyl for hypertriglyceridemia. N Engl J Med. 2019;380:11-22.

2. Bhatt DL, Eagle KA, Ohman EM, et al; REACH Registry Investigators. Comparative determinants of 4-year cardiovascular event rates in stable outpatients at risk of or with atherothrombosis. JAMA. 2010;304:1350-1357.

3. Cannon CP, Braunwald E, McCabe CH, et al; Pravastatin or Atorvastatin Evaluation and Infection Therapy–Thrombolysis in Myocardial Infarction 22 Investigators. Intensive versus moderate lipid lowering with statins after acute coronary syndromes [published correction appears in N Engl J Med. 2006;354:778]. N Engl J Med. 2004;350:1495-1504.

4. Klempfner R, Erez A, Sagit BZ, et al. Elevated triglyceride level is independently associated with increased all-cause mortality in patients with established coronary heart disease: twenty-two-year follow-up of the Bezafibrate Infarction Prevention Study and Registry [published correction appears in Circ Cardiovasc Qual Outcomes. 2016;9:613]. Circ Cardiovasc Qual Outcomes. 2016;9:100-108.

5. Nichols GA, Philip S, Reynolds K, Granowitz CB, Fazio S. Increased cardiovascular risk in hypertriglyceridemic patients with statin-controlled LDL cholesterol. J Clin Endocrinol Metab. 2018;103:3019-3027.

6. Grundy SM, Stone NJ, Bailey AL, et al. 2018 AHA/ACC/AACVPR/AAPA/ABC/ACPM/ADA/AGS/APhA/ASPC/NLA/PCNA guideline on the management of blood cholesterol: a report of the American College of Cardiology/American Heart Association Task Force on Clinical Practice Guidelines [published correction appears in J Am Coll Cardiol. 2019;73:3237-3241]. J Am Coll Cardiol. 2019;73:e285-e350.

7. Ganda OP, Bhatt DL, Mason RP, Miller M, Boden WE. Unmet need for adjunctive dyslipidemia therapy in hypertriglyceridemia management. J Am Coll Cardiol. 2018;72:330-343.

8. Aung T, Halsey J, Kromhout D, et al; Omega-3 Treatment Trialists’ Collaboration. Associations of omega-3 fatty acid supplement use with cardiovascular disease risks: meta-analysis of 10 trials involving 77 917 individuals. JAMA Cardiol. 2018;3:225-234.

9. Yokoyama M, Origasa H, Matsuzaki M, et al; Japan EPA lipid intervention study (JELIS) Investigators. Effects of eicosapentaenoic acid on major coronary events in hypercholesterolaemic patients (JELIS): a randomised open-label, blinded endpoint analysis [published correction appears in Lancet. 2007;370:220]. Lancet. 2007;369:1090-1098.

10. Ballantyne CM, Bays HE, Kastelein JJ, et al. Efficacy and safety of eicosapentaenoic acid ethyl ester (AMR101) therapy in statin-treated patients with persistent high triglycerides (from the ANCHOR study). Am J Cardiol. 2012;110:984-992.

11. FDA approves use of drug to reduce risk of cardiovascular events in certain adult patient groups [news release]. Silver Spring, MD: US Food and Drug Administration; December 13, 2019. www.fda.gov/news-events/press-announcements/fda-approves-use-drug-reduce-risk-cardiovascular-events-certain-adult-patient-groups. Accessed November 30, 2020.

12. Vascepa. GoodRx. www.goodrx.com/vascepa. Accessed November 30, 2020.

13. The VASCEPA Savings Program. www.vascepa.com/getting-started/savings-card/. Accessed November 30, 2020.

References

1. Bhatt DL, Steg PG, Miller M, et al; REDUCE-IT Investigators. Cardiovascular risk reduction with icosapent ethyl for hypertriglyceridemia. N Engl J Med. 2019;380:11-22.

2. Bhatt DL, Eagle KA, Ohman EM, et al; REACH Registry Investigators. Comparative determinants of 4-year cardiovascular event rates in stable outpatients at risk of or with atherothrombosis. JAMA. 2010;304:1350-1357.

3. Cannon CP, Braunwald E, McCabe CH, et al; Pravastatin or Atorvastatin Evaluation and Infection Therapy–Thrombolysis in Myocardial Infarction 22 Investigators. Intensive versus moderate lipid lowering with statins after acute coronary syndromes [published correction appears in N Engl J Med. 2006;354:778]. N Engl J Med. 2004;350:1495-1504.

4. Klempfner R, Erez A, Sagit BZ, et al. Elevated triglyceride level is independently associated with increased all-cause mortality in patients with established coronary heart disease: twenty-two-year follow-up of the Bezafibrate Infarction Prevention Study and Registry [published correction appears in Circ Cardiovasc Qual Outcomes. 2016;9:613]. Circ Cardiovasc Qual Outcomes. 2016;9:100-108.

5. Nichols GA, Philip S, Reynolds K, Granowitz CB, Fazio S. Increased cardiovascular risk in hypertriglyceridemic patients with statin-controlled LDL cholesterol. J Clin Endocrinol Metab. 2018;103:3019-3027.

6. Grundy SM, Stone NJ, Bailey AL, et al. 2018 AHA/ACC/AACVPR/AAPA/ABC/ACPM/ADA/AGS/APhA/ASPC/NLA/PCNA guideline on the management of blood cholesterol: a report of the American College of Cardiology/American Heart Association Task Force on Clinical Practice Guidelines [published correction appears in J Am Coll Cardiol. 2019;73:3237-3241]. J Am Coll Cardiol. 2019;73:e285-e350.

7. Ganda OP, Bhatt DL, Mason RP, Miller M, Boden WE. Unmet need for adjunctive dyslipidemia therapy in hypertriglyceridemia management. J Am Coll Cardiol. 2018;72:330-343.

8. Aung T, Halsey J, Kromhout D, et al; Omega-3 Treatment Trialists’ Collaboration. Associations of omega-3 fatty acid supplement use with cardiovascular disease risks: meta-analysis of 10 trials involving 77 917 individuals. JAMA Cardiol. 2018;3:225-234.

9. Yokoyama M, Origasa H, Matsuzaki M, et al; Japan EPA lipid intervention study (JELIS) Investigators. Effects of eicosapentaenoic acid on major coronary events in hypercholesterolaemic patients (JELIS): a randomised open-label, blinded endpoint analysis [published correction appears in Lancet. 2007;370:220]. Lancet. 2007;369:1090-1098.

10. Ballantyne CM, Bays HE, Kastelein JJ, et al. Efficacy and safety of eicosapentaenoic acid ethyl ester (AMR101) therapy in statin-treated patients with persistent high triglycerides (from the ANCHOR study). Am J Cardiol. 2012;110:984-992.

11. FDA approves use of drug to reduce risk of cardiovascular events in certain adult patient groups [news release]. Silver Spring, MD: US Food and Drug Administration; December 13, 2019. www.fda.gov/news-events/press-announcements/fda-approves-use-drug-reduce-risk-cardiovascular-events-certain-adult-patient-groups. Accessed November 30, 2020.

12. Vascepa. GoodRx. www.goodrx.com/vascepa. Accessed November 30, 2020.

13. The VASCEPA Savings Program. www.vascepa.com/getting-started/savings-card/. Accessed November 30, 2020.

Issue
The Journal of Family Practice - 69(10)
Issue
The Journal of Family Practice - 69(10)
Page Number
518-525
Page Number
518-525
Publications
Publications
Topics
Article Type
Sections
Inside the Article

PRACTICE CHANGER

Consider icosapent ethyl, 2 g twice daily, for secondary prevention of adverse cardiovascular events in patients with elevated triglycerides who are already taking a statin.


STRENGTH OF RECOMMENDATION

B: Based on a single, good-quality, multicenter, randomized controlled trial. Bhatt DL, Steg PG, Miller M, et al; REDUCE-IT Investigators. Cardiovascular risk reduction with icosapent ethyl for hypertriglyceridemia. N Engl J Med. 2019;380:11-22.1

 

Bhatt DL, Steg PG, Miller M, et al; REDUCE-IT Investigators. Cardiovascular risk reduction with icosapent ethyl for hypertriglyceridemia. N Engl J Med. 2019;380:11-22.1

Disallow All Ads
Content Gating
No Gating (article Unlocked/Free)
Alternative CME
Disqus Comments
Default
Use ProPublica
Hide sidebar & use full width
render the right sidebar.
Conference Recap Checkbox
Not Conference Recap
Clinical Edge
Display the Slideshow in this Article
Medscape Article
Display survey writer
Reuters content
Disable Inline Native ads
Article PDF Media
Image
Teambase ID
18001A54.SIG
Disable zoom
Off
Media Files

Do-it-yourself cervical cancer screening?

Article Type
Changed
Wed, 10/14/2020 - 11:38
Display Headline
Do-it-yourself cervical cancer screening?

ILLUSTRATIVE CASE

A 40-year-old woman presents to your office to establish care. During your interview you realize that she has never been screened for cervical cancer. In fact, she has not had a pelvic exam because she is fearful of the procedure. She would like to know if alternatives exist for cervical cancer screening. What can you suggest?

Although deaths from cervical cancer decreased in the United States from 1975 to 2017, demographic and social disparities in the burden of the disease remain.2,3 Data from 2016 reveal that cervical cancer incidence per 100,000 women is lowest among white (7.5), Asian-Pacific Islander (5.8), and American Indian/Alaska native (5.6) women, and highest among Hispanic (9.8) and black (8.7) women, which could be explained by lower screening rates in these populations.4,5 The National Cancer Institute’s publication on reducing cancer health disparities states that the most effective way to reduce cervical cancer incidence and mortality is by increasing screening rates among women who have not been screened or who have not been screened regularly.6

The US Food and Drug Administration (FDA) approved the first human papillomavirus (HPV) screening test in 2003.7 Evidence now suggests that high-risk HPV screening provides greater protection against cervical cancer than screening with cytology alone.8 The American College of Obstetricians and Gynecologists (ACOG) and the US Preventive Services Task Force (USPSTF) have changed their recommendations to include primary HPV testing as an alternative method to Pap smears for cervical cancer screening.9

An advantage of primary HPV screening is that it can be performed on a specimen collected by the patient, which could potentially increase rates of screening and help to decrease demographic and social disparities. A randomized trial of almost 2000 women ages 21 to 65 years that evaluated the acceptability of this method to patients revealed that more than half of women prefer the idea of a self-collected specimen to one that is collected by a clinician because it is more convenient and obviates the need for a pelvic exam.10

A meta-analysis of 36 studies and more than 150,000 women concluded that when self-collected samples were used with signal-based assays, the tests were not as sensitive or specific as when clinician-collected samples were used.11 However, the meta-analysis also found that some polymerase chain reaction (PCR)-based HPV tests were similarly sensitive for both self- and clinician-collected samples.

STUDY SUMMARY

PCR vs signal amplification HPV tests with collection by patients vs clinicians

This meta-analysis compared the accuracy of high-risk HPV self-screening with clinician collection of samples (56 diagnostic accuracy trials; total N not provided) in identifying cervical intraepithelial neoplasia grade 2 or worse (CIN 2+) with signal amplification and PCR tests evaluated separately.1 In addition, this review evaluated strategies to screen women who are underscreened or not screened, which was defined as women who were irregularly or never screened, or did not respond to reminder letters about cervical cancer screening (25 randomized controlled trials [RCTs]; total N not provided).

In the diagnostic accuracy studies, patients collected a vaginal sample themselves and then had a sample taken by a clinician. CIN 2+ or 3+ was confirmed by either colposcopy and biopsy performed on all patients or by a positive high-risk HPV test result. Studies were further divided into those using assays based on signal amplification or PCR.

Continue to: In signal amplification assays...

 

 

In signal amplification assays, the pooled sensitivity for CIN 2+ was lower in the group with the self-collected samples than in the clinician-collected sample group (77%; 95% confidence interval [CI], 69%-82% vs 93%; 95% CI, 89%-96%). The pooled specificity to exclude CIN 2+ was also lower in the group with the self-collected samples (84%; 95% CI, 77%-88% vs 86%; 95% CI, 81%-90%). In high-risk HPV assays based on PCR, there was no difference in sensitivity (96%) or specificity (79%) between the specimen groups.

This study offers robust evidence that high-risk HPV PCR-based assays using patient-collected specimens are as sensitive at diagnosing CIN 2+ or 3+ as using clinician-collected samples.

With regard to the pooled relative sensitivity and specificity of signal amplification assays, those using self-swab samples were less sensitive and less specific for CIN 2+ (sensitivity ratio = 0.85; 95% CI, 0.80-0.89; specificity ratio = 0.96; 95% CI, 0.93-0.98) and CIN 3+ (sensitivity ratio = 0.86; 95% CI, 0.76-0.98; specificity ratio = 0.97; 95% CI, 0.95-0.99). Using PCR assays, there was no difference between groups in relative sensitivity for the diagnosis of CIN 2+ (sensitivity ratio = 0.99; 95% CI, 0.97-1.02) and CIN 3+ (sensitivity ratio = 0.99; 95% CI, 0.96-1.02). Relative specificity was slightly lower in the self-swab group for CIN 2+ (specificity ratio = 0.98; 95% CI, 0.97-0.99) and CIN 3+ (specificity ratio = 0.98; 95% CI, 0.97-0.99).

The second analysis to evaluate which outreach strategies are effective methods for screening underscreened/unscreened women found that delivering self-sample kits to patients was more effective than the control method, which was sending reminders to women to undergo conventional screening (95% vs 53%; mean difference [MD], 41%; 95% CI, 3%-78%). Similarly, mailing kits to patients compared favorably to the control method (25% vs 12%; MD, 13%; 95% CI, 10%-15%).

WHAT’S NEW

Self-collected specimens can beas reliable as clinician-collected ones

This is the first study to provide robust evidence that high-risk HPV PCR-based assays using patient self-collected specimens are as sensitive at diagnosing CIN 2+ or 3+ as using clinician-collected samples.

CAVEATS

Balancing lower specificity with reaching underscreened populations

Patients with a positive HPV test result require additional testing. The success rates for this follow-up are not known and could be a barrier to accurate diagnoses because of accessibility and patient willingness to follow up with a pelvic exam. In addition, self-collection may be less specific than cytology and could increase colposcopy referrals that lead to negative findings and overtreatment.12 However, the increased acceptance of this screening method could make it an effective strategy to reach underscreened or reluctant patients.

Continue to: CHALLENGES TO IMPLEMENTATION

 

 

CHALLENGES TO IMPLEMENTATION

Availability of PCR-based HPV assays may be an issue

HPV PCR assays may not be available at all laboratories, but signal amplification HPV tests have been shown to be inferior to PCR assays. Physicians will have to confirm with their laboratories whether PCR-based HPV assays are available.

ACKNOWLEDGEMENT

The PURLs Surveillance System was supported in part by Grant Number UL1RR024999 from the National Center For Research Resources, a Clinical Translational Science Award to the University of Chicago. The content is solely the responsibility of the authors and does not necessarily represent the official views of the National Center For Research Resources or the National Institutes of Health.

Files
References

1. Arbyn M, Smith SB, Temin S, et al; Collaboration on Self-Sampling and HPV Testing. Detecting cervical precancer and reaching underscreened women by using HPV testing on self-samples: updated meta-analyses. BMJ. 2018;363:k4823.

2. National Cancer Institute Surveillance, Epidemiology, and End Results Program. Cancer stat facts: cervical cancer. www.seer.cancer.gov/statfacts/html/cervix.html. Accessed June 29, 2020.

3. Singh GK, Azuine RE, Siahpush M. Global inequalities in cervical cancer incidence and mortality are linked to deprivation, low socioeconomic status, and human development. Int J MCH AIDS. 2012;1:17‐30.

4. US Cancer Statistics Working Group. US Cancer Statistics Data Visualizations Tool, based on November 2018 submission data (1999-2016): US Department of Health and Human Services, Centers for Disease Control and Prevention and National Cancer Institute. June 2019. www.cdc.gov/cancer/dataviz. Accessed June 29, 2020.

5. MacLaughlin KL, Jacobson RM, Breitkopf CR, et al. Trends over time in Pap and Pap-HPV cotesting for cervical cancer screening. J Womens Health. 2019;28:244-249.

6. Freeman HP, Wingrove BK. Excess Cervical Cancer Mortality: A Marker for Low Access to Health Care in Poor Communities. NIH Pub. No. 05–5282. Rockville, MD: National Cancer Institute, Center to Reduce Cancer Health Disparities, May 2005. www.cancer.gov/about-nci/organization/crchd/about-health-disparities/resources/excess-cervical-cancer-mortality.pdf. Accessed June 29, 2020.

7. FDA approves expanded use of HPV test. Infection Control Today. March 31, 2003. https://www.infectioncontroltoday.com/view/fda-approves-expanded-use-hpv-test. Accessed June 29, 2020.

8. Ronco G, Dillner J, Elfström K, et al. Efficacy of HPV-based screening for prevention of invasive cervical cancer: follow-up of four European randomised controlled trials. Lancet. 2014;383:524-532.

9. CDC. Cervical cancer screening guidelines for average-risk women. www.cdc.gov/cancer/cervical/pdf/guidelines.pdf. Accessed June 29, 2020.

10. Mao C, Kulasingam S, Whitham H, et al. Clinician and patient acceptability of self-collected human papillomavirus testing for cervical cancer screening. J Womens Health. 2017;26:609-615.

11. Arbyn M, Verdoodt F, Snijders PJ, et al. Accuracy of human papillomavirus testing on self-collected versus clinician-collected samples: a meta-analysis. Lancet Oncol. 2014;15:172-183.

12. Lazcano-Ponce E, Lorincz A, Cruz-Valdez A, et al. Self-collection of vaginal specimens for human papillomavirus testing in cervical cancer prevention (MARCH): a community-based randomised controlled trial. Lancet. 2011;378:1868-1873.

Article PDF
Author and Disclosure Information

University of Colorado Family Medicine Residency, Denver

DEPUTY EDITOR
Shailendra Prasad, MBBS, MPH

University of Minnesota North Memorial Family Medicine Residency Program, Minneapolis

Issue
The Journal of Family Practice - 69(6)
Publications
Topics
Page Number
306-308
Sections
Files
Files
Author and Disclosure Information

University of Colorado Family Medicine Residency, Denver

DEPUTY EDITOR
Shailendra Prasad, MBBS, MPH

University of Minnesota North Memorial Family Medicine Residency Program, Minneapolis

Author and Disclosure Information

University of Colorado Family Medicine Residency, Denver

DEPUTY EDITOR
Shailendra Prasad, MBBS, MPH

University of Minnesota North Memorial Family Medicine Residency Program, Minneapolis

Article PDF
Article PDF

ILLUSTRATIVE CASE

A 40-year-old woman presents to your office to establish care. During your interview you realize that she has never been screened for cervical cancer. In fact, she has not had a pelvic exam because she is fearful of the procedure. She would like to know if alternatives exist for cervical cancer screening. What can you suggest?

Although deaths from cervical cancer decreased in the United States from 1975 to 2017, demographic and social disparities in the burden of the disease remain.2,3 Data from 2016 reveal that cervical cancer incidence per 100,000 women is lowest among white (7.5), Asian-Pacific Islander (5.8), and American Indian/Alaska native (5.6) women, and highest among Hispanic (9.8) and black (8.7) women, which could be explained by lower screening rates in these populations.4,5 The National Cancer Institute’s publication on reducing cancer health disparities states that the most effective way to reduce cervical cancer incidence and mortality is by increasing screening rates among women who have not been screened or who have not been screened regularly.6

The US Food and Drug Administration (FDA) approved the first human papillomavirus (HPV) screening test in 2003.7 Evidence now suggests that high-risk HPV screening provides greater protection against cervical cancer than screening with cytology alone.8 The American College of Obstetricians and Gynecologists (ACOG) and the US Preventive Services Task Force (USPSTF) have changed their recommendations to include primary HPV testing as an alternative method to Pap smears for cervical cancer screening.9

An advantage of primary HPV screening is that it can be performed on a specimen collected by the patient, which could potentially increase rates of screening and help to decrease demographic and social disparities. A randomized trial of almost 2000 women ages 21 to 65 years that evaluated the acceptability of this method to patients revealed that more than half of women prefer the idea of a self-collected specimen to one that is collected by a clinician because it is more convenient and obviates the need for a pelvic exam.10

A meta-analysis of 36 studies and more than 150,000 women concluded that when self-collected samples were used with signal-based assays, the tests were not as sensitive or specific as when clinician-collected samples were used.11 However, the meta-analysis also found that some polymerase chain reaction (PCR)-based HPV tests were similarly sensitive for both self- and clinician-collected samples.

STUDY SUMMARY

PCR vs signal amplification HPV tests with collection by patients vs clinicians

This meta-analysis compared the accuracy of high-risk HPV self-screening with clinician collection of samples (56 diagnostic accuracy trials; total N not provided) in identifying cervical intraepithelial neoplasia grade 2 or worse (CIN 2+) with signal amplification and PCR tests evaluated separately.1 In addition, this review evaluated strategies to screen women who are underscreened or not screened, which was defined as women who were irregularly or never screened, or did not respond to reminder letters about cervical cancer screening (25 randomized controlled trials [RCTs]; total N not provided).

In the diagnostic accuracy studies, patients collected a vaginal sample themselves and then had a sample taken by a clinician. CIN 2+ or 3+ was confirmed by either colposcopy and biopsy performed on all patients or by a positive high-risk HPV test result. Studies were further divided into those using assays based on signal amplification or PCR.

Continue to: In signal amplification assays...

 

 

In signal amplification assays, the pooled sensitivity for CIN 2+ was lower in the group with the self-collected samples than in the clinician-collected sample group (77%; 95% confidence interval [CI], 69%-82% vs 93%; 95% CI, 89%-96%). The pooled specificity to exclude CIN 2+ was also lower in the group with the self-collected samples (84%; 95% CI, 77%-88% vs 86%; 95% CI, 81%-90%). In high-risk HPV assays based on PCR, there was no difference in sensitivity (96%) or specificity (79%) between the specimen groups.

This study offers robust evidence that high-risk HPV PCR-based assays using patient-collected specimens are as sensitive at diagnosing CIN 2+ or 3+ as using clinician-collected samples.

With regard to the pooled relative sensitivity and specificity of signal amplification assays, those using self-swab samples were less sensitive and less specific for CIN 2+ (sensitivity ratio = 0.85; 95% CI, 0.80-0.89; specificity ratio = 0.96; 95% CI, 0.93-0.98) and CIN 3+ (sensitivity ratio = 0.86; 95% CI, 0.76-0.98; specificity ratio = 0.97; 95% CI, 0.95-0.99). Using PCR assays, there was no difference between groups in relative sensitivity for the diagnosis of CIN 2+ (sensitivity ratio = 0.99; 95% CI, 0.97-1.02) and CIN 3+ (sensitivity ratio = 0.99; 95% CI, 0.96-1.02). Relative specificity was slightly lower in the self-swab group for CIN 2+ (specificity ratio = 0.98; 95% CI, 0.97-0.99) and CIN 3+ (specificity ratio = 0.98; 95% CI, 0.97-0.99).

The second analysis to evaluate which outreach strategies are effective methods for screening underscreened/unscreened women found that delivering self-sample kits to patients was more effective than the control method, which was sending reminders to women to undergo conventional screening (95% vs 53%; mean difference [MD], 41%; 95% CI, 3%-78%). Similarly, mailing kits to patients compared favorably to the control method (25% vs 12%; MD, 13%; 95% CI, 10%-15%).

WHAT’S NEW

Self-collected specimens can beas reliable as clinician-collected ones

This is the first study to provide robust evidence that high-risk HPV PCR-based assays using patient self-collected specimens are as sensitive at diagnosing CIN 2+ or 3+ as using clinician-collected samples.

CAVEATS

Balancing lower specificity with reaching underscreened populations

Patients with a positive HPV test result require additional testing. The success rates for this follow-up are not known and could be a barrier to accurate diagnoses because of accessibility and patient willingness to follow up with a pelvic exam. In addition, self-collection may be less specific than cytology and could increase colposcopy referrals that lead to negative findings and overtreatment.12 However, the increased acceptance of this screening method could make it an effective strategy to reach underscreened or reluctant patients.

Continue to: CHALLENGES TO IMPLEMENTATION

 

 

CHALLENGES TO IMPLEMENTATION

Availability of PCR-based HPV assays may be an issue

HPV PCR assays may not be available at all laboratories, but signal amplification HPV tests have been shown to be inferior to PCR assays. Physicians will have to confirm with their laboratories whether PCR-based HPV assays are available.

ACKNOWLEDGEMENT

The PURLs Surveillance System was supported in part by Grant Number UL1RR024999 from the National Center For Research Resources, a Clinical Translational Science Award to the University of Chicago. The content is solely the responsibility of the authors and does not necessarily represent the official views of the National Center For Research Resources or the National Institutes of Health.

ILLUSTRATIVE CASE

A 40-year-old woman presents to your office to establish care. During your interview you realize that she has never been screened for cervical cancer. In fact, she has not had a pelvic exam because she is fearful of the procedure. She would like to know if alternatives exist for cervical cancer screening. What can you suggest?

Although deaths from cervical cancer decreased in the United States from 1975 to 2017, demographic and social disparities in the burden of the disease remain.2,3 Data from 2016 reveal that cervical cancer incidence per 100,000 women is lowest among white (7.5), Asian-Pacific Islander (5.8), and American Indian/Alaska native (5.6) women, and highest among Hispanic (9.8) and black (8.7) women, which could be explained by lower screening rates in these populations.4,5 The National Cancer Institute’s publication on reducing cancer health disparities states that the most effective way to reduce cervical cancer incidence and mortality is by increasing screening rates among women who have not been screened or who have not been screened regularly.6

The US Food and Drug Administration (FDA) approved the first human papillomavirus (HPV) screening test in 2003.7 Evidence now suggests that high-risk HPV screening provides greater protection against cervical cancer than screening with cytology alone.8 The American College of Obstetricians and Gynecologists (ACOG) and the US Preventive Services Task Force (USPSTF) have changed their recommendations to include primary HPV testing as an alternative method to Pap smears for cervical cancer screening.9

An advantage of primary HPV screening is that it can be performed on a specimen collected by the patient, which could potentially increase rates of screening and help to decrease demographic and social disparities. A randomized trial of almost 2000 women ages 21 to 65 years that evaluated the acceptability of this method to patients revealed that more than half of women prefer the idea of a self-collected specimen to one that is collected by a clinician because it is more convenient and obviates the need for a pelvic exam.10

A meta-analysis of 36 studies and more than 150,000 women concluded that when self-collected samples were used with signal-based assays, the tests were not as sensitive or specific as when clinician-collected samples were used.11 However, the meta-analysis also found that some polymerase chain reaction (PCR)-based HPV tests were similarly sensitive for both self- and clinician-collected samples.

STUDY SUMMARY

PCR vs signal amplification HPV tests with collection by patients vs clinicians

This meta-analysis compared the accuracy of high-risk HPV self-screening with clinician collection of samples (56 diagnostic accuracy trials; total N not provided) in identifying cervical intraepithelial neoplasia grade 2 or worse (CIN 2+) with signal amplification and PCR tests evaluated separately.1 In addition, this review evaluated strategies to screen women who are underscreened or not screened, which was defined as women who were irregularly or never screened, or did not respond to reminder letters about cervical cancer screening (25 randomized controlled trials [RCTs]; total N not provided).

In the diagnostic accuracy studies, patients collected a vaginal sample themselves and then had a sample taken by a clinician. CIN 2+ or 3+ was confirmed by either colposcopy and biopsy performed on all patients or by a positive high-risk HPV test result. Studies were further divided into those using assays based on signal amplification or PCR.

Continue to: In signal amplification assays...

 

 

In signal amplification assays, the pooled sensitivity for CIN 2+ was lower in the group with the self-collected samples than in the clinician-collected sample group (77%; 95% confidence interval [CI], 69%-82% vs 93%; 95% CI, 89%-96%). The pooled specificity to exclude CIN 2+ was also lower in the group with the self-collected samples (84%; 95% CI, 77%-88% vs 86%; 95% CI, 81%-90%). In high-risk HPV assays based on PCR, there was no difference in sensitivity (96%) or specificity (79%) between the specimen groups.

This study offers robust evidence that high-risk HPV PCR-based assays using patient-collected specimens are as sensitive at diagnosing CIN 2+ or 3+ as using clinician-collected samples.

With regard to the pooled relative sensitivity and specificity of signal amplification assays, those using self-swab samples were less sensitive and less specific for CIN 2+ (sensitivity ratio = 0.85; 95% CI, 0.80-0.89; specificity ratio = 0.96; 95% CI, 0.93-0.98) and CIN 3+ (sensitivity ratio = 0.86; 95% CI, 0.76-0.98; specificity ratio = 0.97; 95% CI, 0.95-0.99). Using PCR assays, there was no difference between groups in relative sensitivity for the diagnosis of CIN 2+ (sensitivity ratio = 0.99; 95% CI, 0.97-1.02) and CIN 3+ (sensitivity ratio = 0.99; 95% CI, 0.96-1.02). Relative specificity was slightly lower in the self-swab group for CIN 2+ (specificity ratio = 0.98; 95% CI, 0.97-0.99) and CIN 3+ (specificity ratio = 0.98; 95% CI, 0.97-0.99).

The second analysis to evaluate which outreach strategies are effective methods for screening underscreened/unscreened women found that delivering self-sample kits to patients was more effective than the control method, which was sending reminders to women to undergo conventional screening (95% vs 53%; mean difference [MD], 41%; 95% CI, 3%-78%). Similarly, mailing kits to patients compared favorably to the control method (25% vs 12%; MD, 13%; 95% CI, 10%-15%).

WHAT’S NEW

Self-collected specimens can beas reliable as clinician-collected ones

This is the first study to provide robust evidence that high-risk HPV PCR-based assays using patient self-collected specimens are as sensitive at diagnosing CIN 2+ or 3+ as using clinician-collected samples.

CAVEATS

Balancing lower specificity with reaching underscreened populations

Patients with a positive HPV test result require additional testing. The success rates for this follow-up are not known and could be a barrier to accurate diagnoses because of accessibility and patient willingness to follow up with a pelvic exam. In addition, self-collection may be less specific than cytology and could increase colposcopy referrals that lead to negative findings and overtreatment.12 However, the increased acceptance of this screening method could make it an effective strategy to reach underscreened or reluctant patients.

Continue to: CHALLENGES TO IMPLEMENTATION

 

 

CHALLENGES TO IMPLEMENTATION

Availability of PCR-based HPV assays may be an issue

HPV PCR assays may not be available at all laboratories, but signal amplification HPV tests have been shown to be inferior to PCR assays. Physicians will have to confirm with their laboratories whether PCR-based HPV assays are available.

ACKNOWLEDGEMENT

The PURLs Surveillance System was supported in part by Grant Number UL1RR024999 from the National Center For Research Resources, a Clinical Translational Science Award to the University of Chicago. The content is solely the responsibility of the authors and does not necessarily represent the official views of the National Center For Research Resources or the National Institutes of Health.

References

1. Arbyn M, Smith SB, Temin S, et al; Collaboration on Self-Sampling and HPV Testing. Detecting cervical precancer and reaching underscreened women by using HPV testing on self-samples: updated meta-analyses. BMJ. 2018;363:k4823.

2. National Cancer Institute Surveillance, Epidemiology, and End Results Program. Cancer stat facts: cervical cancer. www.seer.cancer.gov/statfacts/html/cervix.html. Accessed June 29, 2020.

3. Singh GK, Azuine RE, Siahpush M. Global inequalities in cervical cancer incidence and mortality are linked to deprivation, low socioeconomic status, and human development. Int J MCH AIDS. 2012;1:17‐30.

4. US Cancer Statistics Working Group. US Cancer Statistics Data Visualizations Tool, based on November 2018 submission data (1999-2016): US Department of Health and Human Services, Centers for Disease Control and Prevention and National Cancer Institute. June 2019. www.cdc.gov/cancer/dataviz. Accessed June 29, 2020.

5. MacLaughlin KL, Jacobson RM, Breitkopf CR, et al. Trends over time in Pap and Pap-HPV cotesting for cervical cancer screening. J Womens Health. 2019;28:244-249.

6. Freeman HP, Wingrove BK. Excess Cervical Cancer Mortality: A Marker for Low Access to Health Care in Poor Communities. NIH Pub. No. 05–5282. Rockville, MD: National Cancer Institute, Center to Reduce Cancer Health Disparities, May 2005. www.cancer.gov/about-nci/organization/crchd/about-health-disparities/resources/excess-cervical-cancer-mortality.pdf. Accessed June 29, 2020.

7. FDA approves expanded use of HPV test. Infection Control Today. March 31, 2003. https://www.infectioncontroltoday.com/view/fda-approves-expanded-use-hpv-test. Accessed June 29, 2020.

8. Ronco G, Dillner J, Elfström K, et al. Efficacy of HPV-based screening for prevention of invasive cervical cancer: follow-up of four European randomised controlled trials. Lancet. 2014;383:524-532.

9. CDC. Cervical cancer screening guidelines for average-risk women. www.cdc.gov/cancer/cervical/pdf/guidelines.pdf. Accessed June 29, 2020.

10. Mao C, Kulasingam S, Whitham H, et al. Clinician and patient acceptability of self-collected human papillomavirus testing for cervical cancer screening. J Womens Health. 2017;26:609-615.

11. Arbyn M, Verdoodt F, Snijders PJ, et al. Accuracy of human papillomavirus testing on self-collected versus clinician-collected samples: a meta-analysis. Lancet Oncol. 2014;15:172-183.

12. Lazcano-Ponce E, Lorincz A, Cruz-Valdez A, et al. Self-collection of vaginal specimens for human papillomavirus testing in cervical cancer prevention (MARCH): a community-based randomised controlled trial. Lancet. 2011;378:1868-1873.

References

1. Arbyn M, Smith SB, Temin S, et al; Collaboration on Self-Sampling and HPV Testing. Detecting cervical precancer and reaching underscreened women by using HPV testing on self-samples: updated meta-analyses. BMJ. 2018;363:k4823.

2. National Cancer Institute Surveillance, Epidemiology, and End Results Program. Cancer stat facts: cervical cancer. www.seer.cancer.gov/statfacts/html/cervix.html. Accessed June 29, 2020.

3. Singh GK, Azuine RE, Siahpush M. Global inequalities in cervical cancer incidence and mortality are linked to deprivation, low socioeconomic status, and human development. Int J MCH AIDS. 2012;1:17‐30.

4. US Cancer Statistics Working Group. US Cancer Statistics Data Visualizations Tool, based on November 2018 submission data (1999-2016): US Department of Health and Human Services, Centers for Disease Control and Prevention and National Cancer Institute. June 2019. www.cdc.gov/cancer/dataviz. Accessed June 29, 2020.

5. MacLaughlin KL, Jacobson RM, Breitkopf CR, et al. Trends over time in Pap and Pap-HPV cotesting for cervical cancer screening. J Womens Health. 2019;28:244-249.

6. Freeman HP, Wingrove BK. Excess Cervical Cancer Mortality: A Marker for Low Access to Health Care in Poor Communities. NIH Pub. No. 05–5282. Rockville, MD: National Cancer Institute, Center to Reduce Cancer Health Disparities, May 2005. www.cancer.gov/about-nci/organization/crchd/about-health-disparities/resources/excess-cervical-cancer-mortality.pdf. Accessed June 29, 2020.

7. FDA approves expanded use of HPV test. Infection Control Today. March 31, 2003. https://www.infectioncontroltoday.com/view/fda-approves-expanded-use-hpv-test. Accessed June 29, 2020.

8. Ronco G, Dillner J, Elfström K, et al. Efficacy of HPV-based screening for prevention of invasive cervical cancer: follow-up of four European randomised controlled trials. Lancet. 2014;383:524-532.

9. CDC. Cervical cancer screening guidelines for average-risk women. www.cdc.gov/cancer/cervical/pdf/guidelines.pdf. Accessed June 29, 2020.

10. Mao C, Kulasingam S, Whitham H, et al. Clinician and patient acceptability of self-collected human papillomavirus testing for cervical cancer screening. J Womens Health. 2017;26:609-615.

11. Arbyn M, Verdoodt F, Snijders PJ, et al. Accuracy of human papillomavirus testing on self-collected versus clinician-collected samples: a meta-analysis. Lancet Oncol. 2014;15:172-183.

12. Lazcano-Ponce E, Lorincz A, Cruz-Valdez A, et al. Self-collection of vaginal specimens for human papillomavirus testing in cervical cancer prevention (MARCH): a community-based randomised controlled trial. Lancet. 2011;378:1868-1873.

Issue
The Journal of Family Practice - 69(6)
Issue
The Journal of Family Practice - 69(6)
Page Number
306-308
Page Number
306-308
Publications
Publications
Topics
Article Type
Display Headline
Do-it-yourself cervical cancer screening?
Display Headline
Do-it-yourself cervical cancer screening?
Sections
PURLs Copyright
Copyright © 2020. The Family Physicians Inquiries Network. All rights reserved.
Inside the Article

PRACTICE CHANGER

Have patients who decline a pelvic examination self-collect a specimen for human papillomavirus polymerase chain reaction testing as an alternative to a clinician-collected one.

STRENGTH OF RECOMMENDATION

B: Meta-analysis of observational trials.1

Arbyn M, Smith SB, Temin S, et al; Collaboration on Self-Sampling and HPV Testing. Detecting cervical precancer and reaching under-screened women by using HPV testing on self-samples: updated meta-analyses. BMJ. 2018;363:k4823.

Disallow All Ads
Content Gating
No Gating (article Unlocked/Free)
Alternative CME
PubMed ID
32724910
Disqus Comments
Default
Use ProPublica
Hide sidebar & use full width
render the right sidebar.
Conference Recap Checkbox
Not Conference Recap
Clinical Edge
Display the Slideshow in this Article
Article PDF Media
Image
Teambase ID
18001882.SIG
Disable zoom
Off
Media Files
Image
Disable zoom
Off
Media Folder

Immediate or delayed pushing in the second stage of labor?

Article Type
Changed
Mon, 02/01/2021 - 15:16
Display Headline
Immediate or delayed pushing in the second stage of labor?

ILLUSTRATIVE CASE

A 27-year-old G1P000 at term with an uncomplicated pregnancy has been laboring for 6 hours with an epidural in place and a reassuring fetal heart tracing. She is at –2 station with complete cervical dilation and effacement. Should she push now or delay pushing to allow for more descent?

More than 10,000 women give birth each day in the United States, yet few of our approaches to labor management are evidence based.2 For example, there are no clear guidelines on whether immediate pushing or delayed pushing (waiting 1-2 hours) in the second stage of labor (the time from complete cervical dilation to delivery of the fetus) leads to better outcomes.

A recent Cochrane review, which included very low- to moderate-quality trials of nulliparous and multiparous women using epidural analgesia showed that delayed pushing resulted in more vaginal deliveries, longer duration of second stage of labor, and shorter duration of pushing.3 But many of the trials included in this Cochrane review were noted to have study design limitations and significant heterogeneity.

A recent retrospective study found that delayed pushing resulted in longer duration of pushing and increased risks for cesarean section, operative vaginal delivery, and postpartum hemorrhage in nulliparous patients with and without epidurals.4 The World Health Organization recommends delayed pushing in women with epidural analgesia if time and fetal monitoring resources are available.5

STUDY SUMMARY

Does the timing of second stage pushing efforts affect outcomes?

This multicenter randomized controlled trial (RCT) evaluated the effect on spontaneous vaginal delivery of delayed pushing vs immediate pushing in 2404 term nulliparous women using epidural analgesia.1 Patients were ≥ 37 weeks’ gestation. Once patients achieved 10 cm of cervical dilation, they were randomized in a 1:1 ratio to either immediate pushing or to delayed (for 60 minutes) pushing (unless there was an irresistible urge to push or they were otherwise instructed by their provider).

Outcome and results. The primary outcome was spontaneous vaginal delivery without the use of any operative support. The mean time to pushing after complete cervical dilation was 19 minutes in the immediate pushing group and 60 minutes in the delayed group. There was no difference in the rate of spontaneous vaginal delivery between the immediate and delayed pushing groups (86% vs 87%, respectively; P = .67). The immediate pushing group had a shorter duration of second stage of labor (102 minutes vs 134 minutes; mean difference [MD] = –32 minutes; 95% confidence interval [CI], –37 to –27; P < .001) and a slightly longer duration of active pushing (84 minutes vs 75 minutes; MD = 9.2 minutes; 95% CI, 6-13; P < .001).

Delaying pushing once the cervix is completely dilated is not indicated, even for nulliparous women receiving epidural analgesia.

There was no significant difference in operative vaginal or cesarean deliveries. Postpartum hemorrhage was lower in the immediate pushing group (2.3% vs 4%; risk ratio [RR] = 0.6; 95% CI, 0.3-0.9; P = .03; number needed to treat [NNT] = 58), as was chorioamnionitis (6.7% vs 9.1%; RR = 0.7; 95% CI, 0.66-0.90; P = .005; NNT = 40). There was no significant difference in neonatal morbidity between groups. And in subgroup analysis, there was no significant difference in rates of vaginal delivery based on fetal position (occiput anterior, posterior, or transverse) or station (defined as high [< 2 cm] or low [≥ 2 cm]) between groups. Recruitment was stopped early at 75% because there was no difference in the primary outcome and there was concern regarding an increased risk of hemorrhage in the delayed pushing group.

Continue to: WHAT'S NEW

 

 

WHAT’S NEW

There’s no good reason to delay pushing

Delaying pushing once the cervix is completely dilated is not indicated, even for nulliparous women receiving epidural analgesia, as it does not decrease the rate of spontaneous vaginal delivery. It does, however, increase the length of second stage labor and the risk of postpartum hemorrhage and chorioamnionitis.

CAVEATS

Study was stopped early, and groups were unblinded

This study was stopped early, so it is not known if it was underpowered for some of the secondary outcomes. Also, it was not possible to blind the groups, so it is not clear if any bias in patient management or diagnosis resulted.

CHALLENGES TO IMPLEMENTATION

Will current practice and culture pose obstacles?

Although the overt challenges to enacting a policy of immediate pushing are minimal, the inertia of current practice and culture could affect the implementation of this strategy.

 

ACKNOWLEDGEMENT

The PURLs Surveillance System was supported in part by Grant Number UL1RR024999 from the National Center For Research Resources, a Clinical Translational Science Award to the University of Chicago. The content is solely the responsibility of the authors and does not necessarily represent the official views of the National Center For Research Resources or the National Institutes of Health.

Files
References

1. Cahill AG, Srinivas SK, Tita ATN, et al. Effect of immediate vs delayed pushing on rates of spontaneous vaginal delivery among nulliparous women receiving neuraxial analgesia: a randomized clinical trial. JAMA. 2018;320:1444-1454.

2. Hamilton BE, Martin JA, Osterman MJK, et al. Births: provisional data for 2018. Vital Statistics Rapid Release. May 2019; Report No. 007. www.cdc.gov/nchs/data/vsrr/vsrr-007-508.pdf. Accessed April 22, 2020.

3. Lemos A, Amorim MM, Domales de Andrade A, et al. Pushing/bearing down methods for the second stage of labour. Cochrane Database Syst Rev. 2017;3:CD009124.

4. Yee LM, Sandoval G, Bailit J, et al. Maternal and neonatal outcomes with early compared with delayed pushing among nulliparous women. Obstet Gynecol. 2016;128:1039-1047.

5. WHO recommendations: intrapartum care for a positive childbirth experience. Geneva: World Health Organization; 2018. www.who.int/reproductivehealth/publications/intrapartum-care-guidelines/en/. Accessed April 22, 2020.

Article PDF
Author and Disclosure Information

University of Colorado Family Medicine Residency, Denver

DEPUTY EDITOR
Anne Mounsey, MD

Department of Family Medicine, University of North Carolina, Chapel Hill

Issue
The Journal of Family Practice - 69(4)
Publications
Topics
Page Number
207-208
Sections
Files
Files
Author and Disclosure Information

University of Colorado Family Medicine Residency, Denver

DEPUTY EDITOR
Anne Mounsey, MD

Department of Family Medicine, University of North Carolina, Chapel Hill

Author and Disclosure Information

University of Colorado Family Medicine Residency, Denver

DEPUTY EDITOR
Anne Mounsey, MD

Department of Family Medicine, University of North Carolina, Chapel Hill

Article PDF
Article PDF

ILLUSTRATIVE CASE

A 27-year-old G1P000 at term with an uncomplicated pregnancy has been laboring for 6 hours with an epidural in place and a reassuring fetal heart tracing. She is at –2 station with complete cervical dilation and effacement. Should she push now or delay pushing to allow for more descent?

More than 10,000 women give birth each day in the United States, yet few of our approaches to labor management are evidence based.2 For example, there are no clear guidelines on whether immediate pushing or delayed pushing (waiting 1-2 hours) in the second stage of labor (the time from complete cervical dilation to delivery of the fetus) leads to better outcomes.

A recent Cochrane review, which included very low- to moderate-quality trials of nulliparous and multiparous women using epidural analgesia showed that delayed pushing resulted in more vaginal deliveries, longer duration of second stage of labor, and shorter duration of pushing.3 But many of the trials included in this Cochrane review were noted to have study design limitations and significant heterogeneity.

A recent retrospective study found that delayed pushing resulted in longer duration of pushing and increased risks for cesarean section, operative vaginal delivery, and postpartum hemorrhage in nulliparous patients with and without epidurals.4 The World Health Organization recommends delayed pushing in women with epidural analgesia if time and fetal monitoring resources are available.5

STUDY SUMMARY

Does the timing of second stage pushing efforts affect outcomes?

This multicenter randomized controlled trial (RCT) evaluated the effect on spontaneous vaginal delivery of delayed pushing vs immediate pushing in 2404 term nulliparous women using epidural analgesia.1 Patients were ≥ 37 weeks’ gestation. Once patients achieved 10 cm of cervical dilation, they were randomized in a 1:1 ratio to either immediate pushing or to delayed (for 60 minutes) pushing (unless there was an irresistible urge to push or they were otherwise instructed by their provider).

Outcome and results. The primary outcome was spontaneous vaginal delivery without the use of any operative support. The mean time to pushing after complete cervical dilation was 19 minutes in the immediate pushing group and 60 minutes in the delayed group. There was no difference in the rate of spontaneous vaginal delivery between the immediate and delayed pushing groups (86% vs 87%, respectively; P = .67). The immediate pushing group had a shorter duration of second stage of labor (102 minutes vs 134 minutes; mean difference [MD] = –32 minutes; 95% confidence interval [CI], –37 to –27; P < .001) and a slightly longer duration of active pushing (84 minutes vs 75 minutes; MD = 9.2 minutes; 95% CI, 6-13; P < .001).

Delaying pushing once the cervix is completely dilated is not indicated, even for nulliparous women receiving epidural analgesia.

There was no significant difference in operative vaginal or cesarean deliveries. Postpartum hemorrhage was lower in the immediate pushing group (2.3% vs 4%; risk ratio [RR] = 0.6; 95% CI, 0.3-0.9; P = .03; number needed to treat [NNT] = 58), as was chorioamnionitis (6.7% vs 9.1%; RR = 0.7; 95% CI, 0.66-0.90; P = .005; NNT = 40). There was no significant difference in neonatal morbidity between groups. And in subgroup analysis, there was no significant difference in rates of vaginal delivery based on fetal position (occiput anterior, posterior, or transverse) or station (defined as high [< 2 cm] or low [≥ 2 cm]) between groups. Recruitment was stopped early at 75% because there was no difference in the primary outcome and there was concern regarding an increased risk of hemorrhage in the delayed pushing group.

Continue to: WHAT'S NEW

 

 

WHAT’S NEW

There’s no good reason to delay pushing

Delaying pushing once the cervix is completely dilated is not indicated, even for nulliparous women receiving epidural analgesia, as it does not decrease the rate of spontaneous vaginal delivery. It does, however, increase the length of second stage labor and the risk of postpartum hemorrhage and chorioamnionitis.

CAVEATS

Study was stopped early, and groups were unblinded

This study was stopped early, so it is not known if it was underpowered for some of the secondary outcomes. Also, it was not possible to blind the groups, so it is not clear if any bias in patient management or diagnosis resulted.

CHALLENGES TO IMPLEMENTATION

Will current practice and culture pose obstacles?

Although the overt challenges to enacting a policy of immediate pushing are minimal, the inertia of current practice and culture could affect the implementation of this strategy.

 

ACKNOWLEDGEMENT

The PURLs Surveillance System was supported in part by Grant Number UL1RR024999 from the National Center For Research Resources, a Clinical Translational Science Award to the University of Chicago. The content is solely the responsibility of the authors and does not necessarily represent the official views of the National Center For Research Resources or the National Institutes of Health.

ILLUSTRATIVE CASE

A 27-year-old G1P000 at term with an uncomplicated pregnancy has been laboring for 6 hours with an epidural in place and a reassuring fetal heart tracing. She is at –2 station with complete cervical dilation and effacement. Should she push now or delay pushing to allow for more descent?

More than 10,000 women give birth each day in the United States, yet few of our approaches to labor management are evidence based.2 For example, there are no clear guidelines on whether immediate pushing or delayed pushing (waiting 1-2 hours) in the second stage of labor (the time from complete cervical dilation to delivery of the fetus) leads to better outcomes.

A recent Cochrane review, which included very low- to moderate-quality trials of nulliparous and multiparous women using epidural analgesia showed that delayed pushing resulted in more vaginal deliveries, longer duration of second stage of labor, and shorter duration of pushing.3 But many of the trials included in this Cochrane review were noted to have study design limitations and significant heterogeneity.

A recent retrospective study found that delayed pushing resulted in longer duration of pushing and increased risks for cesarean section, operative vaginal delivery, and postpartum hemorrhage in nulliparous patients with and without epidurals.4 The World Health Organization recommends delayed pushing in women with epidural analgesia if time and fetal monitoring resources are available.5

STUDY SUMMARY

Does the timing of second stage pushing efforts affect outcomes?

This multicenter randomized controlled trial (RCT) evaluated the effect on spontaneous vaginal delivery of delayed pushing vs immediate pushing in 2404 term nulliparous women using epidural analgesia.1 Patients were ≥ 37 weeks’ gestation. Once patients achieved 10 cm of cervical dilation, they were randomized in a 1:1 ratio to either immediate pushing or to delayed (for 60 minutes) pushing (unless there was an irresistible urge to push or they were otherwise instructed by their provider).

Outcome and results. The primary outcome was spontaneous vaginal delivery without the use of any operative support. The mean time to pushing after complete cervical dilation was 19 minutes in the immediate pushing group and 60 minutes in the delayed group. There was no difference in the rate of spontaneous vaginal delivery between the immediate and delayed pushing groups (86% vs 87%, respectively; P = .67). The immediate pushing group had a shorter duration of second stage of labor (102 minutes vs 134 minutes; mean difference [MD] = –32 minutes; 95% confidence interval [CI], –37 to –27; P < .001) and a slightly longer duration of active pushing (84 minutes vs 75 minutes; MD = 9.2 minutes; 95% CI, 6-13; P < .001).

Delaying pushing once the cervix is completely dilated is not indicated, even for nulliparous women receiving epidural analgesia.

There was no significant difference in operative vaginal or cesarean deliveries. Postpartum hemorrhage was lower in the immediate pushing group (2.3% vs 4%; risk ratio [RR] = 0.6; 95% CI, 0.3-0.9; P = .03; number needed to treat [NNT] = 58), as was chorioamnionitis (6.7% vs 9.1%; RR = 0.7; 95% CI, 0.66-0.90; P = .005; NNT = 40). There was no significant difference in neonatal morbidity between groups. And in subgroup analysis, there was no significant difference in rates of vaginal delivery based on fetal position (occiput anterior, posterior, or transverse) or station (defined as high [< 2 cm] or low [≥ 2 cm]) between groups. Recruitment was stopped early at 75% because there was no difference in the primary outcome and there was concern regarding an increased risk of hemorrhage in the delayed pushing group.

Continue to: WHAT'S NEW

 

 

WHAT’S NEW

There’s no good reason to delay pushing

Delaying pushing once the cervix is completely dilated is not indicated, even for nulliparous women receiving epidural analgesia, as it does not decrease the rate of spontaneous vaginal delivery. It does, however, increase the length of second stage labor and the risk of postpartum hemorrhage and chorioamnionitis.

CAVEATS

Study was stopped early, and groups were unblinded

This study was stopped early, so it is not known if it was underpowered for some of the secondary outcomes. Also, it was not possible to blind the groups, so it is not clear if any bias in patient management or diagnosis resulted.

CHALLENGES TO IMPLEMENTATION

Will current practice and culture pose obstacles?

Although the overt challenges to enacting a policy of immediate pushing are minimal, the inertia of current practice and culture could affect the implementation of this strategy.

 

ACKNOWLEDGEMENT

The PURLs Surveillance System was supported in part by Grant Number UL1RR024999 from the National Center For Research Resources, a Clinical Translational Science Award to the University of Chicago. The content is solely the responsibility of the authors and does not necessarily represent the official views of the National Center For Research Resources or the National Institutes of Health.

References

1. Cahill AG, Srinivas SK, Tita ATN, et al. Effect of immediate vs delayed pushing on rates of spontaneous vaginal delivery among nulliparous women receiving neuraxial analgesia: a randomized clinical trial. JAMA. 2018;320:1444-1454.

2. Hamilton BE, Martin JA, Osterman MJK, et al. Births: provisional data for 2018. Vital Statistics Rapid Release. May 2019; Report No. 007. www.cdc.gov/nchs/data/vsrr/vsrr-007-508.pdf. Accessed April 22, 2020.

3. Lemos A, Amorim MM, Domales de Andrade A, et al. Pushing/bearing down methods for the second stage of labour. Cochrane Database Syst Rev. 2017;3:CD009124.

4. Yee LM, Sandoval G, Bailit J, et al. Maternal and neonatal outcomes with early compared with delayed pushing among nulliparous women. Obstet Gynecol. 2016;128:1039-1047.

5. WHO recommendations: intrapartum care for a positive childbirth experience. Geneva: World Health Organization; 2018. www.who.int/reproductivehealth/publications/intrapartum-care-guidelines/en/. Accessed April 22, 2020.

References

1. Cahill AG, Srinivas SK, Tita ATN, et al. Effect of immediate vs delayed pushing on rates of spontaneous vaginal delivery among nulliparous women receiving neuraxial analgesia: a randomized clinical trial. JAMA. 2018;320:1444-1454.

2. Hamilton BE, Martin JA, Osterman MJK, et al. Births: provisional data for 2018. Vital Statistics Rapid Release. May 2019; Report No. 007. www.cdc.gov/nchs/data/vsrr/vsrr-007-508.pdf. Accessed April 22, 2020.

3. Lemos A, Amorim MM, Domales de Andrade A, et al. Pushing/bearing down methods for the second stage of labour. Cochrane Database Syst Rev. 2017;3:CD009124.

4. Yee LM, Sandoval G, Bailit J, et al. Maternal and neonatal outcomes with early compared with delayed pushing among nulliparous women. Obstet Gynecol. 2016;128:1039-1047.

5. WHO recommendations: intrapartum care for a positive childbirth experience. Geneva: World Health Organization; 2018. www.who.int/reproductivehealth/publications/intrapartum-care-guidelines/en/. Accessed April 22, 2020.

Issue
The Journal of Family Practice - 69(4)
Issue
The Journal of Family Practice - 69(4)
Page Number
207-208
Page Number
207-208
Publications
Publications
Topics
Article Type
Display Headline
Immediate or delayed pushing in the second stage of labor?
Display Headline
Immediate or delayed pushing in the second stage of labor?
Sections
PURLs Copyright
Copyright © 2020. The Family Physicians Inquiries Network. All rights reserved.
Inside the Article

PRACTICE CHANGER

Recommend immediate, rather than delayed, pushing in the second stage of labor for nulliparous women receiving epidural analgesia. The rate of spontaneous vaginal delivery is the same, and there is a lower risk of postpartum hemorrhage and chorioamnionitis.

STRENGTH OF RECOMMENDATION

B: Based on an individual randomized controlled trial. 1

Cahill AG, Srinivas SK, Tita ATN, et al. Effect of immediate vs delayed pushing on rates of spontaneous vaginal delivery among nulliparous women receiving neuraxial analgesia: a randomized clinical trial. JAMA. 2018;320:1444-1454.

Disallow All Ads
Content Gating
No Gating (article Unlocked/Free)
Alternative CME
PubMed ID
32437485
Disqus Comments
Default
Use ProPublica
Hide sidebar & use full width
render the right sidebar.
Conference Recap Checkbox
Not Conference Recap
Clinical Edge
Display the Slideshow in this Article
Medscape Article
Article PDF Media
Image
Teambase ID
18001772.SIG
Disable zoom
Off
Media Files
Image
Disable zoom
Off
Media Folder

Aspirin, Yes, for at-risk elderly—but what about the healthy elderly?

Article Type
Changed
Wed, 04/15/2020 - 07:48
Display Headline
Aspirin, Yes, for at-risk elderly—but what about the healthy elderly?

ILLUSTRATIVE CASE

A healthy 72-year-old man with well-controlled hypertension on amlodipine 10 mg/d presents to you for an annual exam. He has no history of coronary artery disease or stroke. Should you recommend that he start aspirin for primary prevention of cardiovascular disease?

Cardiovascular disease (CVD) remains the leading cause of death in the United States.2 Aspirin therapy remains the standard of care for secondary prevention of CVD in patients with known coronary artery disease (CAD).3 Aspirin reduces the risk of atherothrombosis by irreversibly inhibiting platelet function. At the same time, it increases the risk of major bleeding, including gastrointestinal bleeds and hemorrhagic strokes. Even though the benefit of aspirin in patients with known CAD is well established, the benefit of aspirin as primary prevention is less certain.

Two recent large randomized controlled trials (RCTs) examined the benefits and risks of aspirin in a variety of patient populations. The ARRIVE trial looked at more than 12,000 patients with a mean age of 63 years with moderate risk of CVD (approximately 15% risk of a cardiovascular event in 10 years) and randomly assigned them to receive aspirin or placebo.4 After an average follow-up period of 5 years, researchers observed that actual cardiovascular event risk was < 10% in both groups, and there was no significant difference in the primary outcome of first cardiovascular event or all-cause mortality. There was, however, a significant increase in bleeding events in the group receiving aspirin.4

The ASCEND trial evaluated aspirin vs placebo in more than 15,000 adult patients with type 2 diabetes mellitus and a low risk of CVD (< 10% risk of cardiovascular event in 5 years). 5 The primary endpoint of the study was first cardiovascular event. The authors found a significantly lower rate of cardiovascular events in the aspirin group, as well as more major bleeding events. Additionally, there was no difference between the aspirin and placebo groups in all-cause ­mortality after 7 years. The authors concluded that the benefits of aspirin in this group were ­counterbalanced by the harms.5

Currently, several organizations offer recommendations on aspirin use in people 40 to 70 years of age based on a patient’s risk of bleeding and risk of CVD.6-8 Recommendations regarding aspirin use as primary prevention have been less clear for patients < 40 and > 70 years of age.6

Elderly patients are at higher risk of CVD and bleeding, but until recently, few studies had evaluated elderly populations to assess the benefits vs the risks of aspirin for primary CVD prevention. As of 2016, the US Preventive Services Task Force (USPSTF) stated the evidence was insufficient to assess the balance of the benefits and harms of initiating aspirin use for primary prevention of CVD in patients older than 70 years of age.6 This trial focuses on aspirin use for primary prevention of CVD in healthy elderly adults.

STUDY SUMMARY

Don’t use aspirin as primary prevention of CVD in the elderly

This secondary analysis of a prior double-blind RCT, which found low-dose aspirin did not prolong survival in elderly patients, examined the effect of aspirin on CVD and hemorrhage in 19,114 elderly patients without known CVD.1 The patients were ≥ 70 years of age (≥ 65 years for blacks and Hispanics) with a mean age of 74 years and were from Australia (87%) and the United States (13%). Approximately one-third of the patients were taking a statin, and 14% were taking a nonsteroidal anti-inflammatory drug (NSAID) regularly. Patients were randomized to either aspirin 100 mg/d or matching placebo and were followed for an average of 4.7 years.

Continue to: Outcomes

 

 

Outcomes. The outcome of CVD was a composite of fatal coronary heart disease, nonfatal myocardial infarction (MI), fatal or nonfatal ischemic stroke, or hospitalization for heart failure, and the outcome of major adverse cardiovascular event was a composite of fatal cardiovascular disease (excluding death from heart failure), nonfatal MI, or fatal and nonfatal ischemic stroke.

Results. No difference was seen between the aspirin and placebo groups in CVD outcomes (10.7 events per 1000 person-years vs 11.3 events per 1000 person-years, respectively; hazard ratio [HR] = 0.95; 95% confidence interval [CI], 0.83-1.08) or major cardiovascular events (7.8 events per 1000 person-years vs 8.8 events per 1000 person-years, respectively; HR = 0.89; 95% CI, 0.77-1.03). The composite and individual endpoints of fatal cardiovascular disease, heart failure hospitalizations, fatal and nonfatal MI, and ischemic stroke also did not differ significantly between the groups.

Because of this trial, the ACC and AHA have updated their guidelines on primary prevention of CVD to recommend against the routine use of aspirin in patients > 70 years of age.

The rate of major hemorrhagic events (composite of hemorrhagic stroke, intracranial bleed, or extracranial bleed), however, was higher in the aspirin vs the placebo group (8.6 events per 1000 person-years vs 6.2 events per 1000 person-years, respectively; HR = 1.4; 95% CI, 1.2-1.6; number needed to harm = 98).

 

WHAT’S NEW

Finding of more harm than good leads to change in ACC/AHA guidelines

Although the most recent USPSTF guidelines state the evidence is insufficient to assess the risks and benefits of aspirin for the primary prevention of cardiovascular disease in this age group, this trial reveals there is a greater risk of hemorrhagic events than there is prevention of cardiovascular outcomes with aspirin use in healthy elderly patients > 70 years of age.6 Because of this trial, the American College of Cardiology (ACC) and the American Heart Association (AHA) have updated their guidelines on the primary prevention of cardiovascular disease to recommend that aspirin not be used routinely in patients > 70 years of age.7

CAVEATS

Potential benefit to people at higher risk?

The rate of cardiovascular disease was lower than expected in this overall healthy population, so it is not known if cardiovascular benefits may outweigh the risk of bleeding in a higher-risk population. The trial also didn’t address the potential harms of deprescribing aspirin. Additionally, although aspirin may not be protective for cardiovascular events and may lead to more bleeding, there may be other benefits to aspirin in this patient population that were not addressed by this study.

Continue to: CHALLENGES TO IMPLEMENTATION

 

 

CHALLENGES TO IMPLEMENTATION

Popular beliefs and wide availability may make tide difficult to change

Patients have been told for years to take a daily aspirin to “protect their heart”; this behavior may be difficult to change. And because aspirin is widely available over the counter, patients may take it without their physician’s knowledge.

ACKNOWLEDGEMENT

The PURLs Surveillance System was supported in part by Grant Number UL1RR024999 from the National Center For Research Resources, a Clinical Translational Science Award to the University of Chicago. The content is solely the responsibility of the authors and does not necessarily represent the official views of the National Center For Research Resources or the National Institutes of Health.

Files
References

1. McNeil JJ, Wolfe R, Woods RL, et al. Effect of aspirin on cardiovascular events and bleeding in the healthy elderly. N Engl J Med. 2018;379:1509-1518.

2. Murphy SL, Xu JQ, Kochanek KD, et al. Mortality in the United States, 2017. NCHS Data Brief, no. 328. Hyattsville, MD: National Center for Health Statistics. 2018.

3. Smith SC Jr, Benjamin EJ, Bonow RO, et al. AHA/ACCF secondary prevention and risk reduction therapy for patients with coronary and other atherosclerotic vascular disease: a guideline from the American Heart Association and American College of Cardiology Foundation. Circulation. 2011;124:2458-2473.

4. Gaziano JM, Brotons C, Coppolecchia R, et al. Use of aspirin to reduce risk of initial vascular events in patients at moderate risk of cardiovascular disease (ARRIVE): a randomised, double-blind, placebo-controlled trial. Lancet. 2018;392:1036-1046.

5. Bowman L, Mafham M, Wallendszus K, et al; ASCEND Study Collaborative Group. Effects of aspirin for primary prevention in persons with diabetes mellitus. N Engl J Med. 2018;379:1529-1539. 

6. Bibbins-Domingo K; U.S. Preventive Services Task Force. Aspirin use for the primary prevention of cardiovascular disease and colorectal cancer: U.S. Preventive Services Task Force Recommendation Statement. Ann Intern Med. 2016;164:836-845.

7. Arnett DK, Blumenthal RS, Albert MA, et al. 2019 ACC/AHA Guideline on the Primary Prevention of Cardiovascular Disease: Executive Summary: A Report of the American College of Cardiology/American Heart Association Task Force on Clinical Practice Guidelines. J Amer Coll Cardiol. 2019;74:1376-1414. 

8. American Diabetes Association. Cardiovascular Disease and Risk Management: Standards of Medical Care in Diabetes-2019. Diabetes Care. 2019;42(Suppl 1):S103-S123.

Article PDF
Author and Disclosure Information

University of Colorado Family Medicine Residency, Denver

DEPUTY EDITOR
Anne Mounsey, MD

Department of Family Medicine, University of North Carolina, Chapel Hill

Issue
The Journal of Family Practice - 69(3)
Publications
Topics
Page Number
E16-E18
Sections
Files
Files
Author and Disclosure Information

University of Colorado Family Medicine Residency, Denver

DEPUTY EDITOR
Anne Mounsey, MD

Department of Family Medicine, University of North Carolina, Chapel Hill

Author and Disclosure Information

University of Colorado Family Medicine Residency, Denver

DEPUTY EDITOR
Anne Mounsey, MD

Department of Family Medicine, University of North Carolina, Chapel Hill

Article PDF
Article PDF

ILLUSTRATIVE CASE

A healthy 72-year-old man with well-controlled hypertension on amlodipine 10 mg/d presents to you for an annual exam. He has no history of coronary artery disease or stroke. Should you recommend that he start aspirin for primary prevention of cardiovascular disease?

Cardiovascular disease (CVD) remains the leading cause of death in the United States.2 Aspirin therapy remains the standard of care for secondary prevention of CVD in patients with known coronary artery disease (CAD).3 Aspirin reduces the risk of atherothrombosis by irreversibly inhibiting platelet function. At the same time, it increases the risk of major bleeding, including gastrointestinal bleeds and hemorrhagic strokes. Even though the benefit of aspirin in patients with known CAD is well established, the benefit of aspirin as primary prevention is less certain.

Two recent large randomized controlled trials (RCTs) examined the benefits and risks of aspirin in a variety of patient populations. The ARRIVE trial looked at more than 12,000 patients with a mean age of 63 years with moderate risk of CVD (approximately 15% risk of a cardiovascular event in 10 years) and randomly assigned them to receive aspirin or placebo.4 After an average follow-up period of 5 years, researchers observed that actual cardiovascular event risk was < 10% in both groups, and there was no significant difference in the primary outcome of first cardiovascular event or all-cause mortality. There was, however, a significant increase in bleeding events in the group receiving aspirin.4

The ASCEND trial evaluated aspirin vs placebo in more than 15,000 adult patients with type 2 diabetes mellitus and a low risk of CVD (< 10% risk of cardiovascular event in 5 years). 5 The primary endpoint of the study was first cardiovascular event. The authors found a significantly lower rate of cardiovascular events in the aspirin group, as well as more major bleeding events. Additionally, there was no difference between the aspirin and placebo groups in all-cause ­mortality after 7 years. The authors concluded that the benefits of aspirin in this group were ­counterbalanced by the harms.5

Currently, several organizations offer recommendations on aspirin use in people 40 to 70 years of age based on a patient’s risk of bleeding and risk of CVD.6-8 Recommendations regarding aspirin use as primary prevention have been less clear for patients < 40 and > 70 years of age.6

Elderly patients are at higher risk of CVD and bleeding, but until recently, few studies had evaluated elderly populations to assess the benefits vs the risks of aspirin for primary CVD prevention. As of 2016, the US Preventive Services Task Force (USPSTF) stated the evidence was insufficient to assess the balance of the benefits and harms of initiating aspirin use for primary prevention of CVD in patients older than 70 years of age.6 This trial focuses on aspirin use for primary prevention of CVD in healthy elderly adults.

STUDY SUMMARY

Don’t use aspirin as primary prevention of CVD in the elderly

This secondary analysis of a prior double-blind RCT, which found low-dose aspirin did not prolong survival in elderly patients, examined the effect of aspirin on CVD and hemorrhage in 19,114 elderly patients without known CVD.1 The patients were ≥ 70 years of age (≥ 65 years for blacks and Hispanics) with a mean age of 74 years and were from Australia (87%) and the United States (13%). Approximately one-third of the patients were taking a statin, and 14% were taking a nonsteroidal anti-inflammatory drug (NSAID) regularly. Patients were randomized to either aspirin 100 mg/d or matching placebo and were followed for an average of 4.7 years.

Continue to: Outcomes

 

 

Outcomes. The outcome of CVD was a composite of fatal coronary heart disease, nonfatal myocardial infarction (MI), fatal or nonfatal ischemic stroke, or hospitalization for heart failure, and the outcome of major adverse cardiovascular event was a composite of fatal cardiovascular disease (excluding death from heart failure), nonfatal MI, or fatal and nonfatal ischemic stroke.

Results. No difference was seen between the aspirin and placebo groups in CVD outcomes (10.7 events per 1000 person-years vs 11.3 events per 1000 person-years, respectively; hazard ratio [HR] = 0.95; 95% confidence interval [CI], 0.83-1.08) or major cardiovascular events (7.8 events per 1000 person-years vs 8.8 events per 1000 person-years, respectively; HR = 0.89; 95% CI, 0.77-1.03). The composite and individual endpoints of fatal cardiovascular disease, heart failure hospitalizations, fatal and nonfatal MI, and ischemic stroke also did not differ significantly between the groups.

Because of this trial, the ACC and AHA have updated their guidelines on primary prevention of CVD to recommend against the routine use of aspirin in patients > 70 years of age.

The rate of major hemorrhagic events (composite of hemorrhagic stroke, intracranial bleed, or extracranial bleed), however, was higher in the aspirin vs the placebo group (8.6 events per 1000 person-years vs 6.2 events per 1000 person-years, respectively; HR = 1.4; 95% CI, 1.2-1.6; number needed to harm = 98).

 

WHAT’S NEW

Finding of more harm than good leads to change in ACC/AHA guidelines

Although the most recent USPSTF guidelines state the evidence is insufficient to assess the risks and benefits of aspirin for the primary prevention of cardiovascular disease in this age group, this trial reveals there is a greater risk of hemorrhagic events than there is prevention of cardiovascular outcomes with aspirin use in healthy elderly patients > 70 years of age.6 Because of this trial, the American College of Cardiology (ACC) and the American Heart Association (AHA) have updated their guidelines on the primary prevention of cardiovascular disease to recommend that aspirin not be used routinely in patients > 70 years of age.7

CAVEATS

Potential benefit to people at higher risk?

The rate of cardiovascular disease was lower than expected in this overall healthy population, so it is not known if cardiovascular benefits may outweigh the risk of bleeding in a higher-risk population. The trial also didn’t address the potential harms of deprescribing aspirin. Additionally, although aspirin may not be protective for cardiovascular events and may lead to more bleeding, there may be other benefits to aspirin in this patient population that were not addressed by this study.

Continue to: CHALLENGES TO IMPLEMENTATION

 

 

CHALLENGES TO IMPLEMENTATION

Popular beliefs and wide availability may make tide difficult to change

Patients have been told for years to take a daily aspirin to “protect their heart”; this behavior may be difficult to change. And because aspirin is widely available over the counter, patients may take it without their physician’s knowledge.

ACKNOWLEDGEMENT

The PURLs Surveillance System was supported in part by Grant Number UL1RR024999 from the National Center For Research Resources, a Clinical Translational Science Award to the University of Chicago. The content is solely the responsibility of the authors and does not necessarily represent the official views of the National Center For Research Resources or the National Institutes of Health.

ILLUSTRATIVE CASE

A healthy 72-year-old man with well-controlled hypertension on amlodipine 10 mg/d presents to you for an annual exam. He has no history of coronary artery disease or stroke. Should you recommend that he start aspirin for primary prevention of cardiovascular disease?

Cardiovascular disease (CVD) remains the leading cause of death in the United States.2 Aspirin therapy remains the standard of care for secondary prevention of CVD in patients with known coronary artery disease (CAD).3 Aspirin reduces the risk of atherothrombosis by irreversibly inhibiting platelet function. At the same time, it increases the risk of major bleeding, including gastrointestinal bleeds and hemorrhagic strokes. Even though the benefit of aspirin in patients with known CAD is well established, the benefit of aspirin as primary prevention is less certain.

Two recent large randomized controlled trials (RCTs) examined the benefits and risks of aspirin in a variety of patient populations. The ARRIVE trial looked at more than 12,000 patients with a mean age of 63 years with moderate risk of CVD (approximately 15% risk of a cardiovascular event in 10 years) and randomly assigned them to receive aspirin or placebo.4 After an average follow-up period of 5 years, researchers observed that actual cardiovascular event risk was < 10% in both groups, and there was no significant difference in the primary outcome of first cardiovascular event or all-cause mortality. There was, however, a significant increase in bleeding events in the group receiving aspirin.4

The ASCEND trial evaluated aspirin vs placebo in more than 15,000 adult patients with type 2 diabetes mellitus and a low risk of CVD (< 10% risk of cardiovascular event in 5 years). 5 The primary endpoint of the study was first cardiovascular event. The authors found a significantly lower rate of cardiovascular events in the aspirin group, as well as more major bleeding events. Additionally, there was no difference between the aspirin and placebo groups in all-cause ­mortality after 7 years. The authors concluded that the benefits of aspirin in this group were ­counterbalanced by the harms.5

Currently, several organizations offer recommendations on aspirin use in people 40 to 70 years of age based on a patient’s risk of bleeding and risk of CVD.6-8 Recommendations regarding aspirin use as primary prevention have been less clear for patients < 40 and > 70 years of age.6

Elderly patients are at higher risk of CVD and bleeding, but until recently, few studies had evaluated elderly populations to assess the benefits vs the risks of aspirin for primary CVD prevention. As of 2016, the US Preventive Services Task Force (USPSTF) stated the evidence was insufficient to assess the balance of the benefits and harms of initiating aspirin use for primary prevention of CVD in patients older than 70 years of age.6 This trial focuses on aspirin use for primary prevention of CVD in healthy elderly adults.

STUDY SUMMARY

Don’t use aspirin as primary prevention of CVD in the elderly

This secondary analysis of a prior double-blind RCT, which found low-dose aspirin did not prolong survival in elderly patients, examined the effect of aspirin on CVD and hemorrhage in 19,114 elderly patients without known CVD.1 The patients were ≥ 70 years of age (≥ 65 years for blacks and Hispanics) with a mean age of 74 years and were from Australia (87%) and the United States (13%). Approximately one-third of the patients were taking a statin, and 14% were taking a nonsteroidal anti-inflammatory drug (NSAID) regularly. Patients were randomized to either aspirin 100 mg/d or matching placebo and were followed for an average of 4.7 years.

Continue to: Outcomes

 

 

Outcomes. The outcome of CVD was a composite of fatal coronary heart disease, nonfatal myocardial infarction (MI), fatal or nonfatal ischemic stroke, or hospitalization for heart failure, and the outcome of major adverse cardiovascular event was a composite of fatal cardiovascular disease (excluding death from heart failure), nonfatal MI, or fatal and nonfatal ischemic stroke.

Results. No difference was seen between the aspirin and placebo groups in CVD outcomes (10.7 events per 1000 person-years vs 11.3 events per 1000 person-years, respectively; hazard ratio [HR] = 0.95; 95% confidence interval [CI], 0.83-1.08) or major cardiovascular events (7.8 events per 1000 person-years vs 8.8 events per 1000 person-years, respectively; HR = 0.89; 95% CI, 0.77-1.03). The composite and individual endpoints of fatal cardiovascular disease, heart failure hospitalizations, fatal and nonfatal MI, and ischemic stroke also did not differ significantly between the groups.

Because of this trial, the ACC and AHA have updated their guidelines on primary prevention of CVD to recommend against the routine use of aspirin in patients > 70 years of age.

The rate of major hemorrhagic events (composite of hemorrhagic stroke, intracranial bleed, or extracranial bleed), however, was higher in the aspirin vs the placebo group (8.6 events per 1000 person-years vs 6.2 events per 1000 person-years, respectively; HR = 1.4; 95% CI, 1.2-1.6; number needed to harm = 98).

 

WHAT’S NEW

Finding of more harm than good leads to change in ACC/AHA guidelines

Although the most recent USPSTF guidelines state the evidence is insufficient to assess the risks and benefits of aspirin for the primary prevention of cardiovascular disease in this age group, this trial reveals there is a greater risk of hemorrhagic events than there is prevention of cardiovascular outcomes with aspirin use in healthy elderly patients > 70 years of age.6 Because of this trial, the American College of Cardiology (ACC) and the American Heart Association (AHA) have updated their guidelines on the primary prevention of cardiovascular disease to recommend that aspirin not be used routinely in patients > 70 years of age.7

CAVEATS

Potential benefit to people at higher risk?

The rate of cardiovascular disease was lower than expected in this overall healthy population, so it is not known if cardiovascular benefits may outweigh the risk of bleeding in a higher-risk population. The trial also didn’t address the potential harms of deprescribing aspirin. Additionally, although aspirin may not be protective for cardiovascular events and may lead to more bleeding, there may be other benefits to aspirin in this patient population that were not addressed by this study.

Continue to: CHALLENGES TO IMPLEMENTATION

 

 

CHALLENGES TO IMPLEMENTATION

Popular beliefs and wide availability may make tide difficult to change

Patients have been told for years to take a daily aspirin to “protect their heart”; this behavior may be difficult to change. And because aspirin is widely available over the counter, patients may take it without their physician’s knowledge.

ACKNOWLEDGEMENT

The PURLs Surveillance System was supported in part by Grant Number UL1RR024999 from the National Center For Research Resources, a Clinical Translational Science Award to the University of Chicago. The content is solely the responsibility of the authors and does not necessarily represent the official views of the National Center For Research Resources or the National Institutes of Health.

References

1. McNeil JJ, Wolfe R, Woods RL, et al. Effect of aspirin on cardiovascular events and bleeding in the healthy elderly. N Engl J Med. 2018;379:1509-1518.

2. Murphy SL, Xu JQ, Kochanek KD, et al. Mortality in the United States, 2017. NCHS Data Brief, no. 328. Hyattsville, MD: National Center for Health Statistics. 2018.

3. Smith SC Jr, Benjamin EJ, Bonow RO, et al. AHA/ACCF secondary prevention and risk reduction therapy for patients with coronary and other atherosclerotic vascular disease: a guideline from the American Heart Association and American College of Cardiology Foundation. Circulation. 2011;124:2458-2473.

4. Gaziano JM, Brotons C, Coppolecchia R, et al. Use of aspirin to reduce risk of initial vascular events in patients at moderate risk of cardiovascular disease (ARRIVE): a randomised, double-blind, placebo-controlled trial. Lancet. 2018;392:1036-1046.

5. Bowman L, Mafham M, Wallendszus K, et al; ASCEND Study Collaborative Group. Effects of aspirin for primary prevention in persons with diabetes mellitus. N Engl J Med. 2018;379:1529-1539. 

6. Bibbins-Domingo K; U.S. Preventive Services Task Force. Aspirin use for the primary prevention of cardiovascular disease and colorectal cancer: U.S. Preventive Services Task Force Recommendation Statement. Ann Intern Med. 2016;164:836-845.

7. Arnett DK, Blumenthal RS, Albert MA, et al. 2019 ACC/AHA Guideline on the Primary Prevention of Cardiovascular Disease: Executive Summary: A Report of the American College of Cardiology/American Heart Association Task Force on Clinical Practice Guidelines. J Amer Coll Cardiol. 2019;74:1376-1414. 

8. American Diabetes Association. Cardiovascular Disease and Risk Management: Standards of Medical Care in Diabetes-2019. Diabetes Care. 2019;42(Suppl 1):S103-S123.

References

1. McNeil JJ, Wolfe R, Woods RL, et al. Effect of aspirin on cardiovascular events and bleeding in the healthy elderly. N Engl J Med. 2018;379:1509-1518.

2. Murphy SL, Xu JQ, Kochanek KD, et al. Mortality in the United States, 2017. NCHS Data Brief, no. 328. Hyattsville, MD: National Center for Health Statistics. 2018.

3. Smith SC Jr, Benjamin EJ, Bonow RO, et al. AHA/ACCF secondary prevention and risk reduction therapy for patients with coronary and other atherosclerotic vascular disease: a guideline from the American Heart Association and American College of Cardiology Foundation. Circulation. 2011;124:2458-2473.

4. Gaziano JM, Brotons C, Coppolecchia R, et al. Use of aspirin to reduce risk of initial vascular events in patients at moderate risk of cardiovascular disease (ARRIVE): a randomised, double-blind, placebo-controlled trial. Lancet. 2018;392:1036-1046.

5. Bowman L, Mafham M, Wallendszus K, et al; ASCEND Study Collaborative Group. Effects of aspirin for primary prevention in persons with diabetes mellitus. N Engl J Med. 2018;379:1529-1539. 

6. Bibbins-Domingo K; U.S. Preventive Services Task Force. Aspirin use for the primary prevention of cardiovascular disease and colorectal cancer: U.S. Preventive Services Task Force Recommendation Statement. Ann Intern Med. 2016;164:836-845.

7. Arnett DK, Blumenthal RS, Albert MA, et al. 2019 ACC/AHA Guideline on the Primary Prevention of Cardiovascular Disease: Executive Summary: A Report of the American College of Cardiology/American Heart Association Task Force on Clinical Practice Guidelines. J Amer Coll Cardiol. 2019;74:1376-1414. 

8. American Diabetes Association. Cardiovascular Disease and Risk Management: Standards of Medical Care in Diabetes-2019. Diabetes Care. 2019;42(Suppl 1):S103-S123.

Issue
The Journal of Family Practice - 69(3)
Issue
The Journal of Family Practice - 69(3)
Page Number
E16-E18
Page Number
E16-E18
Publications
Publications
Topics
Article Type
Display Headline
Aspirin, Yes, for at-risk elderly—but what about the healthy elderly?
Display Headline
Aspirin, Yes, for at-risk elderly—but what about the healthy elderly?
Sections
PURLs Copyright
Copyright © 2020. The Family Physicians Inquiries Network. All rights reserved.
Inside the Article

PRACTICE CHANGER

Do not prescribe aspirin for primary prevention of cardiovascular disease in your elderly patients. Aspirin does not improve cardiovascular outcomes and it significantly increases the risk of bleeding events.

STRENGTH OF RECOMMENDATION

B: Based on a single randomized controlled trial.

McNeil JJ, Wolfe R, Woods RL, et al. Effect of aspirin on cardiovascular events and bleeding in the healthy elderly. N Engl J Med. 2018;379:1509-1518.1

Disallow All Ads
Content Gating
No Gating (article Unlocked/Free)
Alternative CME
PubMed ID
32289133
Disqus Comments
Default
Use ProPublica
Hide sidebar & use full width
render the right sidebar.
Article PDF Media
Image
Teambase ID
180016F4.SIG
Disable zoom
Off
Media Files
Image
Disable zoom
Off
Media Folder

What is the best treatment for wrist ganglion cysts?

Article Type
Changed
Wed, 02/12/2020 - 08:26
Display Headline
What is the best treatment for wrist ganglion cysts?

EVIDENCE SUMMARY

A 2015 meta-analysis of 35 studies (7 RCTs, 6 cohort studies, 22 case series) of 2239 wrist ganglion cysts examined the recurrence rate of cysts after common treatments.1 Two RCTs and 4 cohort studies compared open surgical excision with aspiration with or without corticosteroid injection.

The RCTs found significantly lower recurrence rates following open surgical excision compared with aspiration (2 trials; 60 cysts; risk ratio [RR] = 0.24; 95% confidence interval [CI], 0.08-0.71; number needed to treat [NNT] = 3). The cohort studies likewise found markedly less recurrence of cysts after open surgical excision than aspiration (4 studies; 461 cysts; RR = 0.42; 95% CI, 0.21-0.85; NNT = 4). Recurrence rates didn’t ­differ between aspiration and observation (2 cohort studies; 209 cysts; RR = 0.99; 95% CI, 0.77-1.28).

Overall, the RCT evidence was of moderate quality because of a lack of significant heterogeneity, and the cohort evidence was graded as very low quality because of heterogeneity.

 

More evidence of lower recurrence with surgical excision

A 2014 prospective RCT, not included in the foregoing meta-analysis because it was published after the search date, compared ganglion cyst recurrence at 6 months for 2 groups: one group received aspiration accompanied by corticosteroid injection and the other had surgical treatment.2 The trial included 173 patients ages 16 to 47 years with 187 ganglia of the wrist, ankle, or knee (143 wrist ganglia). Patients were excluded if they had a history of recurrent ganglia, prior treatment of ganglia, nearby joint injury, bleeding disorders, pregnancy, compound palmar ganglion, ganglion near arteries, infected ganglion, ganglion associated with arthritic disease, or ganglion measuring < 5 mm in size.

Patients were allowed to choose aspiration with corticosteroid injection or surgical excision. The aspiration group (143 ganglia: 106 wrist, 21 ankle, 16 knee) underwent aspiration using a 19-gauge needle and 10-mL syringe followed by injection of 0.25 to 1.0 mL of triamcinolone acetonide. Aspiration and injection were repeated if indicated at either 6 weeks or 3 months. The surgical excision group comprised 44 ganglia: 37 wrist and 7 ankle.

The success rate at 6 months following aspiration with corticosteroid injection was 81% compared with 93% after surgical excision (NNT = 8). Surgical treatment was associated with significantly less recurrence than aspiration and injection (7% vs 19%; P < .028).

Patients report symptomatic improvement after aspiration

A 2015 retrospective case series assessed the long-term outcomes of 21 patients following aspiration of wrist ganglia.3 The patients, who were 41 to 49 years of age, each had a single wrist ganglion that was treated with aspiration between 2001 and 2011 by a single surgeon. Mean time to follow-up was 6.3 years. Outcomes reviewed included recurrence, satisfaction, and improvement in symptoms—pain, function, range of motion, and appearance—using a 1 to 5 Likert scale (1 = significantly worse; 5 = significantly improved).

Continue to: Overall, 52.4% of patients...

 

 

Overall, 52.4% of patients experienced recurrence of their ganglia. However, 95% expressed satisfaction with treatment independent of recurrence. Mean symptom scores improved from baseline for pain (4.1 points), function (3.9 points), range of motion (3.8 points), and appearance (4.1 points). Improvements in all symptoms were independent of recurrence.

Aspiration plus steroids results in 43% recurrence rate

A 2015 prospective study examined the recurrence rate at 1 year after therapy in 30 patients, ages 15 to 55 years, with a wrist ganglion treated by aspiration and steroid injection.4 Patients chose aspiration and steroid injection with 40 mg/mL methyl-­prednisolone acetate over reassurance or surgical intervention. The recurrence rate at 1-year follow-up was 43.3% (13 patients).

Editor’s takeaway

Surgical excision of ganglion cysts results in fewer recurrences than aspiration. However, moderately high-quality evidence shows that both methods help most patients.

References

1. Head L, Gencarelli JR, Allen M, et al. Wrist ganglion treatment: systematic review and meta-analysis. J Hand Surg Am. 2015;40:546-553.e8.

2. Latif A, Ansar A, Butt MQ. Treatment of ganglions; a five year experience. J Pak Med Assoc. 2014;64:1278-1281.

3. Head L, Allen M, Boyd KU. Long-term outcomes and patient satisfaction following wrist ganglion aspiration. Plast Surg (Oakv). 2015;23:51-53.

4. Hussain S, Akhtar S, Aslam V, et al. Efficacy of aspiration and steroid injection in treatment of ganglion cyst. PJMHS. 2015;9:1403-1405.

Article PDF
Author and Disclosure Information

Corey Lyon, DO
Stephanie V. Eldred, MD

University of Colorado Family Medicine Residency, Denver

Kristen DeSanto, MSLS, MS, RD, AHIP
University of Colorado Health Sciences Library, Denver

DEPUTY EDITOR
Rick Guthmann, MD, MPH

Advocate Illinois Masonic Family Medicine Residency, Chicago

Issue
The Journal of Family Practice - 69(1)
Publications
Topics
Page Number
E23-E24
Sections
Author and Disclosure Information

Corey Lyon, DO
Stephanie V. Eldred, MD

University of Colorado Family Medicine Residency, Denver

Kristen DeSanto, MSLS, MS, RD, AHIP
University of Colorado Health Sciences Library, Denver

DEPUTY EDITOR
Rick Guthmann, MD, MPH

Advocate Illinois Masonic Family Medicine Residency, Chicago

Author and Disclosure Information

Corey Lyon, DO
Stephanie V. Eldred, MD

University of Colorado Family Medicine Residency, Denver

Kristen DeSanto, MSLS, MS, RD, AHIP
University of Colorado Health Sciences Library, Denver

DEPUTY EDITOR
Rick Guthmann, MD, MPH

Advocate Illinois Masonic Family Medicine Residency, Chicago

Article PDF
Article PDF

EVIDENCE SUMMARY

A 2015 meta-analysis of 35 studies (7 RCTs, 6 cohort studies, 22 case series) of 2239 wrist ganglion cysts examined the recurrence rate of cysts after common treatments.1 Two RCTs and 4 cohort studies compared open surgical excision with aspiration with or without corticosteroid injection.

The RCTs found significantly lower recurrence rates following open surgical excision compared with aspiration (2 trials; 60 cysts; risk ratio [RR] = 0.24; 95% confidence interval [CI], 0.08-0.71; number needed to treat [NNT] = 3). The cohort studies likewise found markedly less recurrence of cysts after open surgical excision than aspiration (4 studies; 461 cysts; RR = 0.42; 95% CI, 0.21-0.85; NNT = 4). Recurrence rates didn’t ­differ between aspiration and observation (2 cohort studies; 209 cysts; RR = 0.99; 95% CI, 0.77-1.28).

Overall, the RCT evidence was of moderate quality because of a lack of significant heterogeneity, and the cohort evidence was graded as very low quality because of heterogeneity.

 

More evidence of lower recurrence with surgical excision

A 2014 prospective RCT, not included in the foregoing meta-analysis because it was published after the search date, compared ganglion cyst recurrence at 6 months for 2 groups: one group received aspiration accompanied by corticosteroid injection and the other had surgical treatment.2 The trial included 173 patients ages 16 to 47 years with 187 ganglia of the wrist, ankle, or knee (143 wrist ganglia). Patients were excluded if they had a history of recurrent ganglia, prior treatment of ganglia, nearby joint injury, bleeding disorders, pregnancy, compound palmar ganglion, ganglion near arteries, infected ganglion, ganglion associated with arthritic disease, or ganglion measuring < 5 mm in size.

Patients were allowed to choose aspiration with corticosteroid injection or surgical excision. The aspiration group (143 ganglia: 106 wrist, 21 ankle, 16 knee) underwent aspiration using a 19-gauge needle and 10-mL syringe followed by injection of 0.25 to 1.0 mL of triamcinolone acetonide. Aspiration and injection were repeated if indicated at either 6 weeks or 3 months. The surgical excision group comprised 44 ganglia: 37 wrist and 7 ankle.

The success rate at 6 months following aspiration with corticosteroid injection was 81% compared with 93% after surgical excision (NNT = 8). Surgical treatment was associated with significantly less recurrence than aspiration and injection (7% vs 19%; P < .028).

Patients report symptomatic improvement after aspiration

A 2015 retrospective case series assessed the long-term outcomes of 21 patients following aspiration of wrist ganglia.3 The patients, who were 41 to 49 years of age, each had a single wrist ganglion that was treated with aspiration between 2001 and 2011 by a single surgeon. Mean time to follow-up was 6.3 years. Outcomes reviewed included recurrence, satisfaction, and improvement in symptoms—pain, function, range of motion, and appearance—using a 1 to 5 Likert scale (1 = significantly worse; 5 = significantly improved).

Continue to: Overall, 52.4% of patients...

 

 

Overall, 52.4% of patients experienced recurrence of their ganglia. However, 95% expressed satisfaction with treatment independent of recurrence. Mean symptom scores improved from baseline for pain (4.1 points), function (3.9 points), range of motion (3.8 points), and appearance (4.1 points). Improvements in all symptoms were independent of recurrence.

Aspiration plus steroids results in 43% recurrence rate

A 2015 prospective study examined the recurrence rate at 1 year after therapy in 30 patients, ages 15 to 55 years, with a wrist ganglion treated by aspiration and steroid injection.4 Patients chose aspiration and steroid injection with 40 mg/mL methyl-­prednisolone acetate over reassurance or surgical intervention. The recurrence rate at 1-year follow-up was 43.3% (13 patients).

Editor’s takeaway

Surgical excision of ganglion cysts results in fewer recurrences than aspiration. However, moderately high-quality evidence shows that both methods help most patients.

EVIDENCE SUMMARY

A 2015 meta-analysis of 35 studies (7 RCTs, 6 cohort studies, 22 case series) of 2239 wrist ganglion cysts examined the recurrence rate of cysts after common treatments.1 Two RCTs and 4 cohort studies compared open surgical excision with aspiration with or without corticosteroid injection.

The RCTs found significantly lower recurrence rates following open surgical excision compared with aspiration (2 trials; 60 cysts; risk ratio [RR] = 0.24; 95% confidence interval [CI], 0.08-0.71; number needed to treat [NNT] = 3). The cohort studies likewise found markedly less recurrence of cysts after open surgical excision than aspiration (4 studies; 461 cysts; RR = 0.42; 95% CI, 0.21-0.85; NNT = 4). Recurrence rates didn’t ­differ between aspiration and observation (2 cohort studies; 209 cysts; RR = 0.99; 95% CI, 0.77-1.28).

Overall, the RCT evidence was of moderate quality because of a lack of significant heterogeneity, and the cohort evidence was graded as very low quality because of heterogeneity.

 

More evidence of lower recurrence with surgical excision

A 2014 prospective RCT, not included in the foregoing meta-analysis because it was published after the search date, compared ganglion cyst recurrence at 6 months for 2 groups: one group received aspiration accompanied by corticosteroid injection and the other had surgical treatment.2 The trial included 173 patients ages 16 to 47 years with 187 ganglia of the wrist, ankle, or knee (143 wrist ganglia). Patients were excluded if they had a history of recurrent ganglia, prior treatment of ganglia, nearby joint injury, bleeding disorders, pregnancy, compound palmar ganglion, ganglion near arteries, infected ganglion, ganglion associated with arthritic disease, or ganglion measuring < 5 mm in size.

Patients were allowed to choose aspiration with corticosteroid injection or surgical excision. The aspiration group (143 ganglia: 106 wrist, 21 ankle, 16 knee) underwent aspiration using a 19-gauge needle and 10-mL syringe followed by injection of 0.25 to 1.0 mL of triamcinolone acetonide. Aspiration and injection were repeated if indicated at either 6 weeks or 3 months. The surgical excision group comprised 44 ganglia: 37 wrist and 7 ankle.

The success rate at 6 months following aspiration with corticosteroid injection was 81% compared with 93% after surgical excision (NNT = 8). Surgical treatment was associated with significantly less recurrence than aspiration and injection (7% vs 19%; P < .028).

Patients report symptomatic improvement after aspiration

A 2015 retrospective case series assessed the long-term outcomes of 21 patients following aspiration of wrist ganglia.3 The patients, who were 41 to 49 years of age, each had a single wrist ganglion that was treated with aspiration between 2001 and 2011 by a single surgeon. Mean time to follow-up was 6.3 years. Outcomes reviewed included recurrence, satisfaction, and improvement in symptoms—pain, function, range of motion, and appearance—using a 1 to 5 Likert scale (1 = significantly worse; 5 = significantly improved).

Continue to: Overall, 52.4% of patients...

 

 

Overall, 52.4% of patients experienced recurrence of their ganglia. However, 95% expressed satisfaction with treatment independent of recurrence. Mean symptom scores improved from baseline for pain (4.1 points), function (3.9 points), range of motion (3.8 points), and appearance (4.1 points). Improvements in all symptoms were independent of recurrence.

Aspiration plus steroids results in 43% recurrence rate

A 2015 prospective study examined the recurrence rate at 1 year after therapy in 30 patients, ages 15 to 55 years, with a wrist ganglion treated by aspiration and steroid injection.4 Patients chose aspiration and steroid injection with 40 mg/mL methyl-­prednisolone acetate over reassurance or surgical intervention. The recurrence rate at 1-year follow-up was 43.3% (13 patients).

Editor’s takeaway

Surgical excision of ganglion cysts results in fewer recurrences than aspiration. However, moderately high-quality evidence shows that both methods help most patients.

References

1. Head L, Gencarelli JR, Allen M, et al. Wrist ganglion treatment: systematic review and meta-analysis. J Hand Surg Am. 2015;40:546-553.e8.

2. Latif A, Ansar A, Butt MQ. Treatment of ganglions; a five year experience. J Pak Med Assoc. 2014;64:1278-1281.

3. Head L, Allen M, Boyd KU. Long-term outcomes and patient satisfaction following wrist ganglion aspiration. Plast Surg (Oakv). 2015;23:51-53.

4. Hussain S, Akhtar S, Aslam V, et al. Efficacy of aspiration and steroid injection in treatment of ganglion cyst. PJMHS. 2015;9:1403-1405.

References

1. Head L, Gencarelli JR, Allen M, et al. Wrist ganglion treatment: systematic review and meta-analysis. J Hand Surg Am. 2015;40:546-553.e8.

2. Latif A, Ansar A, Butt MQ. Treatment of ganglions; a five year experience. J Pak Med Assoc. 2014;64:1278-1281.

3. Head L, Allen M, Boyd KU. Long-term outcomes and patient satisfaction following wrist ganglion aspiration. Plast Surg (Oakv). 2015;23:51-53.

4. Hussain S, Akhtar S, Aslam V, et al. Efficacy of aspiration and steroid injection in treatment of ganglion cyst. PJMHS. 2015;9:1403-1405.

Issue
The Journal of Family Practice - 69(1)
Issue
The Journal of Family Practice - 69(1)
Page Number
E23-E24
Page Number
E23-E24
Publications
Publications
Topics
Article Type
Display Headline
What is the best treatment for wrist ganglion cysts?
Display Headline
What is the best treatment for wrist ganglion cysts?
Sections
PURLs Copyright
Evidence-based answers from the Family Physicians Inquiries Network
Inside the Article

EVIDENCE-BASED ANSWER:

Open surgical excision of wrist ganglion cysts is associated with a lower recurrence rate than aspiration with or without corticosteroid injection (strength of recommendation [SOR]: B, systematic review of randomized clinical trials [RCTs] and observational trials and RCT).

Even though the recurrence rate with aspiration is about 50%, most patients are satisfied with aspiration and report a decrease in symptoms involving pain, function, and range of motion (SOR: B, individual cohort and case series).

Disallow All Ads
Content Gating
No Gating (article Unlocked/Free)
Alternative CME
PubMed ID
32017840
Disqus Comments
Default
Use ProPublica
Hide sidebar & use full width
render the right sidebar.
Article PDF Media
Image
Teambase ID
18001640.SIG
Disable zoom
Off